当前位置:首页>条据书信>政府绩效与公众信任(热门20篇)

政府绩效与公众信任(热门20篇)

时间:2024-01-05 12:34:02 作者:紫薇儿政府绩效与公众信任(热门20篇)

通过分析范文范本,我们可以学习到不同类型作文的结构和写作思路。为了方便大家了解总结的写作规范,我们为你准备了一些范文供参考。

以“政府绩效与公众信任”为撰写小论文范文

公众是指公共关系的主体成员面临共同问题、共同利益和共同要求的社会群体[。以下是为大家整理的关于,欢迎品鉴!

摘要:政府信任反映了居民对政府的认可程度,是政府政策执行的基础。汶川的地震灾区居民政府信任水平没有随着灾区公共服务水平的提高得到相应的提升,通过运用“汶川地震十周年居民生活情况调查”数据,实证分析探究了人际信任、政府绩效与政府信任之间的关系,研究结果显示,政府绩效、人际信任均对政府信任产生显著的影响。

关键词:灾区居民;政府信任;人际信任;政府绩效。

1问题的提出。

2008年汶川8.0级特大地震给震区带来了巨大的财产和生命安全损失,我国政府在救灾所表现出的强大动员能力、应急能力也成为世界关注的焦点。尤其通过政府的各项震后恢复政策,使得汶川灾区的社会经济发展获得历史性机遇,基础公共社会高水平重构,产业发展能力也持续提升,百姓生活水平相对于震前有较大幅度提高。

较高的政府信任度会有效降低政府运行的成本,于政府及公民都是有利的。而政府信任会受一系列因素的影响,在突发事件发生的关键时期政府政策执行对政府信任的影响更为显著。政府信任作为公民对政府的主观评价,受诸多因素的影响。现有的研究中不乏对政府影响因素的探讨,程倩认为,政府信任是政府合法性的本质内涵,政府信任关系是公民对政府及其政策执行过程中各相关要素、政策执行过程以及政策执行结果的心理预期。在二者的关系解释上,政府高校的运作过程以及较好的绩效水平可以带来良好的政府信任水平,两者呈正相关关系(程倩,2011)。缪婷婷认为,影响政府信任的因素具体包括政府作为、政府绩效以及新闻媒体的兴起(缪婷婷,2016)。在关键时期,政府政策对居民的政府信任影响更为显著。在突发事件中,公民对政府的信任感往往受领导者的价值观以及政府政策执行情况的影响,政府信任与政府及其工作人员对受灾民众的真诚关怀和激励呈显著正相关的影响(王璐,2008)。

因此,本文在梳理了现有研究成果的基础上,基于“汶川地震灾区灾后重建十周年状况调查”数据,对影响灾区居民政府信任度的影响因素进行实证分析。

2文献综述及研究假设。

2.1人际信任。

在文化主义视角下,人际信任会对政府信任产生显著的影响。高巍通过实证研究证实了人际信任对政府信任的影响:民众越容易相信他人,则更容易信任政府(高巍,2015)。胡荣等学者研究发现个人的信任水平对政府信任度会产生显著影响(胡荣,2007)。陈天祥构建了包括村内信任和社会信任的影响因素模型,验证了民众个人的信任水平对政府信任会产生显著的影响(陈天祥,2017)。人际信任程度越高,越有利于民众与政府的良性互动(刘人宁,2018)。人际信任变量作为文化路径中重要的解释变量,对居民政府信任的影响不言而喻,因此本文将其作为自变量的指标之一。据此,提出假设1:

假设1:人际信任度越高,政府信任度越高;人际信任度越低,政府信任度越低。

2.2政府绩效。

制度主义路径认为政府绩效很大程度上影响人们对政府信任度的判定,政府绩效涵盖政治、经济、环境保护等多方面的绩效。许伟通过研究发现,政府绩效变量对政府信任会产生显著的影响,同时验证了政府绩效与政府信任之间的正相关关系。杨培鸿(2007)通过实证研究证明了公共服务绩效作为关键性的影响因素,对公民政府信任评价起着重要的作用。陈天祥在研究中国乡镇政府的政府信任情况时也将政府绩效作为自变量进行分析(陈天祥等,2017)。由此,提出假设2:

假设2:政府绩效水平与政府信任呈正相关关系。

3研究方法。

3.1数据来源。

本研究使用数据来源于国家科技部中国发展战略研究院联同西南交通大学开展的“汶川十周年居民生活情况调查”数据,调查范围包括四川省5市/州30县/区共198个行政村/社区,共抽取了30個受灾县中的4950户住户,共完成家户问卷3826份,完成个人问卷3751份。

3.2变量的操作化。

因变量:政府信任变量。

在“汶川地震十周年居民生活状况调查”问卷中,关于政府信任被划分为对中央政府、省政府、县/市政府、乡/镇/街道四个层级政府的信任,选项分别为“完全信任”、“比较信任”、“不太信任”、“根本不信任”,对四个信任度赋值为1-4,1代表“完全信任”,4代表“根本不信任”,以此类推。通过探索性因子分析提取出“政府信任”因子。

自变量一:人际信任。

“汶川地震十周年居民生活状况调查”问卷中,对人际信任变量操作为题目——对“家人”、“住在你周围的人”、“市场上的商人/买卖人”、“外地人”、“村干部”、“警察”、“医生”、“法官”共8种不同类型群体信任情况,答案与测量“政府信任”的答案一致。通过探索性因子分析,根据指标特征,提取出三个因子,分别为“权威信任”、“社会信任”、“邻里信任”。

自变量二:政府绩效。

本研究中关于政府绩效感知被测量为居民对“贫困问题”、“贫富差距”、“失业情况”、“空气污染”、“水污染”、“缺水问题”、“孩子上学不方便”的严重程度评价,答案包括“特别严重”、“比较严重”、“不太严重”、“不成问题”四个程度的测量,从“特别严重”至“不成问题”分别赋值为1-4。通过因子分析,提取3个公因子为“人民生活”、“环境保护”、“公共服务”。

4研究分析。

4.1相关性分析。

通过相关性分析可知,居民人际信任的三个维度与政府信任的相关显著性均为0.000,小于0.05,均呈显著性相关关系,表明民众个人信任水平会显著影响民众对政府的信任,其中权威人群信任维度的得分系数最高,为0.665。

该部分基于因子分析和相关性分析的基础上,采取线性回归方法进一步验证各影响因素与政府信任的关系,建立了2个回归模型,对影响灾区居民政府信任的影响因素进行实证分析。

模型1中检验的是人际信任与政府信任之间的关系。在研究人际信任与政府信任的回归中国,采用逐步回归法,将人际信任的3个公因子作为自变量,政府信任因子作为因变量进行分析。根据选取自变量的规则p0.05,共选择了3个变量,包括权威信任、邻里信任以及社会信任。模型1的相关系数为0.678,调整后的r方为0.459。因此,最终建立的模型1包含权威信任,邻里信任,社会信任3个影响因素。通过anova表考察模型是否有效,该模型回归的显著性水平均显示p0.05,表明回归程度显著,可以认为该模型有效。从回归系数表可知,权威信任、邻里信任以及社会信任的回归系数分别为0.616、0.161、0.085,均大于0,说明权威信任、邻里信任以及社会信任程度越高,居民对政府信任的程度越高。t检验中,权威信任、邻里信任、社会信任和常数项的显著性p值都小于0.05,因而均有显著意义。

模型2检验政府绩效与政府信任之间的关系。在研究政府绩效与政府信任的回归中,采用逐步回归法,将绩效感知的3个因子作为自变量,政府信任因子作为因变量进行分析。根据选取自变量的规则p0.05,共选择了3个变量,包括制环境保护、公共服务以及人民生活三个维度的变量,建立模型2。根据模型2可知,环境保护、公共服务以及人民生活的回归系数分别为-0.179、-0.118、-0.109,絕对值均大于0,回归系数为负,说明环境保护、公共服务以人民生活各方面问题的严重程度越高,居民对政府信任的程度越低,即政府绩效水平与政府信任呈正相关关系,政府绩效越好,居民政府信任度越高。t检验中,环境保护、公共服务、人民生活和常数项的显著性p值都小于0.05,因而均有显著意义。

5研究结论。

本章首先在因子分析以及验证各影响因素与因变量存在相关性的基础上进行线性回归分析,进而验证了相关假设。具体结论如下:

(1)假设1得到验证:人际信任度越高,政府信任度越高;人际信任度越低,政府信任度越低。通过分析发现,人际信任对政府信任存在正相关关系。人际信任包括权威信任、社会信任及邻里信任。其中对权威群体的信任是本研究中影响政府信任最显著的因素,人际信任对居民的政府信任产生影响最为显著,其次是邻里信任,社会信任对政府信任的影响程度最低。

人际信任一直都被当作影响政府信任的重要因素,本研究通过实证分析也得出相同的结论。如何通过提高居民的人际信任水平从而提高政府信任水平是当前应该思考的问题。从本文的研究中可知,人际信任中居民对警察、医生、法官、村干部这类具有一定社会公信力的群体有着较高的信任水平,这与我国传统的信任特征有关,人们对这部分人群持有敬畏之心。但是,我们也可以看到在当今社会中,这部分过去极具公信力的人群也面临公信力下降的困境。因此,需提高这部分人群的服务能力,提高公共服务水平,更好地为人民服务。人际信任中得分最低的是社会信任,即对陌生人的信任水平的解释。在人际关系越来越“冷漠”的今天,社会信任显得尤为重要。近年出现的“扶不扶”、“小悦悦”等问题和事件极大的打击了人们的信任感,使得人们对外界保持极高的警惕心,人们越发地变得更为“冷漠”。因此,大力弘扬社会主义核心价值观,严厉打击破坏社会和谐稳定的因素。

(2)假设2得到验证:政府绩效越好,公民对政府的信任越强。政府绩效越差,公民对政府的信任越低。通过回归分析发现,居民对政府绩效的感知对政府信任产生显著的影响。本研究从政府在公共服务、环境保护、人民生活三方面的绩效成果来考察政府绩效对政府信任的影响,三个维度中环境保护绩效对政府信任的影响最为显著。

政府绩效是政府行为的直观体现,政府绩效的好坏直接影响民众对政府的信任水平,提高政府工作绩效是我国政府一直努力的方向。本研究关于灾区政府绩效与政府信任的关系中,人民生活、公共服务、环境保护方面的绩效对政府信任产生显著的影响。因此,提高灾区居民政府信任需要提高政府绩效,转变政府职能,提高政府的公共服务能力和水平。

参考文献。

[2]缪婷婷.政府绩效对政府信任的影响[d].苏州:苏州大学,2016.

[4]高巍.当前中国居民政府信任的区域比较[d].济南:山东大学,2015.

[7]刘人宁.基层政府信任的影响因素研究[d].哈尔滨:黑龙江省社会科学院,2018.

[8]许伟.我国当代政府信任的比较研究[d].武汉:武汉大学,2014.

摘要:自从分税制模式实施后,出现了中央与地方政府财权和事权不匹配的问题。2008年金融危机对我国的经济影响非常大,于是我国扩大内需,推出了4万亿元的投资计划。很多地方政府融资平台开始迅速扩张,扩大自己的融资规模。然而这种现象也产生了负面影响,各地政府实施不同的项目,导致政府债务规模不断扩大,政府债务风险严重。鉴于此,对政府财务风险进行分析,并提出解决政府财务风险的对策。

关键词:政府债务;财务风险;防范对策。

我国从1994年开始进行分税制改革,导致政府的财权和事权不统一,政府支出的责任变更非常频繁,但是没有相应的财力作为支撑。2008年,我国为了应对金融危机,提出了4万亿政策,结合中央政府的投资计划,地方政府开展了各种形式的融资。大规模的融资导致债务隐性化,债务风险非常严峻。在党的十九大报告中,明确提出要防范各类风险。在风险防控中,政府的债务风险防控是重点,从而有效地预防金融风险的发生。

1.政府财权事权与支出责任不符。自从1994年我国实施分税制改革后,税种划分成中央税种和地方税种。地方的事权不断增多,导致地方性政府财政支出越来越多。随着城镇化进程的加快,居民对公共产品的需求量非常大,政府需要更多的资金。地方政府为了进行基础设施建设,他们每年都要融资,产生债务风险。地方政府的财政收入非常有限,受到中央强化房地产调控的影响,在转移支付环节中存在各类问题,不能结合地方政府的实际需求。在增量调节环节,地方之间的贫富差距非常大。地方政府的事权不断增加,但是财权非常有限。地方政府体制内的收入不能弥补财政支出,导致地方政府的预算外收入增加,他们只能通过举债的方式弥补财政不足。

2.政府融资不够规范。政府在融资环节中,产生非常大的随意性。地方政府常常通过发行债券的方式,但是资金还是不能满足需求。于是,地方政府开始建立各类融资平台,在融资环节中产生盲目性和随意性。政府的融资运作效率不高,融资的成本非常高,导致政府投资效率不佳。在融资平台建设中,资金不足。政府会通过各类方式补充资金,通过挪用年度预算等方式,甚至出现虚假注资的方式。政府在融資方面,相关的管理体制存在缺陷,很多融资平台的建设没有相关的监督措施加以辅助,采用多头管理的方式。融资监督机制尚未建立,政府在融资环节中,缺乏法律和公众的监督,导致商业腐败的产生。

3.政府债务管理机制不够健全。政府的债务管理机构不完善,尽管我国政府已经积累了很多政府性债务,但是债务管理机构尚不完善。债务资金使用不够规范,有些地方政府的债务资金不能纳入到预算管理环节,债务资金不能充分进行基础设施建设。债务资金管理效果不好,不能进行全方位的预算管理。政府债务领域的信息不够透明,地方政府的融资情况不能及时地回报给人大常委,信息不公开,导致地方性债务的口径不能完全的纳入到预算管理系统中。债务管理的法制建设比较落后,不能结合债务管理建立法律体系。尽管我国制定了《预算法》等,对政府的举债行为加以约束,但随着社会经济的发展,政府投资还是越来越多,已经超过政府的财力。

1.提升思想认识,强化债务管理。政府应强化风险认识,防止债务风险扩散为经济风险,要正确地认识债务风险问题,纵观大局,做到未雨绸缪。政府应树立忧患意识,认识到抵御债务风险的紧迫性,也要认识到政府举债对当地经济发展产生的影响,确保偿债有序。强化底线意识,底线意识在一定程度上是底线思维的应用,其与风险意识是相辅相成的,政府要在对现实情况进行分析的基础上,提高警惕。将国家法律作为“红线”,做好风险的防范准备工作。政府还要提升自身的法治意识,杜绝违法违规的举债行为。有些政府的领导干部,他们的法治意识比较弱,对于一些惩戒视而不见。因此,政府应树立法治意识,规范举债行为,维护法律的权威。充分了解政府职能的权限,掌握政府和市场之间的关系,强化政府职能的转变,发挥市场这只无形手的资源配置作用。

2.推进体制机制改革。合理划分财权和事权,中央政府和地方政府应该明确财政事权和支出责任,有效地抑制政府债务规模。结合权、责和利相统一的原则,各地区的财权和事权应得到明确,制定财权事权调整机制,形成完整和清晰的财权和事权清单,从而提升政府对于财权和事权改革的可操作性。建立相应的配套措施,完善相关的改革协同配套方案,提升转移支付制度的可操作性。进行干部任用制度的改革,完善债务监督考核问责制度。对考核的方法进行改进,抓住基础和发展的根基,充分发挥潜能,改善民生,促进社会进步。完善政府债务考核和问责制度,建立公共服务为中心的理念。在新政策背景下,完善债务监督制度,对金融机构进行科学的监管工作,有效地杜绝违法违规操作,强化责任人制度。对政府债券资金进行全面管理,明确责任主体和财务主体。提升债务信息的透明度,通过政府政务网站的方式,对债务限额明确,建立债务限额报告制度。通过财政预算的方式,对预算情况进行分析。

3.提升政府债务风险管理能力。建立政府债务管理小组,发挥其职能。对政府债务的借、用和还阶段进行监督,进行债务管理工作的指导,对于债务风险要实时掌握。当风险发生后,应采取必要的干预措施。完善政府债务的预算管理工作,分别对政府债务收入、支出、付息和还本进行预算,防止超预算举债的发生。各地的人大应发挥审议作用,充分履行监督职能。提升风险化解的能力,在政府债务管理中,隐性债务管理存在一定的难度。因此,在债务管理中,要对各类风险进行评估,出具评估报告。有些政府债务风险防范能力比较差,风险防控机制比较落后。因此,要提升政府债务风险化解能力,抓住风险产生的主要因素。

政府债务风险是重大风险的类型之一,因此,我国应有效地防范和化解政府财务风险,促进全面小康社会的建设。同时,政府应提升债务风险的化解能力,树立危机意识。

参考文献:

[2]胡才龙,魏建国.多任务委托代理模型下地方政府债务管理激励契约设计——基于省级面板数据的实证检验[j].审计与经济研究,2019,(5):118-127.

摘要:本文主要探究政府绩效对城市居民政府信任程度的影响,对我国现阶段的政府工作进行分析,希望能够提升城市居民对政府的信任程度。

关键词:政府绩效;

政府信任;

城市居民。

而政府代理公众行使行政权利,并通过履行相应的职能获得相关利益。因此,政府的信任是建立在委托、代理关系之上,一旦政府无法满足公众的期望,就会出现信任危机,从而导致两者之间原有的和谐关系破裂,甚至威胁到社会和国家的稳定。

政府绩效视角,主要是通过对政府的表现进行评估,分析政府信任的原因,即确定政府是否值得信任。个人对政府的信任程度受到政府行政能力、经济总体形势、政策执行等多方面的影响。在国外的研究中,将公众不信任政府的原因归纳为以下几点:政府工作效率低下、公款浪费严重、政策执行错误等原因。

政府绩效是影响政府政治信任程度的关键所在。政府绩效主要的参考标准是政府在社会经济管理过程中的行政结果、行政效益、行政效能,以及对政府在行使职能的过程中的管理能力进行研究[1]。政府的绩效是政府执政能力的表现,主要分为社会效益、经济效益和政治效益是三个方面。从理论上分析,政府信任程度对政府工作的顺利运作具有重大意义,因此它也是政治生活中的重要组成部分。

在我国城市居民中,首先城市居民将政府看作是一个统一的整体。城市居民对地方政府的评价相对较高,对中央政府和省政府的评价相对较低。虽然城市居民对中央政府的评价略低,这也体现出地方政府对城市居民日常生活的作用和影响,表明了城市居民对政府信任的理性。其次政府的绩效能够有效提升政府的信任程度,通过政府决策的透明性、依法治国的执行力度、贫富差距的改善情况、社会治安的管理情况,都会对政府信任程度有所加分。因此对于地方政府而言,城市居民的信任程度需求明显对政府绩效提出了更高的要求,只有这样,才能从根本上提升城市居民对政府的信任程度。

结束语。

随着改革开放的深入,我国政府在经济建设方面取得了巨大的成就,但是由于目前我国正处于社会转型期,在经济发展的同时也伴随着贫富差距加大、社会矛盾激化等多种民生问题,由于住房、医疗、教育等问题未能得到有效的解决。这样不仅严重削弱了政府的执政能力,降低了行政效率,也在一定程度上影响了居民对政府的信任程度,所以在一定程度上阻碍了到国家和社会的良性发展。因此,要求政府必须通过有效措施提升政府绩效,为国家转型提供必要的助力和保证,确保服务型政府的构建。

参考文献:

政府业绩评价与传统政府部门管理方法不同,是一种新的管理模式,体现了时代发展对政府行政体制改革的要求。其中,第三方评价作为政府业绩管理的重要机制,能够有效地进行外部平衡,弥补传统政府自我评价的缺陷,在促进我国服务型政府变革方面发挥了不可或缺的推进作用。从西方国家实施第三者评价的经验来看,第三者是指与被评价对象和服务对象独立的第三者,由于第三者和被评价对象(第一者)、被服务对象(第二者)没有行政所属关系和利益关系,也被称为独立的第三者。在西方国家,政府业绩第三者的评价多由一些专业的非政府组织(ngo)作为评价机构。这些非政府组织可以保证第三者的独立性、专业性和权威性要求。

从实践层面看,中国政府业绩的传统评价模式以内部进行的自我评价为主,成为运动员和裁判员,评价过程关闭,评价效果差,影响政府的信赖力。近年来大力发展的政府业绩第三方评价,有效提高政府的信赖力,有助于完善政府业绩监督机制。我国从上世纪90年代初开始重视政府业绩评价,从西方参考经验,从90年代中期开始进行政府业绩评价的理论研究,至今为止。

止,基于我国具体情况的政府绩效第三方评估也有了十几年的实践经验。在这一过程中也发现了一些问题。因此,针对我国的实际情况和实际问题,分析政府绩效第三方评估的未来发展趋势,能够有助于丰富相关理论,引导政府绩效第三方评估更好地发展。

第三方评估与传统的政府自我评估相比,不但具有客观性以及独立性,还具有公正性和专业性等优势,能够有力改变政府部门“既当运动员又当裁判员”的传统局面,但是在实践过程中,我国政府绩效第三方评估的优势并没有得到充分发挥,第三方评估在实践过程中还面临着一些亟待解决的问题。

(一)独立性问题。

政府绩效第三方评估需要做到公平公正,首先应该得到保障的就是“独立性”。然而在我国,由于相关法制尚不健全,对于政府绩效第三方评估的管理制度也不够明确,因此在实践过程中,第三方在评估政府绩效时很容易受到各方的干预,缺乏独立性。这样的情况容易造成评估结果失真、评估结果的公平性和公正性无法保障,评估结果容易遭受质疑,从而丧失第三方评估的意义和作用。

(二)权威性问题。

政府绩效第三方评估的权威性主要依托于评估机构的“专业性”。我国目前的政府绩效第三方评估主要由部分高校或一些商业评估机构承担,这些评估机构的专业性达标与否还得由“官方”说了算。这就在一定程度上造成了第三方评估独立性和权威性的“诺斯悖论”:想要获得独立性就必须具有权威性,想要具有权威性就必须得到官方的认可,想要得到官方的认可又会在一定程度上丧失独立性。

(三)专业性问题。

我国的政府绩效第三方评估起步较晚,到现在为止也只不过积累了短短十几年的实践经验,与二战前就开始有实践经验的一些西方国家相比,在专业性程度上还有很大的差距。我国目前的政府绩效第三方评估机构的资格设置还缺乏一个相对完善的标准,因此各个机构的专业化水平良莠不一,专业人才不足,在评估指标设计、量化分析以及评估结果运用等方面都还存在各种问题。

(四)硬件设施与技术保障问题。

与商业的第三方评估机构相比,这个问题在高校参与的政府绩效评估过程中更为严重。现在已经是信息时代,在评估过程中运用智能终端设备能够有效提高评估效率、提高评估准确性。然而在许多实践中,硬件设施与相关技术得不到保障,评估过程中依然只能采用传统的评估方式,费时又费力。

(五)评估结果的反馈与应用问题。

政府绩效的第三方评估的目的主要是为了发现政府在工作过程中所出现的问题、为了更好地提升政府行政水平。但在实践中,依然存在一些不重视第三方评估结果的现象。一些政府机构对第三方评估的重要性认识不足,认为第三方评估只是“走过场”,抱着这种想法,自然也就缺乏对第三方评估结果的反馈和应用,这样非常不利于提升治理水平和行政能力,第三方评估的结果沦为“花瓶”。

(一)评估机构更加具有独立性、权威性和专业性。

随着法律制度的不断完善和第三方评估人才的不断培养,未来的政府绩效第三方评估将更加具有独立性和权威性。同时对于第三方评估机构资格的设置标准也在不断完善,有利于未来的第三方评估机构将朝着更加专业化的方向发展。

(二)商业性质的第三方评估机构增多。

从目前的实践来看,商业性的第三方评估机构与部分高校相比,具有更加效率、更强独立性、设备与技术保障更完善等优点。未来的第三方评估机构可能会朝着商业性的方向发展,或是以高校与商业性第三方评估机构合作的方式来开展针对政府绩效的第三方评估。

(三)人工智能(artificialintelligence)的引入与运用。

随着政府绩效第三方评估实践的不断发展,评估结果的运用成效也日益显现,政府对于第三方评估的结果越来越重视,第三方评估正在逐渐有力地发挥其应有的作用。在未来,依托于更加先进的信息技术,第三方评估的过程能够变得更加透明、公正公平和公开,第三方评估的结果也能够更加全面、具体。在有授权的情况下,第三方评估的结果可以通过互联网等平台公开,供广大科研工作者进行学术研究和使用,也能够让广大人民群众和媒体通过这些公开的评估結果数据对政府的工作进行监督,在很大程度上提高政府的公信力。

根据问题分析和未来发展趋势分析,要更好地发展我国政府绩效第三方评估,应从以下几个方面着力:

(一)紧扣国情、借鉴国外。

我国的政府绩效第三方评估实践起步较晚,因此可以借鉴许多国外已有的理论或经验,在紧扣我国国情的基础之上进行实践,不断进行理论丰富和创新,最终形成与我国实际情况更加符合的、具有我国特色的政府绩效第三方评估理论,能够从实践层面上更好地指导我国政府绩效第三方评估的发展。

(二)做好法制保障、制度建设与监督工作。

完善相关法律制度与管理制度,能够有效提升第三方评估机构的独立性、专业性和权威性。建立健全的对第三方法评估机构的监管制度、准入机制、行业标准,才能够有利于我国第三方评估机构健康、长久地发展。

(三)推进相关技术运用与开发。

科学技术是第一生产力。相关科学技术的成熟与运用是一个行业或产业快速发展的重要助推器。在已有的实践基础上,还应该大力推进与政府绩效第三方评估相关的技术开发与运用。例如智能辅助评估终端、评估数据和结果信息的网络化管理等等,都是非常有助于提升评估效率和科学性的技术。

综上所述,政府绩效第三方评估在我国的实践中依然还存在一些问题,还有很长的路要走,但总的发展趋势是好的,是在不断向前的。经过十几年的实践,我国的政府绩效第三方评估也取得了十分喜人的成效,越来越多的政府机构开始引入第三方评估的机制、开始重视第三方评估结果的作用、开始公开和反馈第三方评估的结果与数据。未来,我国政府绩效第三方评估一定会朝着更好的方向发展,为深化我国行政体制改革、提升政府行政水平、建设服务型政府提供助力。

【参考文献】。

[1]杜娟、杜义国、张微:“我国政府绩效第三方评估的研究现状及未来展望”,《领导科学》,2019年06期。

[2]徐双敏、陈尉:“第三方评估政府绩效的制度环境分析”,《学习与实践》,2013年09期。

[3]孟惠南:“第三方评估在我国政府绩效评估中的应用”,《领导科学》,2012年23期。

虽然我国已经开始认识到政府部门服务能力的重要性,并且采用了多种方式不断提高政府部门的服务水平。然而,在进行服务型转变的过程中,也需要注重以人为本的理念。目前,很多政府部门在应用以人为本理念的过程中,依然存在问题,主要体现在以下几点:

1、思想认识方面存在不足。

为了更好地在政府绩效管理过程中应用以人为本的理念,需要对相关理念有着深入认识,充分挖掘以人为本理念的内涵。但是,目前很多政府部门对以人为本理念的思想认识还存在不足:首先,很多政府部门依然没有认识到政府绩效评价体系的重要性,认为绩效管理体系无法取得应有的效果,而且还浪费人力物力,对提高政府部门的服务水平并无益处。其次,我国受传统思想文化的影响,官僚主义和封建主义的影响严重,导致很多政府部门并没有重视以人为本理念,在进行政府部门办公的过程中,依然采取高高在上的态度,导致民众投诉政府部门工作态度的事件频繁发生。最后,很多政府部门在进行绩效管理的过程中,过分地重视自身的利益,如果以人为本理念没有触及到政府工作人员的利益,那么工作人员并不重视以人为本理念的普及和应用,从而导致政府绩效管理在发展过程中遇到瓶颈。

2、政府绩效管理无法体现人文关怀。

目前,虽然很多政府部门为了积极响应国家的号召,在进行绩效管理过程中,将以人为本的理念应用其中,但是却仅仅停留在表面,没有更好地体现人文关怀。主要表现在:第一,很多政府部门为了更好地应用以人为本理念进行绩效管理,在对政府部门工作人员进行绩效考核时,往往利用不公平的手段强制要求民众对其工作水平进行认可,导致考核数据不准确,考核不公正的现象发生,无法更好地体现绩效考核的公平、公正、公开理念。第二,很多政府部门将绩效管理流于表面,虽然对工作人员进行绩效考核,但是考核的结果基本上都是内定的,因此无法更好地提高工作人员工作的积极性,同时也无法体现以人为本的理念。

3、加重政府或民众的负担。

在政府绩效考核的过程中加入以人为本的理念,如果应用不合理很可能会造成政府或者民众负担的情况出现:第一,目前,我国很多政府部门都是非营利性的组织,政府本身在推动绩效管理或者以人为本理念落实的过程中动力不足,如果强制使用以人为本理念,可能会加重政府工作的负担;第二,如果政府部门加重负担,那么很可能将这部分负担转嫁到普通民众的身上,导致民众使用政府工作的成本增高,也不利于普及以人为本理念应用到政府绩效的管理工作中。

综上,以人为本理念在政府绩效管理渗透过程中依然存在很多问题。因此,本文在分析了这些问题成因的基础上,进而提出相应的完善策略,主要体现在以下几个方面:

1、提高以人为本理念的重视程度。

以人为本理念能够更好地提高政府绩效管理效果,所以必须要提高政府部门对以人为本理念的重视程度:首先,需要对政府相关部门人员进行思想教育,使其充分认识到以人为本理念的重要性,学习以人为本理念的深刻含义,更好地将其应用到政府部门的工作中,从而提高政府部门的办事效率,更好地为民众提供相关服务;其次,政府绩效机制不仅仅关系到政府部门的未来发展,而且也对民众的生活有着重要的影响,因此必须要充分落实以人为本理念在政府绩效管理的重要程度,从而更好地树立政府部门“正直、廉洁、公正”的形象。

2、需要体现民众的根本利益。

以人为本理念应用到政府绩效管理中,不仅需要保持绩效考核的公正性,而且还需要体现民众的根本利益,可以参考以下几个方面:第一,政府部门是为人民群众服务的部门,因此必须要重视人民群众的利益,所以在为人民群众提供服务的同时,需要重视群众的意见,因此需要建立政府绩效的反馈机制,使人民群众参与到政府绩效的考核过程中,接受群众对政府服务的监督和反馈;第二,政府需要改变以往的官僚主义思想,充分认识到群众的位置,坚定政府部门的服务地位,切实维护好人民群众的根本利益。

3、促进人的全面发展。

以人为本理念应用到政府绩效管理的部门中,需要促进人的全面发展:第一,确保政府绩效管理的公平公正,从而能够对政府部门相关人员的工作内容进行科学合理地评价,对其长处进行表扬,对其不足予以指正,从而能够更好地促进政府部门人员工作能力和服务能力的提高;第二,政府绩效管理人员应该认识到,采用绩效管理的目标是为了更好地为群众服务,提高政府部门的服务水平,提高政府人员的服务水平,避免因为绩效考核损害到政府工作人员的工作积极性,切忌过度损害工作人员的相关利益。

4、鼓励群众参与政府绩效管理。

人民群众是政府的服务对象,因此有权利也有责任对政府部门的服务效果进行评价,所以为了更好地体现以人为本的理念,需要鼓励群众参与政府绩效管理:第一,当群众去政府部门中进行事务办理时,可以对服务人员的工作能力和工作态度进行评价,从而能够更好地从群众角度了解到政府部門的工作程度,真正了解群众关心的内容;第二,建立群众意见反馈渠道,当群众在办理事务时,如果遇到不懂的地方,或者是觉得不合理的地方,可以通过特定渠道进行反馈,政府部门需要积极归纳群众的意见,对合理的意见进行采纳,从而能够更好地提高政府部门的服务水平。

综上所述,我们不仅看到了目前政府绩效管理工作中存在的各类突出问题,更意识到了以人为本理念在政府绩效管理工作中渗透的重要意义。为了更好地打造服务型政府部门,我们需要对政府相关人员的工作进行评价和管理,并坚持以人为本的理念,使得评价的结果公平、公正、合理,更好地提高政府部门的工作成效。与此同时,作者也希望借助本课题的研究,在社会上引起人们对政府绩效管理工作创新改革的深刻关注,促使更多的社会专家、学者广泛的参与到本课题的研究中来,有效推动服务型政府的建设,促使政府绩效管理工作朝着更好的方向发展。

【参考文献】。

[2]李彦娅,何植民.基于科学发展观的地方政府绩效评估体系构建研究[j].人才资源开发,2007(06)9-10.

摘要:近年来,随着中国经济的持续发展和我国政府审计工作的改善,业绩审计在政府监督中发挥着越来越重要的作用。本文阐述了政府绩效审计的现状,深入分析了中国政府发展绩效审计的必要性,揭示了当前政府绩效审计存在的问题及其原因,最后从审计观念、审计监督制度、审计法律环境和预算制度等方面提出了优化政府绩效审计途径的建议,为中国政府绩效审计的高效合理运行和持续发展提供了一定的参考。

关键词:政府业绩审计审计监督优化路径公共管理。

近年来,随着我国经济持续发展和制度的不断完善,人民群众对政府公共财政支出的效率和效果关注度也日益提高,为了加强政府责任,适应我国建设社会主义法治国家的客观要求,产生了政府业绩评价。经过数十年的发展,政府审计在理论和实践层面进入了新阶段,现在以业绩审计为政府审计的重要内容,成为政府审计的新发展趋势。目前,我国政府绩效审计尚不成熟,仍处于起步阶段,存在诸多问题,如何构建适合我国国情的绩效审计体系成为我国政府审计的关键。

(一)是建设廉洁透明服务型政府的需要。

党的十九大报告明确了积极安全推进政治体制改革,建设高效透明的服务型政府。如今,我国正处在经济高速发展阶段,政府公共财政支出成为社会焦点问题。政府绩效考核的主要内容是对政府官员在任期间的工作情况和成果进行绩效考核和财政资金运用的有效性和经济性考核,政府绩效考核的开展拓宽了考核监督的领域和范围,加强了考核机构在行政监督中的作用,增强了各级政府工作人员的责任感,减少了腐败事件的发生,进一步避免了大量经济损失的发生,为建设高效透明的服务型政府营造了良好的氛围。

(2)是提高公共管理效率的需要。

近年来,政府财政收入大幅增加,公共管理投入增加,但公共建设相关项目从立项到最终完成需要多部门多层次审查,项目实施时失去最佳实施时机,同时浪费大量人力物力,这些现象的出现强调政府机构工作管理效率低,人浮于事件。审计机构在对政府部门及其下属单位进行绩效审计过程中,对公共资源配置的经济效率、配置效果等方面进行审查,有效管理和评价政府权力的使用,提高政府行政服务的工作效率水平。

(3)是保证政府职能发挥作用的需要。

近年来,中国财政支出呈现稳定增长态势。但由于目前中国政府财政支出结构不完善,财政支出表现相对较低。从近年来的开支来看,经济建设和行政管理所用的地方财政金额差异较大,人员资金开支已成为主要部分。该支出严重限制了政府职能的正常履行,因此需要加强对政府财政资金的监管和分配。根据政府财政资源有限的基本国情,及时对财政监督财政开支,能够有效控制各级政府部门公共资金开支,提升公共资金业绩。

(1)审查范围逐渐扩大。

21世纪初,政府有关机构及其下属单位的业绩审计提上日程。审计机构审计重点从以前被审计机构财务收支的客观性和合法性转变为被审计机构财务资源使用的有效性和经济性。与此同时,2016年1-11月,国家审计机构完成了21334业绩审计项目。其中,审计局完成113个,省级审计机构完成847个,地级审计机构完成6288个,县级审计机构完成14086个。另外,各省市地方审计机构实施的绩效审计项目数量占审计项目总数的31%。其中,18个省(自治区、直辖市)和5个独立计划城市均超过这一平均水平。2017年1-11月,绩效考核工作量约占考核总工作量的一半左右,比2016年同期增长10%。

(二)加强政策支持力度。

近年来,审计局发布了多份文件,规划了业绩审计的发展方向,十三五国家审计工作的发展规划提出进一步加大业绩审计力度,始终贯穿审计工作,促进发展质量和利益的重大违纪违法问题的暴露和审查力度,促进廉政建设。与此同时,各省市也积极制定政府业绩审计相关政策,2011年徐州市审计部门发布《徐州市审计机关政府投资项目业绩审计操作指南》,规范徐州市业绩审计工作,2012年浦东新区人民政府发布《关于印发浦东新区业绩审计方法的通知》,对浦东新区政府部门业绩审计提出了实施要求。

(一)审计机构缺乏独立性。

从我国政府行政结构来看,我国各地市审计机构属于各地政府,受当地政府和上级审计机构的指导,受经费和行政管理模式的影响,业绩审计目标和具体任务的制定受当地政府的制约,阻碍审计作用确实有效地发挥,同时审计机构业绩审计活动完成后,审计机构发行的审计报告由同级地方政府和上级审计机构审计,不利于审计工作报告的完整性和客观性宝座,审计报告的真实性受到制约。另外,审计机构的组成人员审计机构本身只有提案的权利,其人员编制、人事选拔、任免、奖惩均由同级组织部和同级政府审查批准,在一定程度上阻碍了审计机构职能的正常发挥。

*河北省哲学社会科学计划办公室:ppp模式支持正确扶贫绩效评价研究(hb18gl058)。

(二)缺乏相关法律法规的指导。

我国政府业绩审计起步晚,审计工作的具体方法和程序尚不成熟,目前只有少量法律法规提到政府业绩审计,政府业绩审计工作开展的完整法律依据不足,现行《审计法实施条例》第二条指出,《审计机关依法独立检查被审计机关的会计证明书、会计账簿、财务会计报告书及其他与财政收支、财务收支相关的资料和资产,监督财政收支、财务收支的真实性、合法性和效益行为》,相关法律只进一步明确了该法律规定,但没有列出具体的审计内容和我国政府的具体规定。在这种背景下,审计相关人员只能通过现有的知识储存和现有的经验来探索,审计人员的工作水平也不同,业绩审计效果的正确客观性增加了一定的风险。(3)审计团队人才结构不合理。

完成政府业绩审计需要多个专业审计师团队。在此期间,中国审计机构主要从事传统财务审计。审计师的知识结构比较简单,工程技术、法律、宏观经济、对外贸易和计算机专家数量比较少。根据中国审计师知识结构的调查,会计和审计专家占所有审计师的71.8%。绩效考核要求考核广泛的经济和管理活动的有效性,要求多样化、创新的方法和多学科知识。有些审计师很难满足高标准的绩效审计。另外,审计员整体的数量也很少,难以适应大的任务量,水平越低的机构越明显。

(一)完善独立监督体制。

一是要进一步加强垂直管理改革力度。不仅将地方审计机关正职的任免权接受省级机关,普通审计人员也由省审计厅管理,招聘、提拔、任免由地方审计机关提名,由省审计厅和省人社会局决定,二是地方审计机关经费由省财政厅统一保证,纳入省财政预算。由此可以将人力资源和财务完全独立于地方政府和财政部门,提高独立性,使地方审计部门更有效地对地方政府领导进行审计监督,更有力地揭示和反映问题。

(二)健全相关法律制度。

由于缺乏明确的法律条款,我国政府业绩审计推广和拓展面临一定的困难。我国立法机关应进一步完善我国政府业绩审计相关法律制度。首先要完善审计报告的公开制度,通过将审计活动的各个阶段以审计报告的形式向公众展示。在完善审计报告制度的过程中,可以充分利用媒体作为中介的作用,及时准确地将审计结果传递给社会公众,扩大政府审计的影响力。其次,要不断健全审计结果问责制度,加强审计结果的应用,实现审计报告的公开与审计结果问责的有效联系。

(3)构建合理的评价标准体系。

随着政府职能的逐步完善,经济性和利益性也是政府业绩审计应该关注的重要内容。在预算绩效考核体系的基础上,审计机构应积极探索绩效考核考核标准,建立全国公认、适用、规范的体系,不断修改、完善。业绩评价体系应当包括预期的目标、决策过程、投入资源和能源、生产效益、对社会效益的影响等,评价指标应当坚持定量和定性的结合,定量指标主要指财务指标,定性指标的确定应以被审计项目的特点和被审计部门的管理特点为基础。

(四)改善政府业绩审计人员结构。

审计团队人员的综合素质和业务能力是政府业绩审计顺利进行的重要保障。审计机构不仅要提高现有审计人员的综合素质能力,还要不断引进高水平的审计人才,尤其是专业型的审计人员。要提高审计机构的准入门槛,完善审计人员的准入制度,确保招聘人员在综合素质和业务水平上能够满足政府业绩审计的要求,确保审计队伍中的人才能够胜任其职责。此外,审计机构人员不仅要包括专业审计人员,还要在实际绩效审计工作中招聘工科类、经济管理类等其他专业人员,不断优化审计人员结构。

全面优化我国政府业绩审计可有效促进我国审计持续健康发展,有利于推进社会主义法治社会建设,但结合政府业绩审计在我国起步晚、发展缓慢的现状,如何更好地开发和引导还需要进一步深入研究和探讨。

参考文献。

[1]齐兴利,绍辉.中国政府业绩审计发展途径研究[j].审计和经济研究,2007.2。

[2]周亚荣.中国政府业绩审计理论研究与实践现状[j].审计与经济研究,2008.2。

[3]宋夏云,我国政府业绩审计师能力框架研究[j].会计研究,2013.4。

[4]曲明,中国政府绩效审计评价标准体系框架构建[j].财经问题研究,2016.5。

[5]闵晓蕾.政府业绩审计理论文献综述[j].财经论坛,2006.2。

摘要:政府信任反映了居民对政府的认可程度,是政府政策执行的基础。汶川的地震灾区居民政府信任水平没有随着灾区公共服务水平的提高得到相应的提升,通过运用“汶川地震十周年居民生活情况调查”数据,实证分析探究了人际信任、政府绩效与政府信任之间的关系,研究结果显示,政府绩效、人际信任均对政府信任产生显著的影响。

关键词:灾区居民;政府信任;人际信任;政府绩效。

2008年汶川8.0级特大地震给震区带来了巨大的财产和生命安全损失,我国政府在救灾所表现出的强大动员能力、应急能力也成为世界关注的焦点。尤其通过政府的各项震后恢复政策,使得汶川灾区的社会经济发展获得历史性机遇,基础公共社会高水平重构,产业发展能力也持续提升,百姓生活水平相对于震前有较大幅度提高。

较高的政府信任度会有效降低政府运行的成本,于政府及公民都是有利的。而政府信任会受一系列因素的影响,在突发事件发生的关键时期政府政策执行对政府信任的影响更为显著。政府信任作为公民对政府的主观评价,受诸多因素的影响。现有的研究中不乏对政府影响因素的探讨,程倩认为,政府信任是政府合法性的本质内涵,政府信任关系是公民对政府及其政策执行过程中各相关要素、政策执行过程以及政策执行结果的心理预期。在二者的关系解释上,政府高校的运作过程以及较好的绩效水平可以带来良好的政府信任水平,两者呈正相关关系(程倩,2011)。缪婷婷认为,影响政府信任的因素具体包括政府作为、政府绩效以及新闻媒体的兴起(缪婷婷,2016)。在关键时期,政府政策对居民的政府信任影响更为显著。在突发事件中,公民对政府的信任感往往受领导者的价值观以及政府政策执行情况的影响,政府信任与政府及其工作人员对受灾民众的真诚关怀和激励呈显著正相关的影响(王璐,2008)。

因此,本文在梳理了现有研究成果的基础上,基于“汶川地震灾区灾后重建十周年状况调查”数据,对影响灾区居民政府信任度的影响因素进行实证分析。

2.1人际信任。

在文化主义视角下,人际信任会对政府信任产生显著的影响。高巍通过实证研究证实了人际信任对政府信任的影响:民众越容易相信他人,则更容易信任政府(高巍,2015)。胡荣等学者研究发现个人的信任水平对政府信任度会产生显著影响(胡荣,2007)。陈天祥构建了包括村内信任和社会信任的影响因素模型,验证了民众个人的信任水平对政府信任会产生显著的影响(陈天祥,2017)。人际信任程度越高,越有利于民众与政府的良性互动(刘人宁,2018)。人际信任变量作为文化路径中重要的解释变量,对居民政府信任的影响不言而喻,因此本文将其作为自变量的指标之一。据此,提出假设1:

假设1:人际信任度越高,政府信任度越高;人际信任度越低,政府信任度越低。

2.2政府绩效。

制度主义路径认为政府绩效很大程度上影响人们对政府信任度的判定,政府绩效涵盖政治、经济、环境保护等多方面的绩效。许伟通过研究发现,政府绩效变量对政府信任会产生显著的影响,同时验证了政府绩效与政府信任之间的正相关关系。杨培鸿(2007)通过实证研究证明了公共服务绩效作为关键性的影响因素,对公民政府信任评价起着重要的作用。陈天祥在研究中国乡镇政府的政府信任情况时也将政府绩效作为自变量进行分析(陈天祥等,2017)。由此,提出假设2:

假设2:政府绩效水平与政府信任呈正相关关系。

3.1数据来源。

本研究使用数据来源于国家科技部中国发展战略研究院联同西南交通大学开展的“汶川十周年居民生活情况调查”数据,调查范围包括四川省5市/州30县/区共198个行政村/社区,共抽取了30個受灾县中的4950户住户,共完成家户问卷3826份,完成个人问卷3751份。

3.2变量的操作化。

因变量:政府信任变量。

在“汶川地震十周年居民生活状况调查”问卷中,关于政府信任被划分为对中央政府、省政府、县/市政府、乡/镇/街道四个层级政府的信任,选项分别为“完全信任”、“比较信任”、“不太信任”、“根本不信任”,对四个信任度赋值为1-4,1代表“完全信任”,4代表“根本不信任”,以此类推。通过探索性因子分析提取出“政府信任”因子。

自变量一:人际信任。

“汶川地震十周年居民生活状况调查”问卷中,对人际信任变量操作为题目——对“家人”、“住在你周围的人”、“市场上的商人/买卖人”、“外地人”、“村干部”、“警察”、“医生”、“法官”共8种不同类型群体信任情况,答案与测量“政府信任”的答案一致。通过探索性因子分析,根据指标特征,提取出三个因子,分别为“权威信任”、“社会信任”、“邻里信任”。

自变量二:政府绩效。

本研究中关于政府绩效感知被测量为居民对“贫困问题”、“贫富差距”、“失业情况”、“空气污染”、“水污染”、“缺水问题”、“孩子上学不方便”的严重程度评价,答案包括“特别严重”、“比较严重”、“不太严重”、“不成问题”四个程度的测量,从“特别严重”至“不成问题”分别赋值为1-4。通过因子分析,提取3个公因子为“人民生活”、“环境保护”、“公共服务”。

4.1相关性分析。

通过相关性分析可知,居民人际信任的三个维度与政府信任的相关显著性均为0.000,小于0.05,均呈显著性相关关系,表明民众个人信任水平会显著影响民众对政府的信任,其中权威人群信任维度的得分系数最高,为0.665。

该部分基于因子分析和相关性分析的基础上,采取线性回归方法进一步验证各影响因素与政府信任的关系,建立了2个回归模型,对影响灾区居民政府信任的影响因素进行实证分析。

模型1中检验的是人际信任与政府信任之间的关系。在研究人际信任与政府信任的回归中国,采用逐步回归法,将人际信任的3个公因子作为自变量,政府信任因子作为因变量进行分析。根据选取自变量的规则p0.05,共选择了3个变量,包括权威信任、邻里信任以及社会信任。模型1的相关系数为0.678,调整后的r方为0.459。因此,最终建立的模型1包含权威信任,邻里信任,社会信任3个影响因素。通过anova表考察模型是否有效,该模型回归的显著性水平均显示p0.05,表明回归程度显著,可以认为该模型有效。从回归系数表可知,权威信任、邻里信任以及社会信任的回归系数分别为0.616、0.161、0.085,均大于0,说明权威信任、邻里信任以及社会信任程度越高,居民对政府信任的程度越高。t检验中,权威信任、邻里信任、社会信任和常数项的显著性p值都小于0.05,因而均有显著意义。

模型2检验政府绩效与政府信任之间的关系。在研究政府绩效与政府信任的回归中,采用逐步回归法,将绩效感知的3个因子作为自变量,政府信任因子作为因变量进行分析。根据选取自变量的规则p0.05,共选择了3个变量,包括制环境保护、公共服务以及人民生活三个维度的变量,建立模型2。根据模型2可知,环境保护、公共服务以及人民生活的回归系数分别为-0.179、-0.118、-0.109,絕对值均大于0,回归系数为负,说明环境保护、公共服务以人民生活各方面问题的严重程度越高,居民对政府信任的程度越低,即政府绩效水平与政府信任呈正相关关系,政府绩效越好,居民政府信任度越高。t检验中,环境保护、公共服务、人民生活和常数项的显著性p值都小于0.05,因而均有显著意义。

以“政府绩效与公众信任”为撰写小论文

公众是指公共关系的主体社会成员的共同利益、共同利益和相互作用。以下是为大家整理的关于,欢迎品鉴!

作为以管理主义和市场取向为基础的管理工具,绩效管理在公共管理领域发挥了重要作用.政府采购绩效管理是政府绩效管理的重要组成部分,也是政府采购制度建设的重要内容,可以确保政府财政预算资金的高效使用,同时也可以提升政府采购管理水平.现阶段被纳入政府采购管辖范围的事项几乎涵盖社会生活的各个领域,不仅涉及货物和服务采购,还包括工程采购,几乎涉及经济社会的各个领域,政府采购规模和结构的变化则会对一国经济发展和产业结构产生巨大的影响.因此,分析政府采购绩效管理的价值性,并对政府采购进行价值性评估成为政府采购绩效管理的关注点.

长期以来公共价值在政府行为研究中被忽视,这与公共价值在现实中的重要作用相悖.涉及公众利益的公共价值在政府采购中体现得尤为明显,市场主导型的社会强调根据市场来配置各种社会资源,凸显价值资源的独立性和特殊性,政府采购通过制度运行的绩效使其具有合法性,而制度绩效往往被简单地用经济成就来代替,这样的偏颇之处将会把公共价值和市场经济对立起来,不断提升的绩效水平并不一定带来公民对政府采购满意度的提高以及对政府采购信任水平的增加.与此同时,政府采购价格高于市场价格、采购周期长于自主采购、采购结果的利用以及财政资金的产出效益不透明也广受争议.

公共价值作为一种全新的公共管理理念,体现的是政府的行为逻辑,强调的是公共服务和公共产品的公共效用、政府作为管理主体追求公共利益的价值诉求以及政府在行动过程中对公众行为主张的公益导向.公共价值塑造着公民的公共生活,促进公共福祉和个人福祉.政府采购所提供的产品和服务本身蕴含着满足公共需要的价值属性,政府正是通过实施政府采购满足公众的公共需求,并在这一过程中追求效率、公平、公正、******、责任等公共价值.通过公共价值的输入,确保政府采购绩效管理的正确方向,将基于公共价值的绩效管理和政府采购绩效管理的工具进行理性结合,避免政府采购绩效管理陷入自身工具理性强大、价值理性虚弱的困境.通过政府采购绩效管理中出现的一些问题,剖析公共价值理念在其实践过程中的一些障碍,可以更加深入地理解以工具理性和效率至上作为价值追求的传统绩效管理无法解决的重要问题.

基于公共价值的政府采购绩效管理在主体选择、制度设计、公众参与及结果使用上都有明显不同于传统绩效治理的特征,政府采购在提供公共产品满足公共需求的过程中关注并引导公众的集体偏好,确定并创造公共价值.在政府采购过程中,不仅要利用有限的财政资金在规定的周期内完成项目采购,而且需要关注和回应公民、具体项目业主及提供资金来源的纳税人偏好.公共价值在很大程度上决定着政府采购绩效是否真正得到全面体现,政府采购公共价值的建构与实现构成提升政府采购绩效管理水平的路径依赖,直接影响着政府采购绩效管理的发展方向.

当前,我国正处在一个重要的历史节点上,改革进入攻坚期和深水区,全面建成小康社会进入决定性阶段.经济全球化、信息全球化、国内外贸易环境和规则对政府采购绩效管理理念、体制、程序等方面带来巨大的挑战.在这样的社会发展变化环境中,市场对资源配置起决定性作用,政府的主体地位遭到削弱,而个体的合法逐利,能够实现自我价值,同时也增加了与公共价值冲突的矛盾.政府采购绩效管理既要体现公共价值,矫正传统绩效管理效率占优的价值取向,重新回归政府行为“公共性”的本质和中心,又要体现本身的经济性,因此,政府采购绩效管理创新,建立以公共价值为理论基础的政府采购绩效管理模式,体现了理论和实践的双重需要.

本文基于对政府采购绩效管理问题相关文献的梳理与研究,并借鉴行政学、管理学、财政学的理论基础,遵循马克思主义方法论,按照理论基础——现实状况——原因分析——经验借鉴——创新性思考的逻辑思路,综合运用马克思辩证唯物主义、比较分析法、逻辑分析法、价值链分析法、协同管理理论,以现阶段政府采购绩效管理为切入点,从对政府采购的公共价值分析入手,对当前政府采购绩效管理现状和成因进行了深入分析,在分析公共价值对政府采购绩效管理重要作用的基础之上,论证政府采购绩效管理的价值内生于公共价值当中,政府采购对资源的配置、对物品的分配、对利益的调整等,均应以公共价值为宗旨和归宿.基于公共价值的政府采购绩效管理的终极指向是为公众创造更多的公共价值,而非仅仅重视政府采购任务的完成和单纯的方法、技术的改进.

本文对基于公共价值的我国政府采购绩效管理的基本问题、具体架构、现实形态及总体思路等方面进行了详细分析,设计了以公共价值为基础的政府采购绩效治理的理论架构:从改革预算制度,为政府采购提供合理的运行平台;完善组织模式,为政府采购提供公正的竞争环境;优化流程设计,为政府采购提供明确的架构设定;强化监督机制,为政府采购提供有效的权力制约;建立协同机制,为政府采购提供顺畅的组织协同;推进信息化建设,为政府采购提供充裕的技术储备;设定评估指标,为绩效评估提供规范的操作指引.从以上七个方面提出以公共价值为基础的政府采购绩效管理改革思路.

摘要:及时回应人民群众的期盼和关切是现代政府的一个重要标志。回应人民群众期盼和关切,宏观上要保证经济社会稳定发展,微观上要在具体事件中保障人民切身利益。提升政府回应能力是建设服务型政府的内在要求,是建立和谐社会的有力保障,是增强政府公信力的重要途径,是政府执政能力不可或缺的重要方面,是衡量政府执政水平的重要标杆。

关键词:政府回应内涵价值。

全球治理委员会在《我们的全球之家》的报告中指出,治理有六个基本要素:参与、公开、透明、回应、法治和责任。回应作为治理的要素之一,要求政府在治理过程中,要与老百姓互动,对公众的合法性要求作出及时、负责的反应,不能无故拖延或者不了了之。回应越快,治理的程度越高。

李克强总理说过,及时回应人民群众的期盼和关切是现代政府的一个重要标志。回应人民群众期盼和关切,宏观上要保证经济社会稳定发展,微观上要在具体事件中保障人民切身利益。

1.政府回应的内涵。

政府回应,“就是政府在公共管理过程中,对公众的需求和提出的问题作出积极敏感的反应和回复的过程”[1]。

政府回应的两点要求:其一,政府部门对公众的需求和提出的问题具有敏感性,要及时作出反应,不能置之不理;其二,政府在敏感性的基础上,要作出积极有效的回复,满足公众需求或解决群众问题。

政府回应是个互动过程:对公众来说,可以通过利益诉求渠道表达自身的要求与期望;对政府来说,要及时有效地对公众的诉求给予反应和回复。政府回应的结果可能是一种积极的表态,表明政府对问题的重视;也可能是对满足公众需求,还可能是对公众要求的一种说服和引导[2]。

2.政府回应的要素。

一是回应主体。也就是“谁回应”,主要是政府和政府部门及其工作人员。二是回应对象,也就是“回应谁”,既可以是公民个体,也可以是社会组织,社会大众。三是回应客体,也就是“回应什么”,指公众提出的需要政府满足或解答的诉求。这种诉求可以是明确表达出来的,也可以是潜在的;既可以是反映社会整体利益的诉求,也可以是个人的具体诉求。四是回应渠道,也就是“怎么回应”,即回应的方式和渠道。五是回应反馈,也就是“回应的如何”,即公众对政府的回应作出的反应性信息,并把这种信息反馈给政府。

1.提升政府回应能力是建设服务型政府的内在要求。

传统管制型社会中,政府也要根据社会需要去做事。但由于社会事务相对来说比较简单,而且公众很少有机会向政府提出要求,因此政府不会承受太多回应性的压力。政府之于公众来说,更多的是管控者,缺少对公众的服务和诉求的回应。

服务型政府的建设意味着政府角色由管控者向服务者的转变。服务型政府,以服务人民、服务社会为宗旨。为了更好地提供服务,政府需要准确掌握公众的需求,并及时满足公众需要,解決公众问题,这个过程就是政府积极有效的回应过程。因此,提升政府回应能力是建设服务型政府的内在要求。

2.提升政府回应能力是建立和谐社会的有力保障。

说文解字看“和谐”二字。“和”由“禾”和“口”构成,“禾”代表粮食,“口”表示人。据此,“和”就是指人人有饭吃。“谐”由“言”和“皆”构成,“言”指说话、表达,“皆”是全部的意思。据此,“谐”就是指人人都有表达的、说话的权力,能够畅所欲言。那么“和”“谐”二字连起来,为我们描绘的和谐社会的场景就是:人民在满足物质生活的同时,表达权、参与权、监督权等也都能够得到保障。要实现这样一副场景,就要增强政府的回应性[3]。

而且,从社会稳定的角度说,政府能够及时了解民情、掌握民意,对涉及群众利益的问题能够早发现、早处理、早解决,给予及时回应,才能消除社会中存在的不和谐因素,实现社会的稳定发展。

3.提升政府回应能力是增强政府公信力的重要途径。

2014年3月18日,习近平总书记在河南省兰考县县委常委扩大会议上的讲话中,提到过“塔西佗陷阱”。通俗地讲,“塔西佗陷阱”就是当执掌公权力的政府部门失去公信力时,无论其说真话还是假话,都会被认为说假话;无论其做好事还是坏事,都会被认为是做坏事。习近平总书记在讲话中指出,“我们当然没有走到这一步,但存在的问题也不可谓不严重,必须下大气力加以解决。如果真的到了那一天,就会危及党的执政基础和执政地位。”[4]公信不立,患莫大焉。人民对政府的信任,是政府权威的基础。

提升政府回应能力是增强政府公信力的重要途径。政府回应是政府公信力最直接的来源。如果把政府公信力看作一个“产品”,在它的生产过程中,人民的诉求是“需求侧”,政府回应就是“供给侧”。群众通过各种渠道发出呼声,向政府提出要求,实际上是群众有对政府产生信任的需求,希望政府是一个关心人民疾苦、能够为人民排忧解难的机构,而政府则通过回应向人民群众提供可以信任的政策资源,并使人民知晓政府是关心并愿意保护他们切身利益的可以信赖的管理者。在呼应之间,“需求”和“供给”发生了相互作用并对接起来,政府与人民之间的信任由此建立,并成为一种稳定的社会资本,政府公信力亦随之产生[5]。

在需求与供给间,就像社会经济领域要从拉动需求转向供给侧改革一样,政府不仅要及时回应人民提出的需求,更应该积极主动地了解社会发展和人民权益状况,在需求产生之前主动响应,提高政策水平、提升执政能力、强化服务意识,时刻将改善民生作为政府职能第一要务,以高质量的公共产品和公共服务的“供给”满足人民的需求,保障人民权益,保持政府公信力不断产生和稳定增长[5]。

摘要:近年来,随着中国经济的持续发展和我国政府审计工作的改善,业绩审计在政府监督中发挥着越来越重要的作用。本文阐述了政府绩效审计的现状,深入分析了中国政府发展绩效审计的必要性,揭示了当前政府绩效审计存在的问题及其原因,最后从审计观念、审计监督制度、审计法律环境和预算制度等方面提出了优化政府绩效审计途径的建议,为中国政府绩效审计的高效合理运行和持续发展提供了一定的参考。

关键词:政府业绩审计审计监督优化路径公共管理。

近年来,随着我国经济持续发展和制度的不断完善,人民群众对政府公共财政支出的效率和效果关注度也日益提高,为了加强政府责任,适应我国建设社会主义法治国家的客观要求,产生了政府业绩评价。经过数十年的发展,政府审计在理论和实践层面进入了新阶段,现在以业绩审计为政府审计的重要内容,成为政府审计的新发展趋势。目前,我国政府绩效审计尚不成熟,仍处于起步阶段,存在诸多问题,如何构建适合我国国情的绩效审计体系成为我国政府审计的关键。

(一)是建设廉洁透明服务型政府的需要。

党的十九大报告明确了积极安全推进政治体制改革,建设高效透明的服务型政府。如今,我国正处在经济高速发展阶段,政府公共财政支出成为社会焦点问题。政府绩效考核的主要内容是对政府官员在任期间的工作情况和成果进行绩效考核和财政资金运用的有效性和经济性考核,政府绩效考核的开展拓宽了考核监督的领域和范围,加强了考核机构在行政监督中的作用,增强了各级政府工作人员的责任感,减少了腐败事件的发生,进一步避免了大量经济损失的发生,为建设高效透明的服务型政府营造了良好的氛围。

(2)是提高公共管理效率的需要。

近年来,政府财政收入大幅增加,公共管理投入增加,但公共建设相关项目从立项到最终完成需要多部门多层次审查,项目实施时失去最佳实施时机,同时浪费大量人力物力,这些现象的出现强调政府机构工作管理效率低,人浮于事件。审计机构在对政府部门及其下属单位进行绩效审计过程中,对公共资源配置的经济效率、配置效果等方面进行审查,有效管理和评价政府权力的使用,提高政府行政服务的工作效率水平。

(3)是保证政府职能发挥作用的需要。

近年来,中国财政支出呈现稳定增长态势。但由于目前中国政府财政支出结构不完善,财政支出表现相对较低。从近年来的开支来看,经济建设和行政管理所用的地方财政金额差异较大,人员资金开支已成为主要部分。该支出严重限制了政府职能的正常履行,因此需要加强对政府财政资金的监管和分配。根据政府财政资源有限的基本国情,及时对财政监督财政开支,能够有效控制各级政府部门公共资金开支,提升公共资金业绩。

(1)审查范围逐渐扩大。

21世纪初,政府有关机构及其下属单位的业绩审计提上日程。审计机构审计重点从以前被审计机构财务收支的客观性和合法性转变为被审计机构财务资源使用的有效性和经济性。与此同时,2016年1-11月,国家审计机构完成了21334业绩审计项目。其中,审计局完成113个,省级审计机构完成847个,地级审计机构完成6288个,县级审计机构完成14086个。另外,各省市地方审计机构实施的绩效审计项目数量占审计项目总数的31%。其中,18个省(自治区、直辖市)和5个独立计划城市均超过这一平均水平。2017年1-11月,绩效考核工作量约占考核总工作量的一半左右,比2016年同期增长10%。

(二)加强政策支持力度。

近年来,审计局发布了多份文件,规划了业绩审计的发展方向,十三五国家审计工作的发展规划提出进一步加大业绩审计力度,始终贯穿审计工作,促进发展质量和利益的重大违纪违法问题的暴露和审查力度,促进廉政建设。与此同时,各省市也积极制定政府业绩审计相关政策,2011年徐州市审计部门发布《徐州市审计机关政府投资项目业绩审计操作指南》,规范徐州市业绩审计工作,2012年浦东新区人民政府发布《关于印发浦东新区业绩审计方法的通知》,对浦东新区政府部门业绩审计提出了实施要求。

(一)审计机构缺乏独立性。

从我国政府行政结构来看,我国各地市审计机构属于各地政府,受当地政府和上级审计机构的指导,受经费和行政管理模式的影响,业绩审计目标和具体任务的制定受当地政府的制约,阻碍审计作用确实有效地发挥,同时审计机构业绩审计活动完成后,审计机构发行的审计报告由同级地方政府和上级审计机构审计,不利于审计工作报告的完整性和客观性宝座,审计报告的真实性受到制约。另外,审计机构的组成人员审计机构本身只有提案的权利,其人员编制、人事选拔、任免、奖惩均由同级组织部和同级政府审查批准,在一定程度上阻碍了审计机构职能的正常发挥。

*河北省哲学社会科学计划办公室:ppp模式支持正确扶贫绩效评价研究(hb18gl058)。

(二)缺乏相关法律法规的指导。

我国政府业绩审计起步晚,审计工作的具体方法和程序尚不成熟,目前只有少量法律法规提到政府业绩审计,政府业绩审计工作开展的完整法律依据不足,现行《审计法实施条例》第二条指出,《审计机关依法独立检查被审计机关的会计证明书、会计账簿、财务会计报告书及其他与财政收支、财务收支相关的资料和资产,监督财政收支、财务收支的真实性、合法性和效益行为》,相关法律只进一步明确了该法律规定,但没有列出具体的审计内容和我国政府的具体规定。在这种背景下,审计相关人员只能通过现有的知识储存和现有的经验来探索,审计人员的工作水平也不同,业绩审计效果的正确客观性增加了一定的风险。(3)审计团队人才结构不合理。

完成政府业绩审计需要多个专业审计师团队。在此期间,中国审计机构主要从事传统财务审计。审计师的知识结构比较简单,工程技术、法律、宏观经济、对外贸易和计算机专家数量比较少。根据中国审计师知识结构的调查,会计和审计专家占所有审计师的71.8%。绩效考核要求考核广泛的经济和管理活动的有效性,要求多样化、创新的方法和多学科知识。有些审计师很难满足高标准的绩效审计。另外,审计员整体的数量也很少,难以适应大的任务量,水平越低的机构越明显。

(一)完善独立监督体制。

一是要进一步加强垂直管理改革力度。不仅将地方审计机关正职的任免权接受省级机关,普通审计人员也由省审计厅管理,招聘、提拔、任免由地方审计机关提名,由省审计厅和省人社会局决定,二是地方审计机关经费由省财政厅统一保证,纳入省财政预算。由此可以将人力资源和财务完全独立于地方政府和财政部门,提高独立性,使地方审计部门更有效地对地方政府领导进行审计监督,更有力地揭示和反映问题。

(二)健全相关法律制度。

由于缺乏明确的法律条款,我国政府业绩审计推广和拓展面临一定的困难。我国立法机关应进一步完善我国政府业绩审计相关法律制度。首先要完善审计报告的公开制度,通过将审计活动的各个阶段以审计报告的形式向公众展示。在完善审计报告制度的过程中,可以充分利用媒体作为中介的作用,及时准确地将审计结果传递给社会公众,扩大政府审计的影响力。其次,要不断健全审计结果问责制度,加强审计结果的应用,实现审计报告的公开与审计结果问责的有效联系。

(3)构建合理的评价标准体系。

随着政府职能的逐步完善,经济性和利益性也是政府业绩审计应该关注的重要内容。在预算绩效考核体系的基础上,审计机构应积极探索绩效考核考核标准,建立全国公认、适用、规范的体系,不断修改、完善。业绩评价体系应当包括预期的目标、决策过程、投入资源和能源、生产效益、对社会效益的影响等,评价指标应当坚持定量和定性的结合,定量指标主要指财务指标,定性指标的确定应以被审计项目的特点和被审计部门的管理特点为基础。

(四)改善政府业绩审计人员结构。

审计团队人员的综合素质和业务能力是政府业绩审计顺利进行的重要保障。审计机构不仅要提高现有审计人员的综合素质能力,还要不断引进高水平的审计人才,尤其是专业型的审计人员。要提高审计机构的准入门槛,完善审计人员的准入制度,确保招聘人员在综合素质和业务水平上能够满足政府业绩审计的要求,确保审计队伍中的人才能够胜任其职责。此外,审计机构人员不仅要包括专业审计人员,还要在实际绩效审计工作中招聘工科类、经济管理类等其他专业人员,不断优化审计人员结构。

全面优化我国政府业绩审计可有效促进我国审计持续健康发展,有利于推进社会主义法治社会建设,但结合政府业绩审计在我国起步晚、发展缓慢的现状,如何更好地开发和引导还需要进一步深入研究和探讨。

参考文献。

[1]齐兴利,绍辉.中国政府业绩审计发展途径研究[j].审计和经济研究,2007.2。

[2]周亚荣.中国政府业绩审计理论研究与实践现状[j].审计与经济研究,2008.2。

[3]宋夏云,我国政府业绩审计师能力框架研究[j].会计研究,2013.4。

[4]曲明,中国政府绩效审计评价标准体系框架构建[j].财经问题研究,2016.5。

[5]闵晓蕾.政府业绩审计理论文献综述[j].财经论坛,2006.2。

摘要:以山东省950名青少年学生为调查对象,对青少年的政府信任影响因素进行比较分析与研究,结果发现,理性选择路径、社会文化路径、政治参与路径均对青少年的政府信任有显著影响。在此基础上,提出了改善青少年政府信任的有效策略。

关键词:青少年政府信任影响因素心理机制。

中图分类号:g641文献标识码:a。

1问题的提出。

政府信任,指民众对于政府或政治制度所抱有的信心,相信它们会制定符合民众利益的政策并提供其预期的政策结果[1]。青少年是未来社会主义建设的接班人和生力军,他们的政府信任直接决定了我国政治生态和民主建设的发展与未来。学界关于青少年政府信任现状的研究,得出的结果不尽相同。有些研究表明,当代中国青少年政治信任的总体状况良好[2-3],另外一些研究显示,当代青少年政治认同度不高且呈工见非均衡状态[4]。

加快新旧动能转换是山东省促进经济结构转型升级的重要途径。在新旧动能转换中,各级政府肩负着提供公共产品,支持科技创新,引导产业发展,健全社会保障等重要作用。在这样的背景下,研究山东省青少年的政府信任具有重要的现实意义。

目前关于青少年政府信任的影响因素,主体方面的影响因素主要有青少年的性别、年龄、家庭情况、人际信任、思政政治教育、大众传媒等;客体方面主要有政府的服务水平、公务员的素质、行政执行力等因素[5]。对政治信任的研究主要存在三种竞争性理论解释路径,即理性选择路径、社会文化路径、政治参与路径[6]。青少年政府信任影响因素及心理机制的分析模型,见图1。

什么因素影响山东省青少年的政府信任?三种解释理论预测作用如何?如何提升青少年政府信任?对此,本文进行有益的探索。

2研究方法。

2.1研究对象。

本研究采用随机抽样的方法,对山东省4所中学和4所大学的950名青少年进行调查研究。其中,男生386人(40.6%),女生564人(59.4%);高一学生147人(15.5%),高二学生140人(14.7%),高三学生118人(12.4%),大一学生166人(17.5%),大二学生146人(15.4%),大三学生120人(12.6%),大四学生112人(11.8%);生源地为农村的571人(60.1%),城镇的为379人(39.9%);政治面貌为中共党员(含预备党员)的50人(5.3%),非党员900人(94.7%);学生干部97人(10.2%),非学生干部853人(89.8%)。

2.2研究工具。

通过对国内外大量文献梳理,建构了包含五个因素的大学生政府信任初测问卷,分别为基本信息、政府信任(中央政府、省级政府、市级政府、县区政府、乡镇街道政府)、理性文化路径(制度信任、过程信任、人员信任、结果信任、生活满意度)、社会文化路径(人际信任)、政治参与路径(制度性参与、非制度行参与)。对初测问卷数据实施因子分析和信度分析,最终确定正式问卷包括以上4个维度,共19题。问卷中每个维度的信度检验结果,表明青少年政府信任影响因素问卷是稳定可信的。题项信度分析,见表1。

3调查结果。

3.1青少年政府信任的总体状况。

数据结果显示,在青少年政府信任的四维结构中,受访学生各维度的题项均分在3.591-3.704,处于中等偏上水平。青少年政府信任的总量表得分为3.621,也处于中等偏上水平[7]。青少年政府信任的总体状况,见表2.

青少年在“结果信任”维度上的得分最低,在“制度信任”维度上的得分最高,各维度平均值的大小顺序为:制度信任过程信任人员信任结果信任。

3.2青少年政府信任影响因素的多元回归。

为了分析和比较理性选择路径、社会文化路径以及政治参与路径对于青少年政府信任的影响,构建了四个多元统计分析模型,具体结果见表2。在控制变量方面,模型一到三的结果显示,性别、年级、政治面貌、生源地均对青少年政府信任没有统计上的影响,学生干部这一个变量却具有显著差异。三种理论解释路径对青少年政府信任的影响,见表3。

模型1关注理性选择路径对于因变量的影响。结果显示,除制度信任外,过程信任、人员信任、结果信任、生活满意度均与政府信任具有显著的关系,由此可以推断理性选择路径对于青少年的政府信任具有强大的预测作用。

模型2关注社会文化路径对于因变量的影响。结果显示,人际信任与青少年政府信任具有显著的关系。

模型3关注政治參与路径对于因变量的影响。结果显示,政治参与与青少年政府信任具有显著的关系。

模型4采用逐步回归的方式,保留显著影响青少年政府信任的因子。从表中可以看出,理性选择路径对青少年政府信任的解释力最强(0.282+0.137+0.162=0.581),其次为社会文化路径(0.149),最后为政治参与路径(0.134)。由此可见三种解释路径都对青少年政府信任具有一定的预测力。

4结论与建议。

从表2可以看出,理性选择路径对青少年政府信任的解释力最强,这与现在文献研究结果较为一致。因此,在中国特色社会主义进入新时代的社会背景下,各级政府仍需加大教育投入,加强校园软硬件建设,热爱学生、关注学生、善待学生,不断提高办学水平,让青少年切切实实体会到国家经济和社会发展的成果,提高广大青少年的学校满意度。

社会文化路径(人际信任)对青少年的政府信任有重要的塑造作用。当前青少年学生的人际信任不容乐观,研究发现我国大学生人际信任水平与年代之间呈显著的负相关,说明大学生人际信任正逐年降低[8]。从学校角度来讲,应该通过多种形式加强青少年的“三观”培养和教育,增加社会责任感,提升人格,完善道德,不断提高对学校、教师和同学的信任水平。

政治参与路径对青少年的政府信任有重要影响。学校要通过思想政治教育理论课、选修课、专题讲座、主题班会等形式,加强大学生政治主体观教育,使大学生充分认识其政治权利和义务,增强大学生的政治效能感,提高大学生的政府信任度。高校应该从学生全面发展的高度出发,开设政治实践活动和政治技能训练。积极推行学生自我管理、自我服务、自我教育,鼓励学生参与学校管理,提出建设性的意见和建议;积极与当地政府部门和团群组织协作,开展丰富多彩的政治实践活动,丰富大学生的政治参与经验。

参考文献:

[2]冯岩.当代中国大学生政治信任研究[d].济南大学,2015.

摘要:及时回应人民群众的期盼和关切是现代政府的一个重要标志。回应人民群众期盼和关切,宏观上要保证经济社会稳定发展,微观上要在具体事件中保障人民切身利益。提升政府回应能力是建设服务型政府的内在要求,是建立和谐社会的有力保障,是增强政府公信力的重要途径,是政府执政能力不可或缺的重要方面,是衡量政府执政水平的重要标杆。

全球治理委员会在《我们的全球之家》的报告中指出,治理有六个基本要素:参与、公开、透明、回应、法治和责任。回应作为治理的要素之一,要求政府在治理过程中,要与老百姓互动,对公众的合法性要求作出及时、负责的反应,不能无故拖延或者不了了之。回应越快,治理的程度越高。

李克强总理说过,及时回应人民群众的期盼和关切是现代政府的一个重要标志。回应人民群众期盼和关切,宏观上要保证经济社会稳定发展,微观上要在具体事件中保障人民切身利益。

1.政府回应的内涵。

政府回应,“就是政府在公共管理过程中,对公众的需求和提出的问题作出积极敏感的反应和回复的过程”[1]。

政府回应的两点要求:其一,政府部门对公众的需求和提出的问题具有敏感性,要及时作出反应,不能置之不理;其二,政府在敏感性的基础上,要作出积极有效的回复,满足公众需求或解决群众问题。

政府回应是个互动过程:对公众来说,可以通过利益诉求渠道表达自身的要求与期望;对政府来说,要及时有效地对公众的诉求给予反应和回复。政府回应的结果可能是一种积极的表态,表明政府对问题的重视;也可能是对满足公众需求,还可能是对公众要求的一种说服和引导[2]。

2.政府回应的要素。

一是回应主体。也就是“谁回应”,主要是政府和政府部门及其工作人员。二是回应对象,也就是“回应谁”,既可以是公民个体,也可以是社会组织,社会大众。三是回应客体,也就是“回应什么”,指公众提出的需要政府满足或解答的诉求。这种诉求可以是明确表达出来的,也可以是潜在的;既可以是反映社会整体利益的诉求,也可以是个人的具体诉求。四是回应渠道,也就是“怎么回应”,即回应的方式和渠道。五是回应反馈,也就是“回应的如何”,即公众对政府的回应作出的反应性信息,并把这种信息反馈给政府。

1.提升政府回应能力是建设服务型政府的内在要求。

传统管制型社会中,政府也要根据社会需要去做事。但由于社会事务相对来说比较简单,而且公众很少有机会向政府提出要求,因此政府不会承受太多回应性的压力。政府之于公众来说,更多的是管控者,缺少对公众的服务和诉求的回应。

服务型政府的建设意味着政府角色由管控者向服务者的转变。服务型政府,以服务人民、服务社会为宗旨。为了更好地提供服务,政府需要准确掌握公众的需求,并及时满足公众需要,解決公众问题,这个过程就是政府积极有效的回应过程。因此,提升政府回应能力是建设服务型政府的内在要求。

2.提升政府回应能力是建立和谐社会的有力保障。

说文解字看“和谐”二字。“和”由“禾”和“口”构成,“禾”代表粮食,“口”表示人。据此,“和”就是指人人有饭吃。“谐”由“言”和“皆”构成,“言”指说话、表达,“皆”是全部的意思。据此,“谐”就是指人人都有表达的、说话的权力,能够畅所欲言。那么“和”“谐”二字连起来,为我们描绘的和谐社会的场景就是:人民在满足物质生活的同时,表达权、参与权、监督权等也都能够得到保障。要实现这样一副场景,就要增强政府的回应性[3]。

而且,从社会稳定的角度说,政府能够及时了解民情、掌握民意,对涉及群众利益的问题能够早发现、早处理、早解决,给予及时回应,才能消除社会中存在的不和谐因素,实现社会的稳定发展。

3.提升政府回应能力是增强政府公信力的重要途径。

2014年3月18日,习近平总书记在河南省兰考县县委常委扩大会议上的讲话中,提到过“塔西佗陷阱”。通俗地讲,“塔西佗陷阱”就是当执掌公权力的政府部门失去公信力时,无论其说真话还是假话,都会被认为说假话;无论其做好事还是坏事,都会被认为是做坏事。习近平总书记在讲话中指出,“我们当然没有走到这一步,但存在的问题也不可谓不严重,必须下大气力加以解决。如果真的到了那一天,就会危及党的执政基础和执政地位。”[4]公信不立,患莫大焉。人民对政府的信任,是政府权威的基础。

提升政府回应能力是增强政府公信力的重要途径。政府回应是政府公信力最直接的来源。如果把政府公信力看作一个“产品”,在它的生产过程中,人民的诉求是“需求侧”,政府回应就是“供给侧”。群众通过各种渠道发出呼声,向政府提出要求,实际上是群众有对政府产生信任的需求,希望政府是一个关心人民疾苦、能够为人民排忧解难的机构,而政府则通过回应向人民群众提供可以信任的政策资源,并使人民知晓政府是关心并愿意保护他们切身利益的可以信赖的管理者。在呼应之间,“需求”和“供给”发生了相互作用并对接起来,政府与人民之间的信任由此建立,并成为一种稳定的社会资本,政府公信力亦随之产生[5]。

在需求与供给间,就像社会经济领域要从拉动需求转向供给侧改革一样,政府不仅要及时回应人民提出的需求,更应该积极主动地了解社会发展和人民权益状况,在需求产生之前主动响应,提高政策水平、提升执政能力、强化服务意识,时刻将改善民生作为政府职能第一要务,以高质量的公共产品和公共服务的“供给”满足人民的需求,保障人民权益,保持政府公信力不断产生和稳定增长[5]。

随着公众参与意识、表达意识、监督意识的增强,公众对于诸如分配制度改革、柴米油盐的价格、房屋拆迁的政策、化工厂选址方案这些问题越来越关注,不仅想了解政府部门“怎么看”,更想知道接下来“怎么办”。政府“回应”正由选答题变为必答题。坦率地说,中央在深化改革开放、高压反腐、改进作风、厉行节约等宏观层面对公众呼声给予了积极回应,但不少地方政府或相关部门回应在新闻事件、网络爆料等质疑时,仍时常显得迟疑、被动甚至恐惧。推进民主政治、打造阳光政府依然成为时代潮流。顺应时代潮流,“回应社会关切”是政府建设的内在要求,知情、参与、表达和监督是必须保障的公众权利,因此,各级政府及政府工作部门必须尽快适应并学会在质疑、追问的“杂音”中执政,就成为一门必修课,无法回避,更不可逃避!

参考文献:

[1]何祖坤.关注政府回应[j].中国行政管理,2000(7).

[2]李严昌.政府回应过程研究[m].北京:中国社会科学出版社,2018.

[4]中共中央文献研究室.做焦裕禄式的县委书记[m].北京:中央文献出版社,2015.

[5]张欧阳.政府回应:政府公信力产生机制的“供给侧”[j].江汉论坛,2017(4).

摘要:自从分税制模式实施后,出现了中央与地方政府财权和事权不匹配的问题。2008年金融危机对我国的经济影响非常大,于是我国扩大内需,推出了4万亿元的投资计划。很多地方政府融资平台开始迅速扩张,扩大自己的融资规模。然而这种现象也产生了负面影响,各地政府实施不同的项目,导致政府债务规模不断扩大,政府债务风险严重。鉴于此,对政府财务风险进行分析,并提出解决政府财务风险的对策。

关键词:政府债务;财务风险;防范对策。

我国从1994年开始进行分税制改革,导致政府的财权和事权不统一,政府支出的责任变更非常频繁,但是没有相应的财力作为支撑。2008年,我国为了应对金融危机,提出了4万亿政策,结合中央政府的投资计划,地方政府开展了各种形式的融资。大规模的融资导致债务隐性化,债务风险非常严峻。在党的十九大报告中,明确提出要防范各类风险。在风险防控中,政府的债务风险防控是重点,从而有效地预防金融风险的发生。

1.政府财权事权与支出责任不符。自从1994年我国实施分税制改革后,税种划分成中央税种和地方税种。地方的事权不断增多,导致地方性政府财政支出越来越多。随着城镇化进程的加快,居民对公共产品的需求量非常大,政府需要更多的资金。地方政府为了进行基础设施建设,他们每年都要融资,产生债务风险。地方政府的财政收入非常有限,受到中央强化房地产调控的影响,在转移支付环节中存在各类问题,不能结合地方政府的实际需求。在增量调节环节,地方之间的贫富差距非常大。地方政府的事权不断增加,但是财权非常有限。地方政府体制内的收入不能弥补财政支出,导致地方政府的预算外收入增加,他们只能通过举债的方式弥补财政不足。

2.政府融资不够规范。政府在融资环节中,产生非常大的随意性。地方政府常常通过发行债券的方式,但是资金还是不能满足需求。于是,地方政府开始建立各类融资平台,在融资环节中产生盲目性和随意性。政府的融资运作效率不高,融资的成本非常高,导致政府投资效率不佳。在融资平台建设中,资金不足。政府会通过各类方式补充资金,通过挪用年度预算等方式,甚至出现虚假注资的方式。政府在融資方面,相关的管理体制存在缺陷,很多融资平台的建设没有相关的监督措施加以辅助,采用多头管理的方式。融资监督机制尚未建立,政府在融资环节中,缺乏法律和公众的监督,导致商业腐败的产生。

3.政府债务管理机制不够健全。政府的债务管理机构不完善,尽管我国政府已经积累了很多政府性债务,但是债务管理机构尚不完善。债务资金使用不够规范,有些地方政府的债务资金不能纳入到预算管理环节,债务资金不能充分进行基础设施建设。债务资金管理效果不好,不能进行全方位的预算管理。政府债务领域的信息不够透明,地方政府的融资情况不能及时地回报给人大常委,信息不公开,导致地方性债务的口径不能完全的纳入到预算管理系统中。债务管理的法制建设比较落后,不能结合债务管理建立法律体系。尽管我国制定了《预算法》等,对政府的举债行为加以约束,但随着社会经济的发展,政府投资还是越来越多,已经超过政府的财力。

1.提升思想认识,强化债务管理。政府应强化风险认识,防止债务风险扩散为经济风险,要正确地认识债务风险问题,纵观大局,做到未雨绸缪。政府应树立忧患意识,认识到抵御债务风险的紧迫性,也要认识到政府举债对当地经济发展产生的影响,确保偿债有序。强化底线意识,底线意识在一定程度上是底线思维的应用,其与风险意识是相辅相成的,政府要在对现实情况进行分析的基础上,提高警惕。将国家法律作为“红线”,做好风险的防范准备工作。政府还要提升自身的法治意识,杜绝违法违规的举债行为。有些政府的领导干部,他们的法治意识比较弱,对于一些惩戒视而不见。因此,政府应树立法治意识,规范举债行为,维护法律的权威。充分了解政府职能的权限,掌握政府和市场之间的关系,强化政府职能的转变,发挥市场这只无形手的资源配置作用。

2.推进体制机制改革。合理划分财权和事权,中央政府和地方政府应该明确财政事权和支出责任,有效地抑制政府债务规模。结合权、责和利相统一的原则,各地区的财权和事权应得到明确,制定财权事权调整机制,形成完整和清晰的财权和事权清单,从而提升政府对于财权和事权改革的可操作性。建立相应的配套措施,完善相关的改革协同配套方案,提升转移支付制度的可操作性。进行干部任用制度的改革,完善债务监督考核问责制度。对考核的方法进行改进,抓住基础和发展的根基,充分发挥潜能,改善民生,促进社会进步。完善政府债务考核和问责制度,建立公共服务为中心的理念。在新政策背景下,完善债务监督制度,对金融机构进行科学的监管工作,有效地杜绝违法违规操作,强化责任人制度。对政府债券资金进行全面管理,明确责任主体和财务主体。提升债务信息的透明度,通过政府政务网站的方式,对债务限额明确,建立债务限额报告制度。通过财政预算的方式,对预算情况进行分析。

3.提升政府债务风险管理能力。建立政府债务管理小组,发挥其职能。对政府债务的借、用和还阶段进行监督,进行债务管理工作的指导,对于债务风险要实时掌握。当风险发生后,应采取必要的干预措施。完善政府债务的预算管理工作,分别对政府债务收入、支出、付息和还本进行预算,防止超预算举债的发生。各地的人大应发挥审议作用,充分履行监督职能。提升风险化解的能力,在政府债务管理中,隐性债务管理存在一定的难度。因此,在债务管理中,要对各类风险进行评估,出具评估报告。有些政府债务风险防范能力比较差,风险防控机制比较落后。因此,要提升政府债务风险化解能力,抓住风险产生的主要因素。

政府债务风险是重大风险的类型之一,因此,我国应有效地防范和化解政府财务风险,促进全面小康社会的建设。同时,政府应提升债务风险的化解能力,树立危机意识。

参考文献:

[2]胡才龙,魏建国.多任务委托代理模型下地方政府债务管理激励契约设计——基于省级面板数据的实证检验[j].审计与经济研究,2019,(5):118-127.

政府业绩评价与传统政府部门管理方法不同,是一种新的管理模式,体现了时代发展对政府行政体制改革的要求。其中,第三方评价作为政府业绩管理的重要机制,能够有效地进行外部平衡,弥补传统政府自我评价的缺陷,在促进我国服务型政府变革方面发挥了不可或缺的推进作用。从西方国家实施第三者评价的经验来看,第三者是指与被评价对象和服务对象独立的第三者,由于第三者和被评价对象(第一者)、被服务对象(第二者)没有行政所属关系和利益关系,也被称为独立的第三者。在西方国家,政府业绩第三者的评价多由一些专业的非政府组织(ngo)作为评价机构。这些非政府组织可以保证第三者的独立性、专业性和权威性要求。

从实践层面看,中国政府业绩的传统评价模式以内部进行的自我评价为主,成为运动员和裁判员,评价过程关闭,评价效果差,影响政府的信赖力。近年来大力发展的政府业绩第三方评价,有效提高政府的信赖力,有助于完善政府业绩监督机制。我国从上世纪90年代初开始重视政府业绩评价,从西方参考经验,从90年代中期开始进行政府业绩评价的理论研究,至今为止。

止,

以“政府绩效与公众信任”为撰写小论文范文

摘要:本文主要探究政府绩效对城市居民政府信任程度的影响,对我国现阶段的政府工作进行分析,希望能够提升城市居民对政府的信任程度。

关键词:政府绩效;

政府信任;

城市居民。

而政府代理公众行使行政权利,并通过履行相应的职能获得相关利益。因此,政府的信任是建立在委托、代理关系之上,一旦政府无法满足公众的期望,就会出现信任危机,从而导致两者之间原有的和谐关系破裂,甚至威胁到社会和国家的稳定。

政府绩效视角,主要是通过对政府的表现进行评估,分析政府信任的原因,即确定政府是否值得信任。个人对政府的信任程度受到政府行政能力、经济总体形势、政策执行等多方面的影响。在国外的研究中,将公众不信任政府的原因归纳为以下几点:政府工作效率低下、公款浪费严重、政策执行错误等原因。

政府绩效是影响政府政治信任程度的关键所在。政府绩效主要的参考标准是政府在社会经济管理过程中的行政结果、行政效益、行政效能,以及对政府在行使职能的过程中的管理能力进行研究[1]。政府的绩效是政府执政能力的表现,主要分为社会效益、经济效益和政治效益是三个方面。从理论上分析,政府信任程度对政府工作的顺利运作具有重大意义,因此它也是政治生活中的重要组成部分。

在我国城市居民中,首先城市居民将政府看作是一个统一的整体。城市居民对地方政府的评价相对较高,对中央政府和省政府的评价相对较低。虽然城市居民对中央政府的评价略低,这也体现出地方政府对城市居民日常生活的作用和影响,表明了城市居民对政府信任的理性。其次政府的绩效能够有效提升政府的信任程度,通过政府决策的透明性、依法治国的执行力度、贫富差距的改善情况、社会治安的管理情况,都会对政府信任程度有所加分。因此对于地方政府而言,城市居民的信任程度需求明显对政府绩效提出了更高的要求,只有这样,才能从根本上提升城市居民对政府的信任程度。

结束语。

随着改革开放的深入,我国政府在经济建设方面取得了巨大的成就,但是由于目前我国正处于社会转型期,在经济发展的同时也伴随着贫富差距加大、社会矛盾激化等多种民生问题,由于住房、医疗、教育等问题未能得到有效的解决。这样不仅严重削弱了政府的执政能力,降低了行政效率,也在一定程度上影响了居民对政府的信任程度,所以在一定程度上阻碍了到国家和社会的良性发展。因此,要求政府必须通过有效措施提升政府绩效,为国家转型提供必要的助力和保证,确保服务型政府的构建。

参考文献:

以“政府绩效与公众信任”为撰写小论文范文

摘要:及时回应人民群众的期盼和关切是现代政府的一个重要标志。回应人民群众期盼和关切,宏观上要保证经济社会稳定发展,微观上要在具体事件中保障人民切身利益。提升政府回应能力是建设服务型政府的内在要求,是建立和谐社会的有力保障,是增强政府公信力的重要途径,是政府执政能力不可或缺的重要方面,是衡量政府执政水平的重要标杆。

全球治理委员会在《我们的全球之家》的报告中指出,治理有六个基本要素:参与、公开、透明、回应、法治和责任。回应作为治理的要素之一,要求政府在治理过程中,要与老百姓互动,对公众的合法性要求作出及时、负责的反应,不能无故拖延或者不了了之。回应越快,治理的程度越高。

李克强总理说过,及时回应人民群众的期盼和关切是现代政府的一个重要标志。回应人民群众期盼和关切,宏观上要保证经济社会稳定发展,微观上要在具体事件中保障人民切身利益。

1.政府回应的内涵。

政府回应,“就是政府在公共管理过程中,对公众的需求和提出的问题作出积极敏感的反应和回复的过程”[1]。

政府回应的两点要求:其一,政府部门对公众的需求和提出的问题具有敏感性,要及时作出反应,不能置之不理;其二,政府在敏感性的基础上,要作出积极有效的回复,满足公众需求或解决群众问题。

政府回应是个互动过程:对公众来说,可以通过利益诉求渠道表达自身的要求与期望;对政府来说,要及时有效地对公众的诉求给予反应和回复。政府回应的结果可能是一种积极的表态,表明政府对问题的重视;也可能是对满足公众需求,还可能是对公众要求的一种说服和引导[2]。

2.政府回应的要素。

一是回应主体。也就是“谁回应”,主要是政府和政府部门及其工作人员。二是回应对象,也就是“回应谁”,既可以是公民个体,也可以是社会组织,社会大众。三是回应客体,也就是“回应什么”,指公众提出的需要政府满足或解答的诉求。这种诉求可以是明确表达出来的,也可以是潜在的;既可以是反映社会整体利益的诉求,也可以是个人的具体诉求。四是回应渠道,也就是“怎么回应”,即回应的方式和渠道。五是回应反馈,也就是“回应的如何”,即公众对政府的回应作出的反应性信息,并把这种信息反馈给政府。

1.提升政府回应能力是建设服务型政府的内在要求。

传统管制型社会中,政府也要根据社会需要去做事。但由于社会事务相对来说比较简单,而且公众很少有机会向政府提出要求,因此政府不会承受太多回应性的压力。政府之于公众来说,更多的是管控者,缺少对公众的服务和诉求的回应。

服务型政府的建设意味着政府角色由管控者向服务者的转变。服务型政府,以服务人民、服务社会为宗旨。为了更好地提供服务,政府需要准确掌握公众的需求,并及时满足公众需要,解決公众问题,这个过程就是政府积极有效的回应过程。因此,提升政府回应能力是建设服务型政府的内在要求。

2.提升政府回应能力是建立和谐社会的有力保障。

说文解字看“和谐”二字。“和”由“禾”和“口”构成,“禾”代表粮食,“口”表示人。据此,“和”就是指人人有饭吃。“谐”由“言”和“皆”构成,“言”指说话、表达,“皆”是全部的意思。据此,“谐”就是指人人都有表达的、说话的权力,能够畅所欲言。那么“和”“谐”二字连起来,为我们描绘的和谐社会的场景就是:人民在满足物质生活的同时,表达权、参与权、监督权等也都能够得到保障。要实现这样一副场景,就要增强政府的回应性[3]。

而且,从社会稳定的角度说,政府能够及时了解民情、掌握民意,对涉及群众利益的问题能够早发现、早处理、早解决,给予及时回应,才能消除社会中存在的不和谐因素,实现社会的稳定发展。

3.提升政府回应能力是增强政府公信力的重要途径。

2014年3月18日,习近平总书记在河南省兰考县县委常委扩大会议上的讲话中,提到过“塔西佗陷阱”。通俗地讲,“塔西佗陷阱”就是当执掌公权力的政府部门失去公信力时,无论其说真话还是假话,都会被认为说假话;无论其做好事还是坏事,都会被认为是做坏事。习近平总书记在讲话中指出,“我们当然没有走到这一步,但存在的问题也不可谓不严重,必须下大气力加以解决。如果真的到了那一天,就会危及党的执政基础和执政地位。”[4]公信不立,患莫大焉。人民对政府的信任,是政府权威的基础。

提升政府回应能力是增强政府公信力的重要途径。政府回应是政府公信力最直接的来源。如果把政府公信力看作一个“产品”,在它的生产过程中,人民的诉求是“需求侧”,政府回应就是“供给侧”。群众通过各种渠道发出呼声,向政府提出要求,实际上是群众有对政府产生信任的需求,希望政府是一个关心人民疾苦、能够为人民排忧解难的机构,而政府则通过回应向人民群众提供可以信任的政策资源,并使人民知晓政府是关心并愿意保护他们切身利益的可以信赖的管理者。在呼应之间,“需求”和“供给”发生了相互作用并对接起来,政府与人民之间的信任由此建立,并成为一种稳定的社会资本,政府公信力亦随之产生[5]。

在需求与供给间,就像社会经济领域要从拉动需求转向供给侧改革一样,政府不仅要及时回应人民提出的需求,更应该积极主动地了解社会发展和人民权益状况,在需求产生之前主动响应,提高政策水平、提升执政能力、强化服务意识,时刻将改善民生作为政府职能第一要务,以高质量的公共产品和公共服务的“供给”满足人民的需求,保障人民权益,保持政府公信力不断产生和稳定增长[5]。

随着公众参与意识、表达意识、监督意识的增强,公众对于诸如分配制度改革、柴米油盐的价格、房屋拆迁的政策、化工厂选址方案这些问题越来越关注,不仅想了解政府部门“怎么看”,更想知道接下来“怎么办”。政府“回应”正由选答题变为必答题。坦率地说,中央在深化改革开放、高压反腐、改进作风、厉行节约等宏观层面对公众呼声给予了积极回应,但不少地方政府或相关部门回应在新闻事件、网络爆料等质疑时,仍时常显得迟疑、被动甚至恐惧。推进民主政治、打造阳光政府依然成为时代潮流。顺应时代潮流,“回应社会关切”是政府建设的内在要求,知情、参与、表达和监督是必须保障的公众权利,因此,各级政府及政府工作部门必须尽快适应并学会在质疑、追问的“杂音”中执政,就成为一门必修课,无法回避,更不可逃避!

参考文献:

[1]何祖坤.关注政府回应[j].中国行政管理,2000(7).

[2]李严昌.政府回应过程研究[m].北京:中国社会科学出版社,2018.

[4]中共中央文献研究室.做焦裕禄式的县委书记[m].北京:中央文献出版社,2015.

[5]张欧阳.政府回应:政府公信力产生机制的“供给侧”[j].江汉论坛,2017(4).

以“政府绩效与公众信任”为撰写小论文

公众是指公共关系的主体成员面临共同问题、共同利益和共同要求的社会群体。以下是为大家整理的关于,欢迎品鉴!

摘要:及时回应人民群众的期盼和关切是现代政府的一个重要标志。回应人民群众期盼和关切,宏观上要保证经济社会稳定发展,微观上要在具体事件中保障人民切身利益。提升政府回应能力是建设服务型政府的内在要求,是建立和谐社会的有力保障,是增强政府公信力的重要途径,是政府执政能力不可或缺的重要方面,是衡量政府执政水平的重要标杆。

全球治理委员会在《我们的全球之家》的报告中指出,治理有六个基本要素:参与、公开、透明、回应、法治和责任。回应作为治理的要素之一,要求政府在治理过程中,要与老百姓互动,对公众的合法性要求作出及时、负责的反应,不能无故拖延或者不了了之。回应越快,治理的程度越高。

李克强总理说过,及时回应人民群众的期盼和关切是现代政府的一个重要标志。回应人民群众期盼和关切,宏观上要保证经济社会稳定发展,微观上要在具体事件中保障人民切身利益。

1.政府回应的内涵。

政府回应,“就是政府在公共管理过程中,对公众的需求和提出的问题作出积极敏感的反应和回复的过程”[1]。

政府回应的两点要求:其一,政府部门对公众的需求和提出的问题具有敏感性,要及时作出反应,不能置之不理;其二,政府在敏感性的基础上,要作出积极有效的回复,满足公众需求或解决群众问题。

政府回应是个互动过程:对公众来说,可以通过利益诉求渠道表达自身的要求与期望;对政府来说,要及时有效地对公众的诉求给予反应和回复。政府回应的结果可能是一种积极的表态,表明政府对问题的重视;也可能是对满足公众需求,还可能是对公众要求的一种说服和引导[2]。

2.政府回应的要素。

一是回应主体。也就是“谁回应”,主要是政府和政府部门及其工作人员。二是回应对象,也就是“回应谁”,既可以是公民个体,也可以是社会组织,社会大众。三是回应客体,也就是“回应什么”,指公众提出的需要政府满足或解答的诉求。这种诉求可以是明确表达出来的,也可以是潜在的;既可以是反映社会整体利益的诉求,也可以是个人的具体诉求。四是回应渠道,也就是“怎么回应”,即回应的方式和渠道。五是回应反馈,也就是“回应的如何”,即公众对政府的回应作出的反应性信息,并把这种信息反馈给政府。

1.提升政府回应能力是建设服务型政府的内在要求。

传统管制型社会中,政府也要根据社会需要去做事。但由于社会事务相对来说比较简单,而且公众很少有机会向政府提出要求,因此政府不会承受太多回应性的压力。政府之于公众来说,更多的是管控者,缺少对公众的服务和诉求的回应。

服务型政府的建设意味着政府角色由管控者向服务者的转变。服务型政府,以服务人民、服务社会为宗旨。为了更好地提供服务,政府需要准确掌握公众的需求,并及时满足公众需要,解決公众问题,这个过程就是政府积极有效的回应过程。因此,提升政府回应能力是建设服务型政府的内在要求。

2.提升政府回应能力是建立和谐社会的有力保障。

说文解字看“和谐”二字。“和”由“禾”和“口”构成,“禾”代表粮食,“口”表示人。据此,“和”就是指人人有饭吃。“谐”由“言”和“皆”构成,“言”指说话、表达,“皆”是全部的意思。据此,“谐”就是指人人都有表达的、说话的权力,能够畅所欲言。那么“和”“谐”二字连起来,为我们描绘的和谐社会的场景就是:人民在满足物质生活的同时,表达权、参与权、监督权等也都能够得到保障。要实现这样一副场景,就要增强政府的回应性[3]。

而且,从社会稳定的角度说,政府能够及时了解民情、掌握民意,对涉及群众利益的问题能够早发现、早处理、早解决,给予及时回应,才能消除社会中存在的不和谐因素,实现社会的稳定发展。

3.提升政府回应能力是增强政府公信力的重要途径。

2014年3月18日,习近平总书记在河南省兰考县县委常委扩大会议上的讲话中,提到过“塔西佗陷阱”。通俗地讲,“塔西佗陷阱”就是当执掌公权力的政府部门失去公信力时,无论其说真话还是假话,都会被认为说假话;无论其做好事还是坏事,都会被认为是做坏事。习近平总书记在讲话中指出,“我们当然没有走到这一步,但存在的问题也不可谓不严重,必须下大气力加以解决。如果真的到了那一天,就会危及党的执政基础和执政地位。”[4]公信不立,患莫大焉。人民对政府的信任,是政府权威的基础。

提升政府回应能力是增强政府公信力的重要途径。政府回应是政府公信力最直接的来源。如果把政府公信力看作一个“产品”,在它的生产过程中,人民的诉求是“需求侧”,政府回应就是“供给侧”。群众通过各种渠道发出呼声,向政府提出要求,实际上是群众有对政府产生信任的需求,希望政府是一个关心人民疾苦、能够为人民排忧解难的机构,而政府则通过回应向人民群众提供可以信任的政策资源,并使人民知晓政府是关心并愿意保护他们切身利益的可以信赖的管理者。在呼应之间,“需求”和“供给”发生了相互作用并对接起来,政府与人民之间的信任由此建立,并成为一种稳定的社会资本,政府公信力亦随之产生[5]。

在需求与供给间,就像社会经济领域要从拉动需求转向供给侧改革一样,政府不仅要及时回应人民提出的需求,更应该积极主动地了解社会发展和人民权益状况,在需求产生之前主动响应,提高政策水平、提升执政能力、强化服务意识,时刻将改善民生作为政府职能第一要务,以高质量的公共产品和公共服务的“供给”满足人民的需求,保障人民权益,保持政府公信力不断产生和稳定增长[5]。

随着公众参与意识、表达意识、监督意识的增强,公众对于诸如分配制度改革、柴米油盐的价格、房屋拆迁的政策、化工厂选址方案这些问题越来越关注,不仅想了解政府部门“怎么看”,更想知道接下来“怎么办”。政府“回应”正由选答题变为必答题。坦率地说,中央在深化改革开放、高压反腐、改进作风、厉行节约等宏观层面对公众呼声给予了积极回应,但不少地方政府或相关部门回应在新闻事件、网络爆料等质疑时,仍时常显得迟疑、被动甚至恐惧。推进民主政治、打造阳光政府依然成为时代潮流。顺应时代潮流,“回应社会关切”是政府建设的内在要求,知情、参与、表达和监督是必须保障的公众权利,因此,各级政府及政府工作部门必须尽快适应并学会在质疑、追问的“杂音”中执政,就成为一门必修课,无法回避,更不可逃避!

参考文献:

[1]何祖坤.关注政府回应[j].中国行政管理,2000(7).

[2]李严昌.政府回应过程研究[m].北京:中国社会科学出版社,2018.

[4]中共中央文献研究室.做焦裕禄式的县委书记[m].北京:中央文献出版社,2015.

[5]张欧阳.政府回应:政府公信力产生机制的“供给侧”[j].江汉论坛,2017(4).

[摘要]自从新媒体迅速发展,我国网民规模大幅度扩大。文章运用中国综合社会调查(cgss)2015网民社会意识调查的公开数据,探究普遍信任和政府信任现状以及普遍信任对政府信任的影响效果。研究结果表明,普遍信任同意度介于同意和中立之间,政府信任度呈现“央强地弱”的总体格局。普遍信任对政府信任存在显著的负向影响,且对中央政府的影响程度要高于地方政府。

[关键词]新媒体;普遍信任;政府信任;影响。

[中图分类号]d035[文献标识码]a。

随着新媒体时代的到来,政治生活与网络媒介密切相关,据中国互联网络信息中心(cnnic)发布的第42次《中国互联网络发展状况统计报告》显示,截至2018年6月30日,我国网民规模达8.02亿,互联网普及率为57.7%。可见,网民是民众的一部分,网民在网上表达自己的感受和意愿,体现强大真实的民意。而人际信任和政府信任是信任范畴的两个重要方面。在信任的测量上,人际信任属于社会信任的领域。韦伯(1995)将其划分为普遍信任和特殊信任,普遍信任以相同信仰为基础,也称和陌生人之间的信任;特殊信任以血缘或裙带关系为纽带,建立在自己所熟知的群体之上。而政府信任是民众对政府或政治制度所持有信心的正面期望。因此,网民的政府信任度不仅能够反映民众政府信任的重要组成部分,而且可以在某种程度上展现着社会整体信任水平和国家政府信任的未来趋势。诸多学者在关于人际信任与政府信任的关系研究中,均一致认为人际信任特别是普遍信任是政府信任的基础,普遍信任可以作为解释居民政府信任的有效变量。且随着新媒体的快速发展,网民获取信息方式多元化,网民能从微博、微信等多渠道更及时地了解官方媒体未能及时报道的事态发展与事实真相,可能更能客观、理性地看待政府行为。因此,本文在借鉴前人关于人际信任与政治信任相关研究的基础上,运用cgss2015网民社会意识的调查数据,利用spss软件对调查数据进行深入分析,以了解当前网民对政府信任的现状以及探讨普遍信任对政府信任的影响程度。

1文献回顾与述评。

普遍信任是社会关系网络中重要的社会资本组成要素之一,长期以来深受国内外学者重视,他们通过理论和实证两个层面对人际信任与政府信任之间的关系进行探讨,得出不同的结论。尤斯拉纳(2006)在研究中发现,政府部门之间信任水平会随着人际信任的下跌而降低,政府工作效率会随之发生影响,导致公众对政府信任度下降,即人际信任对政府信任产生正向影响。而stolle(1998)认为,普遍信任会对自己所属的公共组织产生负向影响,即普遍信任越高,政府信任越低。同时,还有部分学者认为人际信任与政府信任之间不存在显著关系,如帕特南(2011)认为应该从理论上区分人际信任和政府信任,二者可能不存在相關关系。

近年来,我国不少学者将普遍信任作为解释变量、政府信任作为被解释变量来研究普遍信任对政府信任的影响。如谢治菊(2013)在基于江浙和贵州高校的实证研究中发现,普遍信任的增强对政府信任的提升有着重要的促进作用。张海良、许伟(2015)在探讨普遍信任与政府信任的关系中,提出普遍信任程度越高,政府信任会随之产生正向影响。赵慧增、王思琦(2016)以西南交通大学为例,普遍信任对政府信任具有显著的正向影响,且与中央政府信任相比,普遍信任对地方政府的影响程度更大。

相较于大量学者研究普遍信任对政府信任的影响,部分国外学者认为信任可以分为政府信任和社会信任,两者是属于不同范畴的概念,并将政府信任作为自变量,普遍信任作为因变量进行研究,得出了相应的成果。什托姆普卡(2005)在对波兰的实证研究中指出,政府信任态势的提高促进了公众间的普遍信任。rahn(1999)、rothstein(2008)等认为,政府信任是影响公众信任他人的重要影响因素,公众对政府的信心程度的提升,直接促进公众信任社会其他人。

关于政府信任的影响因素研究,从不同的角度来看,考察的因素各不相同。从政府的角度来说,社会经济、依法行政、社会治安状况等政府绩效发展良好会提高公众对政府的满意度,进而影响政府信任度。(胡荣、胡康等,2011)从公众的角度来说,公众对公共事务的参与程度展现了他们的互惠态度、社会网络和人际信任的结果,政府关注公民的要求,对行政会产生一定的影响,公民的意愿得以表达,最终促进政府信任度的提升。(高勇,2014)从传播的角度来看,媒体对政府形象的塑造起着不容忽视的作用,公众更关注媒体,便更能多渠道了解社会事件各种观点,进而更能客观、理性看待政府行为,相应地会表现出更加信任政府的态度。此外,刘建平、周云(2017)在对政府信任的影响因素进行探讨时,提出性别、年龄、政治面貌、受教育程度等个人特征因素和文化因素都会对政府信任产生影响。卢海阳、郑逸芳等(2016)基于对我国16个城市的实证研究中,发现引入公共政策满意度及对政府工作满意度后,政府信任程度表现得更加显著。

总的来说,前人对普遍信任与政府信任的关系研究存在争议性,并未有统一的认识,并且国内外研究以理论描述研究居多,定量研究较少。从政府信任影响因素来看,其潜变量还有待深入探究,学者大多从人口学特征变量、政府绩效论、社会资本论等潜变量来研究,可能疏漏了关于公众对公共政策等满意度的影响研究。并且,新媒体时代迅速发展,网民队伍不断扩大,媒体因素对政府信任程度起着举足轻重的作用,而国内外学者基于网民群体的检验较为缺乏。鉴于此,本文致力于通过cgss2015调查数据深入探讨普遍信任对政府信任的影响。

2数据来源与变量测定。

2.1数据来源。

本文所采用的变量的名称、定义和描述性统计如表2所示。

本文将政治信任划分为中央政府信任和地方政府信任,选取问卷中的问题“对中央政府/地方政府,请问您多大程度上信任它们”加以测量,将选项“非常不信任”“比较不信任”“一般”“比较信任”、“非常信任”分别赋值为1、2、3、4、5,平均值越高,表明网民政府信任程度越高。结果发现,中央政府信任均值明显高于地方政府,分别为3.01、1.97。且网民对中央政府比较信任比例最高,约占36.33%,这比地方政府多了20.43个百分比。可见,我国网民对中央政府的信任度明显高于对地方政府的信任度,这与谢星泉(2015)在研究政治信任结构与群体性事件中认为我国公民“央强地弱”的政治信任趋势相一致。

本文将普遍信任作为自变量,选取“在我们这个社会,大多数人是可以信任的”进行数据处理,将选项“非常不信任”“比较不信任”“一般”“比较信任”、“非常信任”分别赋值为1、2、3、4、5,平均值越高,表明网民普遍信任程度越高。结果发现,我国网民对普遍信任度介于“一般”与“比较信任”之间,约为3.27。如表4所示,网民普遍信任程度最高为“一般”,约占36.68%,其次为“比较信任”,约占35.98%。

朱春奎等(2017)认为性别、年龄、收入、教育程度等社会人口属性因素对政府信任具有不同程度的影响,卢海阳等(2016)在公共政策满意度与中央政府信任研究中认为人口学特征、公共政策满意度等方面对政府信任有一定的影响。罗龙真(2014)在基于cgss2010对政府信任与社会信任关联性实证研究中将教育程度、生活幸福感、社会公平感等作为控制变量,实证检验了政府信任的影响效应。生活幸福感本文借鉴前人的研究,选取性别、年龄、政治面貌、学历、职业等人口学特征、民主政治满意度、主观幸福感和时政关注频率为控制变量纳入回归模型中。

3网民政府信任的logistic回归分析。

为研究普遍信任是否对政府信任有正向影响,本文对网民的政府信任进行logistic回归分析。模型1和模型2考察人口学特征、民主政策满意度以及普遍信任变量对网民政府信任度的影响,统计结果见表5。

3.1普遍信任的影响。

模型1和模型2显示,在控制人口学变量及民主政治满意度、主观满意度、时政关注频率等控制变量后,普遍信任在10%的水平上负向影响中央政府信任度,但对地方政府信任度影响不显著,影响系数分别为-0.694和-0.248。这表明,网民普遍信任度越高,政府信任度越低,这与现有的相关的实证研究相抵触,如张海良,许伟等基于数据cgss2010对人际信任、社会公平与政府信任关系的实证研究。由于研究群体的不同,选取控制变量的不同,得出的结论也会有所不同。本文认为普遍信任度对政府信任度的负向影响的解释是:本文选取cgss2015网民社会意识的调查数据进行分析,新媒体背景下,网民更容易依托社交平台与他人交流,刚开始他们愿意选择相信对方,由于网络的虚拟世界存在很多动机不纯或者带有不良目的的行为,经过多起网络交友失范事件报道后,最终使网民趋于培养理性的信任态度,让政府信任度有较大的改变,由此对政府信任产生负向影响。

3.2人口学变量的影响。

在模型1和模型2中,性别在1%的水平上正向影响中央政府信任度,但对地方政府信任度影响不显著,系数分别为0.349和0.012。这可能存在的原因有:首先,这是我国“央强地弱”的政治信任格局的体现;其次,无论在劳动力市场还是政治参与中,性别歧视是常见的现象,这种刻板印象会直接影响公众对女性的信任和支持,长期以来女性便对政府产生较低额度的信任值。而年龄、政治面貌、学历、职业等其他人口学变量对中央政府和地方政府信任度影響均不显著。

3.3其他控制变量的影响。

在模型1、模型2中,网民主观幸福感在1%的水平上正向影响地方政府信任度,但对中央政府信任度未产生显著影响,影响系数分别为1.675和0.530。第一,公众在生活中获得了制度上的保障,自身的幸福感更能表现出来,由此会在一定程度上增强政府信任度,这与刘明明(2016)在对“社会信任对公众主观幸福感的影响研究”的结论相符。第二,地方政府保障制度的执行对公众基本生活以及权益保障有着较大程度的影响,而中央政府的提案等还需地方政府的试点以及真正执行,加之公众主观幸福感会在一定程度上增强政府信任度,最终主观幸福感对地方政府信任度影响更大。而民主政治满意度、时政关注频率均未对中央政府和地方政府产生显著影响。

4结论与讨论。

通过数据调查结果分析,本文得到以下结论:

第一,信任状况方面,我国网民普遍信任同意度平均值为2.78,介于“一般”和“比较信任”之间,中央政府信任均值明显高于地方政府,分别为3.01、1.97,呈现“央强地弱”的政治信任格局。

第二,对政府信任的影响方面,普遍信任对中央政府产生显著的负向影响,而对地方政府信任影响不显著。性别在1%的水平上正向影响中央政府信任度,但对地方政府信任度影响不显著。网民的主观幸福感在1%的水平上正向影响地方政府信任度,而未对中央政府产生显著影响。其他控制变量对中央政府和地方政府影响均不显著。

本文研究的创新点主要有:第一,选择在新媒体背景下考察网民的普遍信任状况,更具有针对性和新颖性。第二,采用logistic回归模型测量有无普遍信任协变量,其他潜在变量对政府信任的影响,研究结论证实了普遍信任对政府信任度有促进作用,同时也证实了其他潜在变量对其影响程度的变化。第三,运用实证方法,为学术界关于普遍信任度对政府信任度的影响争论提供新的理论依据。

[1][波兰]埃里克·尤斯拉纳,著,张敦敏译.信任的道德基础[m].北京:中国社会科学出版社,2006.

[2]stolle,gtogether,bowlingalone:thedevelopmentofgeneralizedtrustinvoluntaryassociations[j].politicalpsychology,1998(19).

[3][美]罗伯特·帕特南著,刘波,祝乃娟,张孜异,等译.独自打保龄球——美国社区的衰落与复兴[m].北京:北京大学出版社,2011.

[4]谢治菊.大学生特殊信任、普遍信任与政府信任成长之关联研究——基于江浙和贵州高校的实证调查[j].民族高等教育研究,2013(04).

[5]張海良,许伟.人际信任、社会公平与政府信任的关系研究——基于数据cgss2010的实证分析[j].理论与改革,2015(01).

[6]赵慧增,王思琦.研究生人际信任与政府信任关联性实证研究——以西南交通大学为例[j].西南交通大学学报(社会科学版),2016(01).

[7]彼得·什托姆普卡.信任——一种社会学理论[m].北京:中华书局,2005.

[8]rahnwm,brehmj,alelectionsasinstitutionsforgeneratingsocialcapital[j].civicengagementinamericandemocracy,1999:111-160.

[9]capital,economicgrowthandqualityofgovernment:thecausalmechanism[j].newpoliticaleconomy,2003,8(1):49-71.

[10]胡荣,胡康,温莹莹.社会资本、政府绩效与城市居民对政府的信任[j].社会学研究,2011(01).

[11]高勇.参与行为与政府信任的关系模式研究[j].社会学研究,2014(05).

[12]刘建平,周云.政府信任的概念、影响因素、变化机制与作用[j].广东社会科学,2017(06).

[13]卢海阳,郑逸芳,黄靖洋.公共政策满意度与中央政府信任——基于中国16个城市的实证分析[j].中国行政管理,2016(08).

[14]谢星全.“央强地弱”政治信任结构与群体性事件[d].西南交通大学,2015.

[15]朱春奎,毛万磊.政府信任的概念测量、影响因素与提升策略[j].厦门大学学报(哲学社会科学版),2017(03).

[16]罗龙真.政府信任与社会信任关联性[d].暨南大学,2014.

[17]刘明明.社会信任对公众主观幸福感的影响研究[j].学习与实践,2016(01).

摘要:政府信任反映了居民对政府的认可程度,是政府政策执行的基础。汶川的地震灾区居民政府信任水平没有随着灾区公共服务水平的提高得到相应的提升,通过运用“汶川地震十周年居民生活情况调查”数据,实证分析探究了人际信任、政府绩效与政府信任之间的关系,研究结果显示,政府绩效、人际信任均对政府信任产生显著的影响。

关键词:灾区居民;政府信任;人际信任;政府绩效。

1问题的提出。

2008年汶川8.0级特大地震给震区带来了巨大的财产和生命安全损失,我国政府在救灾所表现出的强大动员能力、应急能力也成为世界关注的焦点。尤其通过政府的各项震后恢复政策,使得汶川灾区的社会经济发展获得历史性机遇,基础公共社会高水平重构,产业发展能力也持续提升,百姓生活水平相对于震前有较大幅度提高。

较高的政府信任度会有效降低政府运行的成本,于政府及公民都是有利的。而政府信任会受一系列因素的影响,在突发事件发生的关键时期政府政策执行对政府信任的影响更为显著。政府信任作为公民对政府的主观评价,受诸多因素的影响。现有的研究中不乏对政府影响因素的探讨,程倩认为,政府信任是政府合法性的本质内涵,政府信任关系是公民对政府及其政策执行过程中各相关要素、政策执行过程以及政策执行结果的心理预期。在二者的关系解释上,政府高校的运作过程以及较好的绩效水平可以带来良好的政府信任水平,两者呈正相关关系(程倩,2011)。缪婷婷认为,影响政府信任的因素具体包括政府作为、政府绩效以及新闻媒体的兴起(缪婷婷,2016)。在关键时期,政府政策对居民的政府信任影响更为显著。在突发事件中,公民对政府的信任感往往受领导者的价值观以及政府政策执行情况的影响,政府信任与政府及其工作人员对受灾民众的真诚关怀和激励呈显著正相关的影响(王璐,2008)。

因此,本文在梳理了现有研究成果的基础上,基于“汶川地震灾区灾后重建十周年状况调查”数据,对影响灾区居民政府信任度的影响因素进行实证分析。

2文献综述及研究假设。

2.1人际信任。

在文化主义视角下,人际信任会对政府信任产生显著的影响。高巍通过实证研究证实了人际信任对政府信任的影响:民众越容易相信他人,则更容易信任政府(高巍,2015)。胡荣等学者研究发现个人的信任水平对政府信任度会产生显著影响(胡荣,2007)。陈天祥构建了包括村内信任和社会信任的影响因素模型,验证了民众个人的信任水平对政府信任会产生显著的影响(陈天祥,2017)。人际信任程度越高,越有利于民众与政府的良性互动(刘人宁,2018)。人际信任变量作为文化路径中重要的解释变量,对居民政府信任的影响不言而喻,因此本文将其作为自变量的指标之一。据此,提出假设1:

假设1:人际信任度越高,政府信任度越高;人际信任度越低,政府信任度越低。

2.2政府绩效。

制度主义路径认为政府绩效很大程度上影响人们对政府信任度的判定,政府绩效涵盖政治、经济、环境保护等多方面的绩效。许伟通过研究发现,政府绩效变量对政府信任会产生显著的影响,同时验证了政府绩效与政府信任之间的正相关关系。杨培鸿(2007)通过实证研究证明了公共服务绩效作为关键性的影响因素,对公民政府信任评价起着重要的作用。陈天祥在研究中国乡镇政府的政府信任情况时也将政府绩效作为自变量进行分析(陈天祥等,2017)。由此,提出假设2:

假设2:政府绩效水平与政府信任呈正相关关系。

3研究方法。

3.1数据来源。

本研究使用数据来源于国家科技部中国发展战略研究院联同西南交通大学开展的“汶川十周年居民生活情况调查”数据,调查范围包括四川省5市/州30县/区共198个行政村/社区,共抽取了30個受灾县中的4950户住户,共完成家户问卷3826份,完成个人问卷3751份。

3.2变量的操作化。

因变量:政府信任变量。

在“汶川地震十周年居民生活状况调查”问卷中,关于政府信任被划分为对中央政府、省政府、县/市政府、乡/镇/街道四个层级政府的信任,选项分别为“完全信任”、“比较信任”、“不太信任”、“根本不信任”,对四个信任度赋值为1-4,1代表“完全信任”,4代表“根本不信任”,以此类推。通过探索性因子分析提取出“政府信任”因子。

自变量一:人际信任。

“汶川地震十周年居民生活状况调查”问卷中,对人际信任变量操作为题目——对“家人”、“住在你周围的人”、“市场上的商人/买卖人”、“外地人”、“村干部”、“警察”、“医生”、“法官”共8种不同类型群体信任情况,答案与测量“政府信任”的答案一致。通过探索性因子分析,根据指标特征,提取出三个因子,分别为“权威信任”、“社会信任”、“邻里信任”。

自变量二:政府绩效。

本研究中关于政府绩效感知被测量为居民对“贫困问题”、“贫富差距”、“失业情况”、“空气污染”、“水污染”、“缺水问题”、“孩子上学不方便”的严重程度评价,答案包括“特别严重”、“比较严重”、“不太严重”、“不成问题”四个程度的测量,从“特别严重”至“不成问题”分别赋值为1-4。通过因子分析,提取3个公因子为“人民生活”、“环境保护”、“公共服务”。

4研究分析。

4.1相关性分析。

通过相关性分析可知,居民人际信任的三个维度与政府信任的相关显著性均为0.000,小于0.05,均呈显著性相关关系,表明民众个人信任水平会显著影响民众对政府的信任,其中权威人群信任维度的得分系数最高,为0.665。

该部分基于因子分析和相关性分析的基础上,采取线性回归方法进一步验证各影响因素与政府信任的关系,建立了2个回归模型,对影响灾区居民政府信任的影响因素进行实证分析。

模型1中检验的是人际信任与政府信任之间的关系。在研究人际信任与政府信任的回归中国,采用逐步回归法,将人际信任的3个公因子作为自变量,政府信任因子作为因变量进行分析。根据选取自变量的规则p0.05,共选择了3个变量,包括权威信任、邻里信任以及社会信任。模型1的相关系数为0.678,调整后的r方为0.459。因此,最终建立的模型1包含权威信任,邻里信任,社会信任3个影响因素。通过anova表考察模型是否有效,该模型回归的显著性水平均显示p0.05,表明回归程度显著,可以认为该模型有效。从回归系数表可知,权威信任、邻里信任以及社会信任的回归系数分别为0.616、0.161、0.085,均大于0,说明权威信任、邻里信任以及社会信任程度越高,居民对政府信任的程度越高。t检验中,权威信任、邻里信任、社会信任和常数项的显著性p值都小于0.05,因而均有显著意义。

模型2检验政府绩效与政府信任之间的关系。在研究政府绩效与政府信任的回归中,采用逐步回归法,将绩效感知的3个因子作为自变量,政府信任因子作为因变量进行分析。根据选取自变量的规则p0.05,共选择了3个变量,包括制环境保护、公共服务以及人民生活三个维度的变量,建立模型2。根据模型2可知,环境保护、公共服务以及人民生活的回归系数分别为-0.179、-0.118、-0.109,絕对值均大于0,回归系数为负,说明环境保护、公共服务以人民生活各方面问题的严重程度越高,居民对政府信任的程度越低,即政府绩效水平与政府信任呈正相关关系,政府绩效越好,居民政府信任度越高。t检验中,环境保护、公共服务、人民生活和常数项的显著性p值都小于0.05,因而均有显著意义。

5研究结论。

本章首先在因子分析以及验证各影响因素与因变量存在相关性的基础上进行线性回归分析,进而验证了相关假设。具体结论如下:

(1)假设1得到验证:人际信任度越高,政府信任度越高;人际信任度越低,政府信任度越低。通过分析发现,人际信任对政府信任存在正相关关系。人际信任包括权威信任、社会信任及邻里信任。其中对权威群体的信任是本研究中影响政府信任最显著的因素,人际信任对居民的政府信任产生影响最为显著,其次是邻里信任,社会信任对政府信任的影响程度最低。

人际信任一直都被当作影响政府信任的重要因素,本研究通过实证分析也得出相同的结论。如何通过提高居民的人际信任水平从而提高政府信任水平是当前应该思考的问题。从本文的研究中可知,人际信任中居民对警察、医生、法官、村干部这类具有一定社会公信力的群体有着较高的信任水平,这与我国传统的信任特征有关,人们对这部分人群持有敬畏之心。但是,我们也可以看到在当今社会中,这部分过去极具公信力的人群也面临公信力下降的困境。因此,需提高这部分人群的服务能力,提高公共服务水平,更好地为人民服务。人际信任中得分最低的是社会信任,即对陌生人的信任水平的解释。在人际关系越来越“冷漠”的今天,社会信任显得尤为重要。近年出现的“扶不扶”、“小悦悦”等问题和事件极大的打击了人们的信任感,使得人们对外界保持极高的警惕心,人们越发地变得更为“冷漠”。因此,大力弘扬社会主义核心价值观,严厉打击破坏社会和谐稳定的因素。

(2)假设2得到验证:政府绩效越好,公民对政府的信任越强。政府绩效越差,公民对政府的信任越低。通过回归分析发现,居民对政府绩效的感知对政府信任产生显著的影响。本研究从政府在公共服务、环境保护、人民生活三方面的绩效成果来考察政府绩效对政府信任的影响,三个维度中环境保护绩效对政府信任的影响最为显著。

政府绩效是政府行为的直观体现,政府绩效的好坏直接影响民众对政府的信任水平,提高政府工作绩效是我国政府一直努力的方向。本研究关于灾区政府绩效与政府信任的关系中,人民生活、公共服务、环境保护方面的绩效对政府信任产生显著的影响。因此,提高灾区居民政府信任需要提高政府绩效,转变政府职能,提高政府的公共服务能力和水平。

参考文献。

[2]缪婷婷.政府绩效对政府信任的影响[d].苏州:苏州大学,2016.

[4]高巍.当前中国居民政府信任的区域比较[d].济南:山东大学,2015.

[7]刘人宁.基层政府信任的影响因素研究[d].哈尔滨:黑龙江省社会科学院,2018.

[8]许伟.我国当代政府信任的比较研究[d].武汉:武汉大学,2014.

摘要:自从分税制模式实施后,出现了中央与地方政府财权和事权不匹配的问题。2008年金融危机对我国的经济影响非常大,于是我国扩大内需,推出了4万亿元的投资计划。很多地方政府融资平台开始迅速扩张,扩大自己的融资规模。然而这种现象也产生了负面影响,各地政府实施不同的项目,导致政府债务规模不断扩大,政府债务风险严重。鉴于此,对政府财务风险进行分析,并提出解决政府财务风险的对策。

关键词:政府债务;财务风险;防范对策。

我国从1994年开始进行分税制改革,导致政府的财权和事权不统一,政府支出的责任变更非常频繁,但是没有相应的财力作为支撑。2008年,我国为了应对金融危机,提出了4万亿政策,结合中央政府的投资计划,地方政府开展了各种形式的融资。大规模的融资导致债务隐性化,债务风险非常严峻。在党的十九大报告中,明确提出要防范各类风险。在风险防控中,政府的债务风险防控是重点,从而有效地预防金融风险的发生。

1.政府财权事权与支出责任不符。自从1994年我国实施分税制改革后,税种划分成中央税种和地方税种。地方的事权不断增多,导致地方性政府财政支出越来越多。随着城镇化进程的加快,居民对公共产品的需求量非常大,政府需要更多的资金。地方政府为了进行基础设施建设,他们每年都要融资,产生债务风险。地方政府的财政收入非常有限,受到中央强化房地产调控的影响,在转移支付环节中存在各类问题,不能结合地方政府的实际需求。在增量调节环节,地方之间的贫富差距非常大。地方政府的事权不断增加,但是财权非常有限。地方政府体制内的收入不能弥补财政支出,导致地方政府的预算外收入增加,他们只能通过举债的方式弥补财政不足。

2.政府融资不够规范。政府在融资环节中,产生非常大的随意性。地方政府常常通过发行债券的方式,但是资金还是不能满足需求。于是,地方政府开始建立各类融资平台,在融资环节中产生盲目性和随意性。政府的融资运作效率不高,融资的成本非常高,导致政府投资效率不佳。在融资平台建设中,资金不足。政府会通过各类方式补充资金,通过挪用年度预算等方式,甚至出现虚假注资的方式。政府在融資方面,相关的管理体制存在缺陷,很多融资平台的建设没有相关的监督措施加以辅助,采用多头管理的方式。融资监督机制尚未建立,政府在融资环节中,缺乏法律和公众的监督,导致商业腐败的产生。

3.政府债务管理机制不够健全。政府的债务管理机构不完善,尽管我国政府已经积累了很多政府性债务,但是债务管理机构尚不完善。债务资金使用不够规范,有些地方政府的债务资金不能纳入到预算管理环节,债务资金不能充分进行基础设施建设。债务资金管理效果不好,不能进行全方位的预算管理。政府债务领域的信息不够透明,地方政府的融资情况不能及时地回报给人大常委,信息不公开,导致地方性债务的口径不能完全的纳入到预算管理系统中。债务管理的法制建设比较落后,不能结合债务管理建立法律体系。尽管我国制定了《预算法》等,对政府的举债行为加以约束,但随着社会经济的发展,政府投资还是越来越多,已经超过政府的财力。

1.提升思想认识,强化债务管理。政府应强化风险认识,防止债务风险扩散为经济风险,要正确地认识债务风险问题,纵观大局,做到未雨绸缪。政府应树立忧患意识,认识到抵御债务风险的紧迫性,也要认识到政府举债对当地经济发展产生的影响,确保偿债有序。强化底线意识,底线意识在一定程度上是底线思维的应用,其与风险意识是相辅相成的,政府要在对现实情况进行分析的基础上,提高警惕。将国家法律作为“红线”,做好风险的防范准备工作。政府还要提升自身的法治意识,杜绝违法违规的举债行为。有些政府的领导干部,他们的法治意识比较弱,对于一些惩戒视而不见。因此,政府应树立法治意识,规范举债行为,维护法律的权威。充分了解政府职能的权限,掌握政府和市场之间的关系,强化政府职能的转变,发挥市场这只无形手的资源配置作用。

2.推进体制机制改革。合理划分财权和事权,中央政府和地方政府应该明确财政事权和支出责任,有效地抑制政府债务规模。结合权、责和利相统一的原则,各地区的财权和事权应得到明确,制定财权事权调整机制,形成完整和清晰的财权和事权清单,从而提升政府对于财权和事权改革的可操作性。建立相应的配套措施,完善相关的改革协同配套方案,提升转移支付制度的可操作性。进行干部任用制度的改革,完善债务监督考核问责制度。对考核的方法进行改进,抓住基础和发展的根基,充分发挥潜能,改善民生,促进社会进步。完善政府债务考核和问责制度,建立公共服务为中心的理念。在新政策背景下,完善债务监督制度,对金融机构进行科学的监管工作,有效地杜绝违法违规操作,强化责任人制度。对政府债券资金进行全面管理,明确责任主体和财务主体。提升债务信息的透明度,通过政府政务网站的方式,对债务限额明确,建立债务限额报告制度。通过财政预算的方式,对预算情况进行分析。

3.提升政府债务风险管理能力。建立政府债务管理小组,发挥其职能。对政府债务的借、用和还阶段进行监督,进行债务管理工作的指导,对于债务风险要实时掌握。当风险发生后,应采取必要的干预措施。完善政府债务的预算管理工作,分别对政府债务收入、支出、付息和还本进行预算,防止超预算举债的发生。各地的人大应发挥审议作用,充分履行监督职能。提升风险化解的能力,在政府债务管理中,隐性债务管理存在一定的难度。因此,在债务管理中,要对各类风险进行评估,出具评估报告。有些政府债务风险防范能力比较差,风险防控机制比较落后。因此,要提升政府债务风险化解能力,抓住风险产生的主要因素。

政府债务风险是重大风险的类型之一,因此,我国应有效地防范和化解政府财务风险,促进全面小康社会的建设。同时,政府应提升债务风险的化解能力,树立危机意识。

参考文献:

[2]胡才龙,魏建国.多任务委托代理模型下地方政府债务管理激励契约设计——基于省级面板数据的实证检验[j].审计与经济研究,2019,(5):118-127.

摘要:以山东省950名青少年学生为调查对象,对青少年的政府信任影响因素进行比较分析与研究,结果发现,理性选择路径、社会文化路径、政治参与路径均对青少年的政府信任有显著影响。在此基础上,提出了改善青少年政府信任的有效策略。

关键词:青少年政府信任影响因素心理机制。

中图分类号:g641文献标识码:a。

1问题的提出。

政府信任,指民众对于政府或政治制度所抱有的信心,相信它们会制定符合民众利益的政策并提供其预期的政策结果[1]。青少年是未来社会主义建设的接班人和生力军,他们的政府信任直接决定了我国政治生态和民主建设的发展与未来。学界关于青少年政府信任现状的研究,得出的结果不尽相同。有些研究表明,当代中国青少年政治信任的总体状况良好[2-3],另外一些研究显示,当代青少年政治认同度不高且呈工见非均衡状态[4]。

加快新旧动能转换是山东省促进经济结构转型升级的重要途径。在新旧动能转换中,各级政府肩负着提供公共产品,支持科技创新,引导产业发展,健全社会保障等重要作用。在这样的背景下,研究山东省青少年的政府信任具有重要的现实意义。

目前关于青少年政府信任的影响因素,主体方面的影响因素主要有青少年的性别、年龄、家庭情况、人际信任、思政政治教育、大众传媒等;客体方面主要有政府的服务水平、公务员的素质、行政执行力等因素[5]。对政治信任的研究主要存在三种竞争性理论解释路径,即理性选择路径、社会文化路径、政治参与路径[6]。青少年政府信任影响因素及心理机制的分析模型,见图1。

什么因素影响山东省青少年的政府信任?三种解释理论预测作用如何?如何提升青少年政府信任?对此,本文进行有益的探索。

2研究方法。

2.1研究对象。

本研究采用随机抽样的方法,对山东省4所中学和4所大学的950名青少年进行调查研究。其中,男生386人(40.6%),女生564人(59.4%);高一学生147人(15.5%),高二学生140人(14.7%),高三学生118人(12.4%),大一学生166人(17.5%),大二学生146人(15.4%),大三学生120人(12.6%),大四学生112人(11.8%);生源地为农村的571人(60.1%),城镇的为379人(39.9%);政治面貌为中共党员(含预备党员)的50人(5.3%),非党员900人(94.7%);学生干部97人(10.2%),非学生干部853人(89.8%)。

2.2研究工具。

通过对国内外大量文献梳理,建构了包含五个因素的大学生政府信任初测问卷,分别为基本信息、政府信任(中央政府、省级政府、市级政府、县区政府、乡镇街道政府)、理性文化路径(制度信任、过程信任、人员信任、结果信任、生活满意度)、社会文化路径(人际信任)、政治参与路径(制度性参与、非制度行参与)。对初测问卷数据实施因子分析和信度分析,最终确定正式问卷包括以上4个维度,共19题。问卷中每个维度的信度检验结果,表明青少年政府信任影响因素问卷是稳定可信的。题项信度分析,见表1。

3调查结果。

3.1青少年政府信任的总体状况。

数据结果显示,在青少年政府信任的四维结构中,受访学生各维度的题项均分在3.591-3.704,处于中等偏上水平。青少年政府信任的总量表得分为3.621,也处于中等偏上水平[7]。青少年政府信任的总体状况,见表2.

青少年在“结果信任”维度上的得分最低,在“制度信任”维度上的得分最高,各维度平均值的大小顺序为:制度信任过程信任人员信任结果信任。

3.2青少年政府信任影响因素的多元回归。

为了分析和比较理性选择路径、社会文化路径以及政治参与路径对于青少年政府信任的影响,构建了四个多元统计分析模型,具体结果见表2。在控制变量方面,模型一到三的结果显示,性别、年级、政治面貌、生源地均对青少年政府信任没有统计上的影响,学生干部这一个变量却具有显著差异。三种理论解释路径对青少年政府信任的影响,见表3。

模型1关注理性选择路径对于因变量的影响。结果显示,除制度信任外,过程信任、人员信任、结果信任、生活满意度均与政府信任具有显著的关系,由此可以推断理性选择路径对于青少年的政府信任具有强大的预测作用。

模型2关注社会文化路径对于因变量的影响。结果显示,人际信任与青少年政府信任具有显著的关系。

模型3关注政治參与路径对于因变量的影响。结果显示,政治参与与青少年政府信任具有显著的关系。

模型4采用逐步回归的方式,保留显著影响青少年政府信任的因子。从表中可以看出,理性选择路径对青少年政府信任的解释力最强(0.282+0.137+0.162=0.581),其次为社会文化路径(0.149),最后为政治参与路径(0.134)。由此可见三种解释路径都对青少年政府信任具有一定的预测力。

4结论与建议。

从表2可以看出,理性选择路径对青少年政府信任的解释力最强,这与现在文献研究结果较为一致。因此,在中国特色社会主义进入新时代的社会背景下,各级政府仍需加大教育投入,加强校园软硬件建设,热爱学生、关注学生、善待学生,不断提高办学水平,让青少年切切实实体会到国家经济和社会发展的成果,提高广大青少年的学校满意度。

社会文化路径(人际信任)对青少年的政府信任有重要的塑造作用。当前青少年学生的人际信任不容乐观,研究发现我国大学生人际信任水平与年代之间呈显著的负相关,说明大学生人际信任正逐年降低[8]。从学校角度来讲,应该通过多种形式加强青少年的“三观”培养和教育,增加社会责任感,提升人格,完善道德,不断提高对学校、教师和同学的信任水平。

政治参与路径对青少年的政府信任有重要影响。学校要通过思想政治教育理论课、选修课、专题讲座、主题班会等形式,加强大学生政治主体观教育,使大学生充分认识其政治权利和义务,增强大学生的政治效能感,提高大学生的政府信任度。高校应该从学生全面发展的高度出发,开设政治实践活动和政治技能训练。积极推行学生自我管理、自我服务、自我教育,鼓励学生参与学校管理,提出建设性的意见和建议;积极与当地政府部门和团群组织协作,开展丰富多彩的政治实践活动,丰富大学生的政治参与经验。

参考文献:

[2]冯岩.当代中国大学生政治信任研究[d].济南大学,2015.

摘要:政府信任反映了居民对政府的认可程度,是政府政策执行的基础。汶川的地震灾区居民政府信任水平没有随着灾区公共服务水平的提高得到相应的提升,通过运用“汶川地震十周年居民生活情况调查”数据,实证分析探究了人际信任、政府绩效与政府信任之间的关系,研究结果显示,政府绩效、人际信任均对政府信任产生显著的影响。

关键词:灾区居民;政府信任;人际信任;政府绩效。

2008年汶川8.0级特大地震给震区带来了巨大的财产和生命安全损失,我国政府在救灾所表现出的强大动员能力、应急能力也成为世界关注的焦点。尤其通过政府的各项震后恢复政策,使得汶川灾区的社会经济发展获得历史性机遇,基础公共社会高水平重构,产业发展能力也持续提升,百姓生活水平相对于震前有较大幅度提高。

较高的政府信任度会有效降低政府运行的成本,于政府及公民都是有利的。而政府信任会受一系列因素的影响,在突发事件发生的关键时期政府政策执行对政府信任的影响更为显著。政府信任作为公民对政府的主观评价,受诸多因素的影响。现有的研究中不乏对政府影响因素的探讨,程倩认为,政府信任是政府合法性的本质内涵,政府信任关系是公民对政府及其政策执行过程中各相关要素、政策执行过程以及政策执行结果的心理预期。在二者的关系解释上,政府高校的运作过程以及较好的绩效水平可以带来良好的政府信任水平,两者呈正相关关系(程倩,2011)。缪婷婷认为,影响政府信任的因素具体包括政府作为、政府绩效以及新闻媒体的兴起(缪婷婷,2016)。在关键时期,政府政策对居民的政府信任影响更为显著。在突发事件中,公民对政府的信任感往往受领导者的价值观以及政府政策执行情况的影响,政府信任与政府及其工作人员对受灾民众的真诚关怀和激励呈显著正相关的影响(王璐,2008)。

因此,本文在梳理了现有研究成果的基础上,基于“汶川地震灾区灾后重建十周年状况调查”数据,对影响灾区居民政府信任度的影响因素进行实证分析。

2.1人际信任。

在文化主义视角下,人际信任会对政府信任产生显著的影响。高巍通过实证研究证实了人际信任对政府信任的影响:民众越容易相信他人,则更容易信任政府(高巍,2015)。胡荣等学者研究发现个人的信任水平对政府信任度会产生显著影响(胡荣,2007)。陈天祥构建了包括村内信任和社会信任的影响因素模型,验证了民众个人的信任水平对政府信任会产生显著的影响(陈天祥,2017)。人际信任程度越高,越有利于民众与政府的良性互动(刘人宁,2018)。人际信任变量作为文化路径中重要的解释变量,对居民政府信任的影响不言而喻,因此本文将其作为自变量的指标之一。据此,提出假设1:

假设1:人际信任度越高,政府信任度越高;人际信任度越低,政府信任度越低。

2.2政府绩效。

制度主义路径认为政府绩效很大程度上影响人们对政府信任度的判定,政府绩效涵盖政治、经济、环境保护等多方面的绩效。许伟通过研究发现,政府绩效变量对政府信任会产生显著的影响,同时验证了政府绩效与政府信任之间的正相关关系。杨培鸿(2007)通过实证研究证明了公共服务绩效作为关键性的影响因素,对公民政府信任评价起着重要的作用。陈天祥在研究中国乡镇政府的政府信任情况时也将政府绩效作为自变量进行分析(陈天祥等,2017)。由此,提出假设2:

假设2:政府绩效水平与政府信任呈正相关关系。

3.1数据来源。

本研究使用数据来源于国家科技部中国发展战略研究院联同西南交通大学开展的“汶川十周年居民生活情况调查”数据,调查范围包括四川省5市/州30县/区共198个行政村/社区,共抽取了30個受灾县中的4950户住户,共完成家户问卷3826份,完成个人问卷3751份。

3.2变量的操作化。

因变量:政府信任变量。

在“汶川地震十周年居民生活状况调查”问卷中,关于政府信任被划分为对中央政府、省政府、县/市政府、乡/镇/街道四个层级政府的信任,选项分别为“完全信任”、“比较信任”、“不太信任”、“根本不信任”,对四个信任度赋值为1-4,1代表“完全信任”,4代表“根本不信任”,以此类推。通过探索性因子分析提取出“政府信任”因子。

自变量一:人际信任。

“汶川地震十周年居民生活状况调查”问卷中,对人际信任变量操作为题目——对“家人”、“住在你周围的人”、“市场上的商人/买卖人”、“外地人”、“村干部”、“警察”、“医生”、“法官”共8种不同类型群体信任情况,答案与测量“政府信任”的答案一致。通过探索性因子分析,根据指标特征,提取出三个因子,分别为“权威信任”、“社会信任”、“邻里信任”。

自变量二:政府绩效。

本研究中关于政府绩效感知被测量为居民对“贫困问题”、“贫富差距”、“失业情况”、“空气污染”、“水污染”、“缺水问题”、“孩子上学不方便”的严重程度评价,答案包括“特别严重”、“比较严重”、“不太严重”、“不成问题”四个程度的测量,从“特别严重”至“不成问题”分别赋值为1-4。通过因子分析,提取3个公因子为“人民生活”、“环境保护”、“公共服务”。

4.1相关性分析。

通过相关性分析可知,居民人际信任的三个维度与政府信任的相关显著性均为0.000,小于0.05,均呈显著性相关关系,表明民众个人信任水平会显著影响民众对政府的信任,其中权威人群信任维度的得分系数最高,为0.665。

该部分基于因子分析和相关性分析的基础上,采取线性回归方法进一步验证各影响因素与政府信任的关系,建立了2个回归模型,对影响灾区居民政府信任的影响因素进行实证分析。

模型1中检验的是人际信任与政府信任之间的关系。在研究人际信任与政府信任的回归中国,采用逐步回归法,将人际信任的3个公因子作为自变量,政府信任因子作为因变量进行分析。根据选取自变量的规则p0.05,共选择了3个变量,包括权威信任、邻里信任以及社会信任。模型1的相关系数为0.678,调整后的r方为0.459。因此,最终建立的模型1包含权威信任,邻里信任,社会信任3个影响因素。通过anova表考察模型是否有效,该模型回归的显著性水平均显示p0.05,表明回归程度显著,可以认为该模型有效。从回归系数表可知,权威信任、邻里信任以及社会信任的回归系数分别为0.616、0.161、0.085,均大于0,说明权威信任、邻里信任以及社会信任程度越高,居民对政府信任的程度越高。t检验中,权威信任、邻里信任、社会信任和常数项的显著性p值都小于0.05,因而均有显著意义。

模型2检验政府绩效与政府信任之间的关系。在研究政府绩效与政府信任的回归中,采用逐步回归法,将绩效感知的3个因子作为自变量,政府信任因子作为因变量进行分析。根据选取自变量的规则p0.05,共选择了3个变量,包括制环境保护、公共服务以及人民生活三个维度的变量,建立模型2。根据模型2可知,环境保护、公共服务以及人民生活的回归系数分别为-0.179、-0.118、-0.109,絕对值均大于0,回归系数为负,说明环境保护、公共服务以人民生活各方面问题的严重程度越高,居民对政府信任的程度越低,即政府绩效水平与政府信任呈正相关关系,政府绩效越好,居民政府信任度越高。t检验中,环境保护、公共服务、人民生活和常数项的显著性p值都小于0.05,因而均有显著意义。

本章首先在因子分析以及验证各影响因素与因变量存在相关性的基础上进行线性回归分析,进而验证了相关假设。具体结论如下:

以“政府绩效与公众信任”为撰写小论文范文

摘要:近年来,随着中国经济的持续发展和我国政府审计工作的改善,业绩审计在政府监督中发挥着越来越重要的作用。本文阐述了政府绩效审计的现状,深入分析了中国政府发展绩效审计的必要性,揭示了当前政府绩效审计存在的问题及其原因,最后从审计观念、审计监督制度、审计法律环境和预算制度等方面提出了优化政府绩效审计途径的建议,为中国政府绩效审计的高效合理运行和持续发展提供了一定的参考。

关键词:政府业绩审计审计监督优化路径公共管理。

近年来,随着我国经济持续发展和制度的不断完善,人民群众对政府公共财政支出的效率和效果关注度也日益提高,为了加强政府责任,适应我国建设社会主义法治国家的客观要求,产生了政府业绩评价。经过数十年的发展,政府审计在理论和实践层面进入了新阶段,现在以业绩审计为政府审计的重要内容,成为政府审计的新发展趋势。目前,我国政府绩效审计尚不成熟,仍处于起步阶段,存在诸多问题,如何构建适合我国国情的绩效审计体系成为我国政府审计的关键。

(一)是建设廉洁透明服务型政府的需要。

党的十九大报告明确了积极安全推进政治体制改革,建设高效透明的服务型政府。如今,我国正处在经济高速发展阶段,政府公共财政支出成为社会焦点问题。政府绩效考核的主要内容是对政府官员在任期间的工作情况和成果进行绩效考核和财政资金运用的有效性和经济性考核,政府绩效考核的开展拓宽了考核监督的领域和范围,加强了考核机构在行政监督中的作用,增强了各级政府工作人员的责任感,减少了腐败事件的发生,进一步避免了大量经济损失的发生,为建设高效透明的服务型政府营造了良好的氛围。

(2)是提高公共管理效率的需要。

近年来,政府财政收入大幅增加,公共管理投入增加,但公共建设相关项目从立项到最终完成需要多部门多层次审查,项目实施时失去最佳实施时机,同时浪费大量人力物力,这些现象的出现强调政府机构工作管理效率低,人浮于事件。审计机构在对政府部门及其下属单位进行绩效审计过程中,对公共资源配置的经济效率、配置效果等方面进行审查,有效管理和评价政府权力的使用,提高政府行政服务的工作效率水平。

(3)是保证政府职能发挥作用的需要。

近年来,中国财政支出呈现稳定增长态势。但由于目前中国政府财政支出结构不完善,财政支出表现相对较低。从近年来的开支来看,经济建设和行政管理所用的地方财政金额差异较大,人员资金开支已成为主要部分。该支出严重限制了政府职能的正常履行,因此需要加强对政府财政资金的监管和分配。根据政府财政资源有限的基本国情,及时对财政监督财政开支,能够有效控制各级政府部门公共资金开支,提升公共资金业绩。

(1)审查范围逐渐扩大。

21世纪初,政府有关机构及其下属单位的业绩审计提上日程。审计机构审计重点从以前被审计机构财务收支的客观性和合法性转变为被审计机构财务资源使用的有效性和经济性。与此同时,2016年1-11月,国家审计机构完成了21334业绩审计项目。其中,审计局完成113个,省级审计机构完成847个,地级审计机构完成6288个,县级审计机构完成14086个。另外,各省市地方审计机构实施的绩效审计项目数量占审计项目总数的31%。其中,18个省(自治区、直辖市)和5个独立计划城市均超过这一平均水平。2017年1-11月,绩效考核工作量约占考核总工作量的一半左右,比2016年同期增长10%。

(二)加强政策支持力度。

近年来,审计局发布了多份文件,规划了业绩审计的发展方向,十三五国家审计工作的发展规划提出进一步加大业绩审计力度,始终贯穿审计工作,促进发展质量和利益的重大违纪违法问题的暴露和审查力度,促进廉政建设。与此同时,各省市也积极制定政府业绩审计相关政策,2011年徐州市审计部门发布《徐州市审计机关政府投资项目业绩审计操作指南》,规范徐州市业绩审计工作,2012年浦东新区人民政府发布《关于印发浦东新区业绩审计方法的通知》,对浦东新区政府部门业绩审计提出了实施要求。

(一)审计机构缺乏独立性。

从我国政府行政结构来看,我国各地市审计机构属于各地政府,受当地政府和上级审计机构的指导,受经费和行政管理模式的影响,业绩审计目标和具体任务的制定受当地政府的制约,阻碍审计作用确实有效地发挥,同时审计机构业绩审计活动完成后,审计机构发行的审计报告由同级地方政府和上级审计机构审计,不利于审计工作报告的完整性和客观性宝座,审计报告的真实性受到制约。另外,审计机构的组成人员审计机构本身只有提案的权利,其人员编制、人事选拔、任免、奖惩均由同级组织部和同级政府审查批准,在一定程度上阻碍了审计机构职能的正常发挥。

*河北省哲学社会科学计划办公室:ppp模式支持正确扶贫绩效评价研究(hb18gl058)。

(二)缺乏相关法律法规的指导。

我国政府业绩审计起步晚,审计工作的具体方法和程序尚不成熟,目前只有少量法律法规提到政府业绩审计,政府业绩审计工作开展的完整法律依据不足,现行《审计法实施条例》第二条指出,《审计机关依法独立检查被审计机关的会计证明书、会计账簿、财务会计报告书及其他与财政收支、财务收支相关的资料和资产,监督财政收支、财务收支的真实性、合法性和效益行为》,相关法律只进一步明确了该法律规定,但没有列出具体的审计内容和我国政府的具体规定。在这种背景下,审计相关人员只能通过现有的知识储存和现有的经验来探索,审计人员的工作水平也不同,业绩审计效果的正确客观性增加了一定的风险。(3)审计团队人才结构不合理。

完成政府业绩审计需要多个专业审计师团队。在此期间,中国审计机构主要从事传统财务审计。审计师的知识结构比较简单,工程技术、法律、宏观经济、对外贸易和计算机专家数量比较少。根据中国审计师知识结构的调查,会计和审计专家占所有审计师的71.8%。绩效考核要求考核广泛的经济和管理活动的有效性,要求多样化、创新的方法和多学科知识。有些审计师很难满足高标准的绩效审计。另外,审计员整体的数量也很少,难以适应大的任务量,水平越低的机构越明显。

(一)完善独立监督体制。

一是要进一步加强垂直管理改革力度。不仅将地方审计机关正职的任免权接受省级机关,普通审计人员也由省审计厅管理,招聘、提拔、任免由地方审计机关提名,由省审计厅和省人社会局决定,二是地方审计机关经费由省财政厅统一保证,纳入省财政预算。由此可以将人力资源和财务完全独立于地方政府和财政部门,提高独立性,使地方审计部门更有效地对地方政府领导进行审计监督,更有力地揭示和反映问题。

(二)健全相关法律制度。

由于缺乏明确的法律条款,我国政府业绩审计推广和拓展面临一定的困难。我国立法机关应进一步完善我国政府业绩审计相关法律制度。首先要完善审计报告的公开制度,通过将审计活动的各个阶段以审计报告的形式向公众展示。在完善审计报告制度的过程中,可以充分利用媒体作为中介的作用,及时准确地将审计结果传递给社会公众,扩大政府审计的影响力。其次,要不断健全审计结果问责制度,加强审计结果的应用,实现审计报告的公开与审计结果问责的有效联系。

(3)构建合理的评价标准体系。

随着政府职能的逐步完善,经济性和利益性也是政府业绩审计应该关注的重要内容。在预算绩效考核体系的基础上,审计机构应积极探索绩效考核考核标准,建立全国公认、适用、规范的体系,不断修改、完善。业绩评价体系应当包括预期的目标、决策过程、投入资源和能源、生产效益、对社会效益的影响等,评价指标应当坚持定量和定性的结合,定量指标主要指财务指标,定性指标的确定应以被审计项目的特点和被审计部门的管理特点为基础。

(四)改善政府业绩审计人员结构。

审计团队人员的综合素质和业务能力是政府业绩审计顺利进行的重要保障。审计机构不仅要提高现有审计人员的综合素质能力,还要不断引进高水平的审计人才,尤其是专业型的审计人员。要提高审计机构的准入门槛,完善审计人员的准入制度,确保招聘人员在综合素质和业务水平上能够满足政府业绩审计的要求,确保审计队伍中的人才能够胜任其职责。此外,审计机构人员不仅要包括专业审计人员,还要在实际绩效审计工作中招聘工科类、经济管理类等其他专业人员,不断优化审计人员结构。

全面优化我国政府业绩审计可有效促进我国审计持续健康发展,有利于推进社会主义法治社会建设,但结合政府业绩审计在我国起步晚、发展缓慢的现状,如何更好地开发和引导还需要进一步深入研究和探讨。

参考文献。

[1]齐兴利,绍辉.中国政府业绩审计发展途径研究[j].审计和经济研究,2007.2。

[2]周亚荣.中国政府业绩审计理论研究与实践现状[j].审计与经济研究,2008.2。

[3]宋夏云,我国政府业绩审计师能力框架研究[j].会计研究,2013.4。

[4]曲明,中国政府绩效审计评价标准体系框架构建[j].财经问题研究,2016.5。

[5]闵晓蕾.政府业绩审计理论文献综述[j].财经论坛,2006.2。

以“政府绩效与公众信任”为撰写小论文信任为话题范文

政府是指国家进行统治和社会管理的机关,是国家表示意志、发布命令和处理事务的机关,实际上是国家代理组织和官吏的总称。以下是为大家整理的关于,欢迎大家前来参考查阅!

摘要:自从分税制模式实施后,出现了中央与地方政府财权和事权不匹配的问题。2008年金融危机对我国的经济影响非常大,于是我国扩大内需,推出了4万亿元的投资计划。很多地方政府融资平台开始迅速扩张,扩大自己的融资规模。然而这种现象也产生了负面影响,各地政府实施不同的项目,导致政府债务规模不断扩大,政府债务风险严重。鉴于此,对政府财务风险进行分析,并提出解决政府财务风险的对策。

关键词:政府债务;财务风险;防范对策。

我国从1994年开始进行分税制改革,导致政府的财权和事权不统一,政府支出的责任变更非常频繁,但是没有相应的财力作为支撑。2008年,我国为了应对金融危机,提出了4万亿政策,结合中央政府的投资计划,地方政府开展了各种形式的融资。大规模的融资导致债务隐性化,债务风险非常严峻。在党的十九大报告中,明确提出要防范各类风险。在风险防控中,政府的债务风险防控是重点,从而有效地预防金融风险的发生。

1.政府财权事权与支出责任不符。自从1994年我国实施分税制改革后,税种划分成中央税种和地方税种。地方的事权不断增多,导致地方性政府财政支出越来越多。随着城镇化进程的加快,居民对公共产品的需求量非常大,政府需要更多的资金。地方政府为了进行基础设施建设,他们每年都要融资,产生债务风险。地方政府的财政收入非常有限,受到中央强化房地产调控的影响,在转移支付环节中存在各类问题,不能结合地方政府的实际需求。在增量调节环节,地方之间的贫富差距非常大。地方政府的事权不断增加,但是财权非常有限。地方政府体制内的收入不能弥补财政支出,导致地方政府的预算外收入增加,他们只能通过举债的方式弥补财政不足。

2.政府融资不够规范。政府在融资环节中,产生非常大的随意性。地方政府常常通过发行债券的方式,但是资金还是不能满足需求。于是,地方政府开始建立各类融资平台,在融资环节中产生盲目性和随意性。政府的融资运作效率不高,融资的成本非常高,导致政府投资效率不佳。在融资平台建设中,资金不足。政府会通过各类方式补充资金,通过挪用年度预算等方式,甚至出现虚假注资的方式。政府在融資方面,相关的管理体制存在缺陷,很多融资平台的建设没有相关的监督措施加以辅助,采用多头管理的方式。融资监督机制尚未建立,政府在融资环节中,缺乏法律和公众的监督,导致商业腐败的产生。

3.政府债务管理机制不够健全。政府的债务管理机构不完善,尽管我国政府已经积累了很多政府性债务,但是债务管理机构尚不完善。债务资金使用不够规范,有些地方政府的债务资金不能纳入到预算管理环节,债务资金不能充分进行基础设施建设。债务资金管理效果不好,不能进行全方位的预算管理。政府债务领域的信息不够透明,地方政府的融资情况不能及时地回报给人大常委,信息不公开,导致地方性债务的口径不能完全的纳入到预算管理系统中。债务管理的法制建设比较落后,不能结合债务管理建立法律体系。尽管我国制定了《预算法》等,对政府的举债行为加以约束,但随着社会经济的发展,政府投资还是越来越多,已经超过政府的财力。

1.提升思想认识,强化债务管理。政府应强化风险认识,防止债务风险扩散为经济风险,要正确地认识债务风险问题,纵观大局,做到未雨绸缪。政府应树立忧患意识,认识到抵御债务风险的紧迫性,也要认识到政府举债对当地经济发展产生的影响,确保偿债有序。强化底线意识,底线意识在一定程度上是底线思维的应用,其与风险意识是相辅相成的,政府要在对现实情况进行分析的基础上,提高警惕。将国家法律作为“红线”,做好风险的防范准备工作。政府还要提升自身的法治意识,杜绝违法违规的举债行为。有些政府的领导干部,他们的法治意识比较弱,对于一些惩戒视而不见。因此,政府应树立法治意识,规范举债行为,维护法律的权威。充分了解政府职能的权限,掌握政府和市场之间的关系,强化政府职能的转变,发挥市场这只无形手的资源配置作用。

2.推进体制机制改革。合理划分财权和事权,中央政府和地方政府应该明确财政事权和支出责任,有效地抑制政府债务规模。结合权、责和利相统一的原则,各地区的财权和事权应得到明确,制定财权事权调整机制,形成完整和清晰的财权和事权清单,从而提升政府对于财权和事权改革的可操作性。建立相应的配套措施,完善相关的改革协同配套方案,提升转移支付制度的可操作性。进行干部任用制度的改革,完善债务监督考核问责制度。对考核的方法进行改进,抓住基础和发展的根基,充分发挥潜能,改善民生,促进社会进步。完善政府债务考核和问责制度,建立公共服务为中心的理念。在新政策背景下,完善债务监督制度,对金融机构进行科学的监管工作,有效地杜绝违法违规操作,强化责任人制度。对政府债券资金进行全面管理,明确责任主体和财务主体。提升债务信息的透明度,通过政府政务网站的方式,对债务限额明确,建立债务限额报告制度。通过财政预算的方式,对预算情况进行分析。

3.提升政府债务风险管理能力。建立政府债务管理小组,发挥其职能。对政府债务的借、用和还阶段进行监督,进行债务管理工作的指导,对于债务风险要实时掌握。当风险发生后,应采取必要的干预措施。完善政府债务的预算管理工作,分别对政府债务收入、支出、付息和还本进行预算,防止超预算举债的发生。各地的人大应发挥审议作用,充分履行监督职能。提升风险化解的能力,在政府债务管理中,隐性债务管理存在一定的难度。因此,在债务管理中,要对各类风险进行评估,出具评估报告。有些政府债务风险防范能力比较差,风险防控机制比较落后。因此,要提升政府债务风险化解能力,抓住风险产生的主要因素。

政府债务风险是重大风险的类型之一,因此,我国应有效地防范和化解政府财务风险,促进全面小康社会的建设。同时,政府应提升债务风险的化解能力,树立危机意识。

参考文献:

[2]胡才龙,魏建国.多任务委托代理模型下地方政府债务管理激励契约设计——基于省级面板数据的实证检验[j].审计与经济研究,2019,(5):118-127.

摘要:政府信任反映了居民对政府的认可程度,是政府政策执行的基础。汶川的地震灾区居民政府信任水平没有随着灾区公共服务水平的提高得到相应的提升,通过运用“汶川地震十周年居民生活情况调查”数据,实证分析探究了人际信任、政府绩效与政府信任之间的关系,研究结果显示,政府绩效、人际信任均对政府信任产生显著的影响。

关键词:灾区居民;政府信任;人际信任;政府绩效。

2008年汶川8.0级特大地震给震区带来了巨大的财产和生命安全损失,我国政府在救灾所表现出的强大动员能力、应急能力也成为世界关注的焦点。尤其通过政府的各项震后恢复政策,使得汶川灾区的社会经济发展获得历史性机遇,基础公共社会高水平重构,产业发展能力也持续提升,百姓生活水平相对于震前有较大幅度提高。

较高的政府信任度会有效降低政府运行的成本,于政府及公民都是有利的。而政府信任会受一系列因素的影响,在突发事件发生的关键时期政府政策执行对政府信任的影响更为显著。政府信任作为公民对政府的主观评价,受诸多因素的影响。现有的研究中不乏对政府影响因素的探讨,程倩认为,政府信任是政府合法性的本质内涵,政府信任关系是公民对政府及其政策执行过程中各相关要素、政策执行过程以及政策执行结果的心理预期。在二者的关系解释上,政府高校的运作过程以及较好的绩效水平可以带来良好的政府信任水平,两者呈正相关关系(程倩,2011)。缪婷婷认为,影响政府信任的因素具体包括政府作为、政府绩效以及新闻媒体的兴起(缪婷婷,2016)。在关键时期,政府政策对居民的政府信任影响更为显著。在突发事件中,公民对政府的信任感往往受领导者的价值观以及政府政策执行情况的影响,政府信任与政府及其工作人员对受灾民众的真诚关怀和激励呈显著正相关的影响(王璐,2008)。

因此,本文在梳理了现有研究成果的基础上,基于“汶川地震灾区灾后重建十周年状况调查”数据,对影响灾区居民政府信任度的影响因素进行实证分析。

2.1人际信任。

在文化主义视角下,人际信任会对政府信任产生显著的影响。高巍通过实证研究证实了人际信任对政府信任的影响:民众越容易相信他人,则更容易信任政府(高巍,2015)。胡荣等学者研究发现个人的信任水平对政府信任度会产生显著影响(胡荣,2007)。陈天祥构建了包括村内信任和社会信任的影响因素模型,验证了民众个人的信任水平对政府信任会产生显著的影响(陈天祥,2017)。人际信任程度越高,越有利于民众与政府的良性互动(刘人宁,2018)。人际信任变量作为文化路径中重要的解释变量,对居民政府信任的影响不言而喻,因此本文将其作为自变量的指标之一。据此,提出假设1:

假设1:人际信任度越高,政府信任度越高;人际信任度越低,政府信任度越低。

2.2政府绩效。

制度主义路径认为政府绩效很大程度上影响人们对政府信任度的判定,政府绩效涵盖政治、经济、环境保护等多方面的绩效。许伟通过研究发现,政府绩效变量对政府信任会产生显著的影响,同时验证了政府绩效与政府信任之间的正相关关系。杨培鸿(2007)通过实证研究证明了公共服务绩效作为关键性的影响因素,对公民政府信任评价起着重要的作用。陈天祥在研究中国乡镇政府的政府信任情况时也将政府绩效作为自变量进行分析(陈天祥等,2017)。由此,提出假设2:

假设2:政府绩效水平与政府信任呈正相关关系。

3.1数据来源。

本研究使用数据来源于国家科技部中国发展战略研究院联同西南交通大学开展的“汶川十周年居民生活情况调查”数据,调查范围包括四川省5市/州30县/区共198个行政村/社区,共抽取了30個受灾县中的4950户住户,共完成家户问卷3826份,完成个人问卷3751份。

3.2变量的操作化。

因变量:政府信任变量。

在“汶川地震十周年居民生活状况调查”问卷中,关于政府信任被划分为对中央政府、省政府、县/市政府、乡/镇/街道四个层级政府的信任,选项分别为“完全信任”、“比较信任”、“不太信任”、“根本不信任”,对四个信任度赋值为1-4,1代表“完全信任”,4代表“根本不信任”,以此类推。通过探索性因子分析提取出“政府信任”因子。

自变量一:人际信任。

“汶川地震十周年居民生活状况调查”问卷中,对人际信任变量操作为题目——对“家人”、“住在你周围的人”、“市场上的商人/买卖人”、“外地人”、“村干部”、“警察”、“医生”、“法官”共8种不同类型群体信任情况,答案与测量“政府信任”的答案一致。通过探索性因子分析,根据指标特征,提取出三个因子,分别为“权威信任”、“社会信任”、“邻里信任”。

自变量二:政府绩效。

本研究中关于政府绩效感知被测量为居民对“贫困问题”、“贫富差距”、“失业情况”、“空气污染”、“水污染”、“缺水问题”、“孩子上学不方便”的严重程度评价,答案包括“特别严重”、“比较严重”、“不太严重”、“不成问题”四个程度的测量,从“特别严重”至“不成问题”分别赋值为1-4。通过因子分析,提取3个公因子为“人民生活”、“环境保护”、“公共服务”。

4.1相关性分析。

通过相关性分析可知,居民人际信任的三个维度与政府信任的相关显著性均为0.000,小于0.05,均呈显著性相关关系,表明民众个人信任水平会显著影响民众对政府的信任,其中权威人群信任维度的得分系数最高,为0.665。

该部分基于因子分析和相关性分析的基础上,采取线性回归方法进一步验证各影响因素与政府信任的关系,建立了2个回归模型,对影响灾区居民政府信任的影响因素进行实证分析。

模型1中检验的是人际信任与政府信任之间的关系。在研究人际信任与政府信任的回归中国,采用逐步回归法,将人际信任的3个公因子作为自变量,政府信任因子作为因变量进行分析。根据选取自变量的规则p0.05,共选择了3个变量,包括权威信任、邻里信任以及社会信任。模型1的相关系数为0.678,调整后的r方为0.459。因此,最终建立的模型1包含权威信任,邻里信任,社会信任3个影响因素。通过anova表考察模型是否有效,该模型回归的显著性水平均显示p0.05,表明回归程度显著,可以认为该模型有效。从回归系数表可知,权威信任、邻里信任以及社会信任的回归系数分别为0.616、0.161、0.085,均大于0,说明权威信任、邻里信任以及社会信任程度越高,居民对政府信任的程度越高。t检验中,权威信任、邻里信任、社会信任和常数项的显著性p值都小于0.05,因而均有显著意义。

模型2检验政府绩效与政府信任之间的关系。在研究政府绩效与政府信任的回归中,采用逐步回归法,将绩效感知的3个因子作为自变量,政府信任因子作为因变量进行分析。根据选取自变量的规则p0.05,共选择了3个变量,包括制环境保护、公共服务以及人民生活三个维度的变量,建立模型2。根据模型2可知,环境保护、公共服务以及人民生活的回归系数分别为-0.179、-0.118、-0.109,絕对值均大于0,回归系数为负,说明环境保护、公共服务以人民生活各方面问题的严重程度越高,居民对政府信任的程度越低,即政府绩效水平与政府信任呈正相关关系,政府绩效越好,居民政府信任度越高。t检验中,环境保护、公共服务、人民生活和常数项的显著性p值都小于0.05,因而均有显著意义。

本章首先在因子分析以及验证各影响因素与因变量存在相关性的基础上进行线性回归分析,进而验证了相关假设。具体结论如下:

(1)假设1得到验证:人际信任度越高,政府信任度越高;人际信任度越低,政府信任度越低。通过分析发现,人际信任对政府信任存在正相关关系。人际信任包括权威信任、社会信任及邻里信任。其中对权威群体的信任是本研究中影响政府信任最显著的因素,人际信任对居民的政府信任产生影响最为显著,其次是邻里信任,社会信任对政府信任的影响程度最低。

人际信任一直都被当作影响政府信任的重要因素,本研究通过实证分析也得出相同的结论。如何通过提高居民的人际信任水平从而提高政府信任水平是当前应该思考的问题。从本文的研究中可知,人际信任中居民对警察、医生、法官、村干部这类具有一定社会公信力的群体有着较高的信任水平,这与我国传统的信任特征有关,人们对这部分人群持有敬畏之心。但是,我们也可以看到在当今社会中,这部分过去极具公信力的人群也面临公信力下降的困境。因此,需提高这部分人群的服务能力,提高公共服务水平,更好地为人民服务。人际信任中得分最低的是社会信任,即对陌生人的信任水平的解释。在人际关系越来越“冷漠”的今天,社会信任显得尤为重要。近年出现的“扶不扶”、“小悦悦”等问题和事件极大的打击了人们的信任感,使得人们对外界保持极高的警惕心,人们越发地变得更为“冷漠”。因此,大力弘扬社会主义核心价值观,严厉打击破坏社会和谐稳定的因素。

(2)假设2得到验证:政府绩效越好,公民对政府的信任越强。政府绩效越差,公民对政府的信任越低。通过回归分析发现,居民对政府绩效的感知对政府信任产生显著的影响。本研究从政府在公共服务、环境保护、人民生活三方面的绩效成果来考察政府绩效对政府信任的影响,三个维度中环境保护绩效对政府信任的影响最为显著。

政府绩效是政府行为的直观体现,政府绩效的好坏直接影响民众对政府的信任水平,提高政府工作绩效是我国政府一直努力的方向。本研究关于灾区政府绩效与政府信任的关系中,人民生活、公共服务、环境保护方面的绩效对政府信任产生显著的影响。因此,提高灾区居民政府信任需要提高政府绩效,转变政府职能,提高政府的公共服务能力和水平。

参考文献。

[2]缪婷婷.政府绩效对政府信任的影响[d].苏州:苏州大学,2016.

[4]高巍.当前中国居民政府信任的区域比较[d].济南:山东大学,2015.

[7]刘人宁.基层政府信任的影响因素研究[d].哈尔滨:黑龙江省社会科学院,2018.

[8]许伟.我国当代政府信任的比较研究[d].武汉:武汉大学,2014.

摘要:及时回应人民群众的期盼和关切是现代政府的一个重要标志。回应人民群众期盼和关切,宏观上要保证经济社会稳定发展,微观上要在具体事件中保障人民切身利益。提升政府回应能力是建设服务型政府的内在要求,是建立和谐社会的有力保障,是增强政府公信力的重要途径,是政府执政能力不可或缺的重要方面,是衡量政府执政水平的重要标杆。

全球治理委员会在《我们的全球之家》的报告中指出,治理有六个基本要素:参与、公开、透明、回应、法治和责任。回应作为治理的要素之一,要求政府在治理过程中,要与老百姓互动,对公众的合法性要求作出及时、负责的反应,不能无故拖延或者不了了之。回应越快,治理的程度越高。

李克强总理说过,及时回应人民群众的期盼和关切是现代政府的一个重要标志。回应人民群众期盼和关切,宏观上要保证经济社会稳定发展,微观上要在具体事件中保障人民切身利益。

1.政府回应的内涵。

政府回应,“就是政府在公共管理过程中,对公众的需求和提出的问题作出积极敏感的反应和回复的过程”[1]。

政府回应的两点要求:其一,政府部门对公众的需求和提出的问题具有敏感性,要及时作出反应,不能置之不理;其二,政府在敏感性的基础上,要作出积极有效的回复,满足公众需求或解决群众问题。

政府回应是个互动过程:对公众来说,可以通过利益诉求渠道表达自身的要求与期望;对政府来说,要及时有效地对公众的诉求给予反应和回复。政府回应的结果可能是一种积极的表态,表明政府对问题的重视;也可能是对满足公众需求,还可能是对公众要求的一种说服和引导[2]。

2.政府回应的要素。

一是回应主体。也就是“谁回应”,主要是政府和政府部门及其工作人员。二是回应对象,也就是“回应谁”,既可以是公民个体,也可以是社会组织,社会大众。三是回应客体,也就是“回应什么”,指公众提出的需要政府满足或解答的诉求。这种诉求可以是明确表达出来的,也可以是潜在的;既可以是反映社会整体利益的诉求,也可以是个人的具体诉求。四是回应渠道,也就是“怎么回应”,即回应的方式和渠道。五是回应反馈,也就是“回应的如何”,即公众对政府的回应作出的反应性信息,并把这种信息反馈给政府。

1.提升政府回应能力是建设服务型政府的内在要求。

传统管制型社会中,政府也要根据社会需要去做事。但由于社会事务相对来说比较简单,而且公众很少有机会向政府提出要求,因此政府不会承受太多回应性的压力。政府之于公众来说,更多的是管控者,缺少对公众的服务和诉求的回应。

服务型政府的建设意味着政府角色由管控者向服务者的转变。服务型政府,以服务人民、服务社会为宗旨。为了更好地提供服务,政府需要准确掌握公众的需求,并及时满足公众需要,解決公众问题,这个过程就是政府积极有效的回应过程。因此,提升政府回应能力是建设服务型政府的内在要求。

2.提升政府回应能力是建立和谐社会的有力保障。

说文解字看“和谐”二字。“和”由“禾”和“口”构成,“禾”代表粮食,“口”表示人。据此,“和”就是指人人有饭吃。“谐”由“言”和“皆”构成,“言”指说话、表达,“皆”是全部的意思。据此,“谐”就是指人人都有表达的、说话的权力,能够畅所欲言。那么“和”“谐”二字连起来,为我们描绘的和谐社会的场景就是:人民在满足物质生活的同时,表达权、参与权、监督权等也都能够得到保障。要实现这样一副场景,就要增强政府的回应性[3]。

而且,从社会稳定的角度说,政府能够及时了解民情、掌握民意,对涉及群众利益的问题能够早发现、早处理、早解决,给予及时回应,才能消除社会中存在的不和谐因素,实现社会的稳定发展。

3.提升政府回应能力是增强政府公信力的重要途径。

2014年3月18日,习近平总书记在河南省兰考县县委常委扩大会议上的讲话中,提到过“塔西佗陷阱”。通俗地讲,“塔西佗陷阱”就是当执掌公权力的政府部门失去公信力时,无论其说真话还是假话,都会被认为说假话;无论其做好事还是坏事,都会被认为是做坏事。习近平总书记在讲话中指出,“我们当然没有走到这一步,但存在的问题也不可谓不严重,必须下大气力加以解决。如果真的到了那一天,就会危及党的执政基础和执政地位。”[4]公信不立,患莫大焉。人民对政府的信任,是政府权威的基础。

提升政府回应能力是增强政府公信力的重要途径。政府回应是政府公信力最直接的来源。如果把政府公信力看作一个“产品”,在它的生产过程中,人民的诉求是“需求侧”,政府回应就是“供给侧”。群众通过各种渠道发出呼声,向政府提出要求,实际上是群众有对政府产生信任的需求,希望政府是一个关心人民疾苦、能够为人民排忧解难的机构,而政府则通过回应向人民群众提供可以信任的政策资源,并使人民知晓政府是关心并愿意保护他们切身利益的可以信赖的管理者。在呼应之间,“需求”和“供给”发生了相互作用并对接起来,政府与人民之间的信任由此建立,并成为一种稳定的社会资本,政府公信力亦随之产生[5]。

在需求与供给间,就像社会经济领域要从拉动需求转向供给侧改革一样,政府不仅要及时回应人民提出的需求,更应该积极主动地了解社会发展和人民权益状况,在需求产生之前主动响应,提高政策水平、提升执政能力、强化服务意识,时刻将改善民生作为政府职能第一要务,以高质量的公共产品和公共服务的“供给”满足人民的需求,保障人民权益,保持政府公信力不断产生和稳定增长[5]。

随着公众参与意识、表达意识、监督意识的增强,公众对于诸如分配制度改革、柴米油盐的价格、房屋拆迁的政策、化工厂选址方案这些问题越来越关注,不仅想了解政府部门“怎么看”,更想知道接下来“怎么办”。政府“回应”正由选答题变为必答题。坦率地说,中央在深化改革开放、高压反腐、改进作风、厉行节约等宏观层面对公众呼声给予了积极回应,但不少地方政府或相关部门回应在新闻事件、网络爆料等质疑时,仍时常显得迟疑、被动甚至恐惧。推进民主政治、打造阳光政府依然成为时代潮流。顺应时代潮流,“回应社会关切”是政府建设的内在要求,知情、参与、表达和监督是必须保障的公众权利,因此,各级政府及政府工作部门必须尽快适应并学会在质疑、追问的“杂音”中执政,就成为一门必修课,无法回避,更不可逃避!

参考文献:

[1]何祖坤.关注政府回应[j].中国行政管理,2000(7).

[2]李严昌.政府回应过程研究[m].北京:中国社会科学出版社,2018.

[4]中共中央文献研究室.做焦裕禄式的县委书记[m].北京:中央文献出版社,2015.

[5]张欧阳.政府回应:政府公信力产生机制的“供给侧”[j].江汉论坛,2017(4).

摘要:以山东省950名青少年学生为调查对象,对青少年的政府信任影响因素进行比较分析与研究,结果发现,理性选择路径、社会文化路径、政治参与路径均对青少年的政府信任有显著影响。在此基础上,提出了改善青少年政府信任的有效策略。

关键词:青少年政府信任影响因素心理机制。

中图分类号:g641文献标识码:a。

1问题的提出。

政府信任,指民众对于政府或政治制度所抱有的信心,相信它们会制定符合民众利益的政策并提供其预期的政策结果[1]。青少年是未来社会主义建设的接班人和生力军,他们的政府信任直接决定了我国政治生态和民主建设的发展与未来。学界关于青少年政府信任现状的研究,得出的结果不尽相同。有些研究表明,当代中国青少年政治信任的总体状况良好[2-3],另外一些研究显示,当代青少年政治认同度不高且呈工见非均衡状态[4]。

加快新旧动能转换是山东省促进经济结构转型升级的重要途径。在新旧动能转换中,各级政府肩负着提供公共产品,支持科技创新,引导产业发展,健全社会保障等重要作用。在这样的背景下,研究山东省青少年的政府信任具有重要的现实意义。

目前关于青少年政府信任的影响因素,主体方面的影响因素主要有青少年的性别、年龄、家庭情况、人际信任、思政政治教育、大众传媒等;客体方面主要有政府的服务水平、公务员的素质、行政执行力等因素[5]。对政治信任的研究主要存在三种竞争性理论解释路径,即理性选择路径、社会文化路径、政治参与路径[6]。青少年政府信任影响因素及心理机制的分析模型,见图1。

什么因素影响山东省青少年的政府信任?三种解释理论预测作用如何?如何提升青少年政府信任?对此,本文进行有益的探索。

2研究方法。

2.1研究对象。

本研究采用随机抽样的方法,对山东省4所中学和4所大学的950名青少年进行调查研究。其中,男生386人(40.6%),女生564人(59.4%);高一学生147人(15.5%),高二学生140人(14.7%),高三学生118人(12.4%),大一学生166人(17.5%),大二学生146人(15.4%),大三学生120人(12.6%),大四学生112人(11.8%);生源地为农村的571人(60.1%),城镇的为379人(39.9%);政治面貌为中共党员(含预备党员)的50人(5.3%),非党员900人(94.7%);学生干部97人(10.2%),非学生干部853人(89.8%)。

2.2研究工具。

通过对国内外大量文献梳理,建构了包含五个因素的大学生政府信任初测问卷,分别为基本信息、政府信任(中央政府、省级政府、市级政府、县区政府、乡镇街道政府)、理性文化路径(制度信任、过程信任、人员信任、结果信任、生活满意度)、社会文化路径(人际信任)、政治参与路径(制度性参与、非制度行参与)。对初测问卷数据实施因子分析和信度分析,最终确定正式问卷包括以上4个维度,共19题。问卷中每个维度的信度检验结果,表明青少年政府信任影响因素问卷是稳定可信的。题项信度分析,见表1。

3调查结果。

3.1青少年政府信任的总体状况。

数据结果显示,在青少年政府信任的四维结构中,受访学生各维度的题项均分在3.591-3.704,处于中等偏上水平。青少年政府信任的总量表得分为3.621,也处于中等偏上水平[7]。青少年政府信任的总体状况,见表2.

青少年在“结果信任”维度上的得分最低,在“制度信任”维度上的得分最高,各维度平均值的大小顺序为:制度信任过程信任人员信任结果信任。

3.2青少年政府信任影响因素的多元回归。

为了分析和比较理性选择路径、社会文化路径以及政治参与路径对于青少年政府信任的影响,构建了四个多元统计分析模型,具体结果见表2。在控制变量方面,模型一到三的结果显示,性别、年级、政治面貌、生源地均对青少年政府信任没有统计上的影响,学生干部这一个变量却具有显著差异。三种理论解释路径对青少年政府信任的影响,见表3。

模型1关注理性选择路径对于因变量的影响。结果显示,除制度信任外,过程信任、人员信任、结果信任、生活满意度均与政府信任具有显著的关系,由此可以推断理性选择路径对于青少年的政府信任具有强大的预测作用。

模型2关注社会文化路径对于因变量的影响。结果显示,人际信任与青少年政府信任具有显著的关系。

模型3关注政治參与路径对于因变量的影响。结果显示,政治参与与青少年政府信任具有显著的关系。

模型4采用逐步回归的方式,保留显著影响青少年政府信任的因子。从表中可以看出,理性选择路径对青少年政府信任的解释力最强(0.282+0.137+0.162=0.581),其次为社会文化路径(0.149),最后为政治参与路径(0.134)。由此可见三种解释路径都对青少年政府信任具有一定的预测力。

4结论与建议。

从表2可以看出,理性选择路径对青少年政府信任的解释力最强,这与现在文献研究结果较为一致。因此,在中国特色社会主义进入新时代的社会背景下,各级政府仍需加大教育投入,加强校园软硬件建设,热爱学生、关注学生、善待学生,不断提高办学水平,让青少年切切实实体会到国家经济和社会发展的成果,提高广大青少年的学校满意度。

社会文化路径(人际信任)对青少年的政府信任有重要的塑造作用。当前青少年学生的人际信任不容乐观,研究发现我国大学生人际信任水平与年代之间呈显著的负相关,说明大学生人际信任正逐年降低[8]。从学校角度来讲,应该通过多种形式加强青少年的“三观”培养和教育,增加社会责任感,提升人格,完善道德,不断提高对学校、教师和同学的信任水平。

政治参与路径对青少年的政府信任有重要影响。学校要通过思想政治教育理论课、选修课、专题讲座、主题班会等形式,加强大学生政治主体观教育,使大学生充分认识其政治权利和义务,增强大学生的政治效能感,提高大学生的政府信任度。高校应该从学生全面发展的高度出发,开设政治实践活动和政治技能训练。积极推行学生自我管理、自我服务、自我教育,鼓励学生参与学校管理,提出建设性的意见和建议;积极与当地政府部门和团群组织协作,开展丰富多彩的政治实践活动,丰富大学生的政治参与经验。

参考文献:

[2]冯岩.当代中国大学生政治信任研究[d].济南大学,2015.

摘要:政府信任反映了居民对政府的认可程度,是政府政策执行的基础。汶川的地震灾区居民政府信任水平没有随着灾区公共服务水平的提高得到相应的提升,通过运用“汶川地震十周年居民生活情况调查”数据,实证分析探究了人际信任、政府绩效与政府信任之间的关系,研究结果显示,政府绩效、人际信任均对政府信任产生显著的影响。

关键词:灾区居民;政府信任;人际信任;政府绩效。

1问题的提出。

2008年汶川8.0级特大地震给震区带来了巨大的财产和生命安全损失,我国政府在救灾所表现出的强大动员能力、应急能力也成为世界关注的焦点。尤其通过政府的各项震后恢复政策,使得汶川灾区的社会经济发展获得历史性机遇,基础公共社会高水平重构,产业发展能力也持续提升,百姓生活水平相对于震前有较大幅度提高。

较高的政府信任度会有效降低政府运行的成本,于政府及公民都是有利的。而政府信任会受一系列因素的影响,在突发事件发生的关键时期政府政策执行对政府信任的影响更为显著。政府信任作为公民对政府的主观评价,受诸多因素的影响。现有的研究中不乏对政府影响因素的探讨,程倩认为,政府信任是政府合法性的本质内涵,政府信任关系是公民对政府及其政策执行过程中各相关要素、政策执行过程以及政策执行结果的心理预期。在二者的关系解释上,政府高校的运作过程以及较好的绩效水平可以带来良好的政府信任水平,两者呈正相关关系(程倩,2011)。缪婷婷认为,影响政府信任的因素具体包括政府作为、政府绩效以及新闻媒体的兴起(缪婷婷,2016)。在关键时期,政府政策对居民的政府信任影响更为显著。在突发事件中,公民对政府的信任感往往受领导者的价值观以及政府政策执行情况的影响,政府信任与政府及其工作人员对受灾民众的真诚关怀和激励呈显著正相关的影响(王璐,2008)。

因此,本文在梳理了现有研究成果的基础上,基于“汶川地震灾区灾后重建十周年状况调查”数据,对影响灾区居民政府信任度的影响因素进行实证分析。

2文献综述及研究假设。

2.1人际信任。

在文化主义视角下,人际信任会对政府信任产生显著的影响。高巍通过实证研究证实了人际信任对政府信任的影响:民众越容易相信他人,则更容易信任政府(高巍,2015)。胡荣等学者研究发现个人的信任水平对政府信任度会产生显著影响(胡荣,2007)。陈天祥构建了包括村内信任和社会信任的影响因素模型,验证了民众个人的信任水平对政府信任会产生显著的影响(陈天祥,2017)。人际信任程度越高,越有利于民众与政府的良性互动(刘人宁,2018)。人际信任变量作为文化路径中重要的解释变量,对居民政府信任的影响不言而喻,因此本文将其作为自变量的指标之一。据此,提出假设1:

假设1:人际信任度越高,政府信任度越高;人际信任度越低,政府信任度越低。

2.2政府绩效。

制度主义路径认为政府绩效很大程度上影响人们对政府信任度的判定,政府绩效涵盖政治、经济、环境保护等多方面的绩效。许伟通过研究发现,政府绩效变量对政府信任会产生显著的影响,同时验证了政府绩效与政府信任之间的正相关关系。杨培鸿(2007)通过实证研究证明了公共服务绩效作为关键性的影响因素,对公民政府信任评价起着重要的作用。陈天祥在研究中国乡镇政府的政府信任情况时也将政府绩效作为自变量进行分析(陈天祥等,2017)。由此,提出假设2:

假设2:政府绩效水平与政府信任呈正相关关系。

3研究方法。

3.1数据来源。

本研究使用数据来源于国家科技部中国发展战略研究院联同西南交通大学开展的“汶川十周年居民生活情况调查”数据,调查范围包括四川省5市/州30县/区共198个行政村/社区,共抽取了30個受灾县中的4950户住户,共完成家户问卷3826份,完成个人问卷3751份。

3.2变量的操作化。

因变量:政府信任变量。

在“汶川地震十周年居民生活状况调查”问卷中,关于政府信任被划分为对中央政府、省政府、县/市政府、乡/镇/街道四个层级政府的信任,选项分别为“完全信任”、“比较信任”、“不太信任”、“根本不信任”,对四个信任度赋值为1-4,1代表“完全信任”,4代表“根本不信任”,以此类推。通过探索性因子分析提取出“政府信任”因子。

自变量一:人际信任。

“汶川地震十周年居民生活状况调查”问卷中,对人际信任变量操作为题目——对“家人”、“住在你周围的人”、“市场上的商人/买卖人”、“外地人”、“村干部”、“警察”、“医生”、“法官”共8种不同类型群体信任情况,答案与测量“政府信任”的答案一致。通过探索性因子分析,根据指标特征,提取出三个因子,分别为“权威信任”、“社会信任”、“邻里信任”。

自变量二:政府绩效。

本研究中关于政府绩效感知被测量为居民对“贫困问题”、“贫富差距”、“失业情况”、“空气污染”、“水污染”、“缺水问题”、“孩子上学不方便”的严重程度评价,答案包括“特别严重”、“比较严重”、“不太严重”、“不成问题”四个程度的测量,从“特别严重”至“不成问题”分别赋值为1-4。通过因子分析,提取3个公因子为“人民生活”、“环境保护”、“公共服务”。

4研究分析。

4.1相关性分析。

通过相关性分析可知,居民人际信任的三个维度与政府信任的相关显著性均为0.000,小于0.05,均呈显著性相关关系,表明民众个人信任水平会显著影响民众对政府的信任,其中权威人群信任维度的得分系数最高,为0.665。

该部分基于因子分析和相关性分析的基础上,采取线性回归方法进一步验证各影响因素与政府信任的关系,建立了2个回归模型,对影响灾区居民政府信任的影响因素进行实证分析。

模型1中检验的是人际信任与政府信任之间的关系。在研究人际信任与政府信任的回归中国,采用逐步回归法,将人际信任的3个公因子作为自变量,政府信任因子作为因变量进行分析。根据选取自变量的规则p0.05,共选择了3个变量,包括权威信任、邻里信任以及社会信任。模型1的相关系数为0.678,调整后的r方为0.459。因此,最终建立的模型1包含权威信任,邻里信任,社会信任3个影响因素。通过anova表考察模型是否有效,该模型回归的显著性水平均显示p0.05,表明回归程度显著,可以认为该模型有效。从回归系数表可知,权威信任、邻里信任以及社会信任的回归系数分别为0.616、0.161、0.085,均大于0,说明权威信任、邻里信任以及社会信任程度越高,居民对政府信任的程度越高。t检验中,权威信任、邻里信任、社会信任和常数项的显著性p值都小于0.05,因而均有显著意义。

模型2检验政府绩效与政府信任之间的关系。在研究政府绩效与政府信任的回归中,采用逐步回归法,将绩效感知的3个因子作为自变量,政府信任因子作为因变量进行分析。根据选取自变量的规则p0.05,共选择了3个变量,包括制环境保护、公共服务以及人民生活三个维度的变量,建立模型2。根据模型2可知,环境保护、公共服务以及人民生活的回归系数分别为-0.179、-0.118、-0.109,絕对值均大于0,回归系数为负,说明环境保护、公共服务以人民生活各方面问题的严重程度越高,居民对政府信任的程度越低,即政府绩效水平与政府信任呈正相关关系,政府绩效越好,居民政府信任度越高。t检验中,环境保护、公共服务、人民生活和常数项的显著性p值都小于0.05,因而均有显著意义。

5研究结论。

本章首先在因子分析以及验证各影响因素与因变量存在相关性的基础上进行线性回归分析,进而验证了相关假设。具体结论如下:

(1)假设1得到验证:人际信任度越高,政府信任度越高;人际信任度越低,政府信任度越低。通过分析发现,人际信任对政府信任存在正相关关系。人际信任包括权威信任、社会信任及邻里信任。其中对权威群体的信任是本研究中影响政府信任最显著的因素,人际信任对居民的政府信任产生影响最为显著,其次是邻里信任,社会信任对政府信任的影响程度最低。

人际信任一直都被当作影响政府信任的重要因素,本研究通过实证分析也得出相同的结论。如何通过提高居民的人际信任水平从而提高政府信任水平是当前应该思考的问题。从本文的研究中可知,人际信任中居民对警察、医生、法官、村干部这类具有一定社会公信力的群体有着较高的信任水平,这与我国传统的信任特征有关,人们对这部分人群持有敬畏之心。但是,我们也可以看到在当今社会中,这部分过去极具公信力的人群也面临公信力下降的困境。因此,需提高这部分人群的服务能力,提高公共服务水平,更好地为人民服务。人际信任中得分最低的是社会信任,即对陌生人的信任水平的解释。在人际关系越来越“冷漠”的今天,社会信任显得尤为重要。近年出现的“扶不扶”、“小悦悦”等问题和事件极大的打击了人们的信任感,使得人们对外界保持极高的警惕心,人们越发地变得更为“冷漠”。因此,大力弘扬社会主义核心价值观,严厉打击破坏社会和谐稳定的因素。

(2)假设2得到验证:政府绩效越好,公民对政府的信任越强。政府绩效越差,公民对政府的信任越低。通过回归分析发现,居民对政府绩效的感知对政府信任产生显著的影响。本研究从政府在公共服务、环境保护、人民生活三方面的绩效成果来考察政府绩效对政府信任的影响,三个维度中环境保护绩效对政府信任的影响最为显著。

政府绩效是政府行为的直观体现,政府绩效的好坏直接影响民众对政府的信任水平,提高政府工作绩效是我国政府一直努力的方向。本研究关于灾区政府绩效与政府信任的关系中,人民生活、公共服务、环境保护方面的绩效对政府信任产生显著的影响。因此,提高灾区居民政府信任需要提高政府绩效,转变政府职能,提高政府的公共服务能力和水平。

参考文献。

[2]缪婷婷.政府绩效对政府信任的影响[d].苏州:苏州大学,2016.

[4]高巍.当前中国居民政府信任的区域比较[d].济南:山东大学,2015.

[7]刘人宁.基层政府信任的影响因素研究[d].哈尔滨:黑龙江省社会科学院,2018.

[8]许伟.我国当代政府信任的比较研究[d].武汉:武汉大学,2014.

[摘要]自从新媒体迅速发展,我国网民规模大幅度扩大。文章运用中国综合社会调查(cgss)2015网民社会意识调查的公开数据,探究普遍信任和政府信任现状以及普遍信任对政府信任的影响效果。研究结果表明,普遍信任同意度介于同意和中立之间,政府信任度呈现“央强地弱”的总体格局。普遍信任对政府信任存在显著的负向影响,且对中央政府的影响程度要高于地方政府。

[关键词]新媒体;普遍信任;政府信任;影响。

[中图分类号]d035[文献标识码]a。

随着新媒体时代的到来,政治生活与网络媒介密切相关,据中国互联网络信息中心(cnnic)发布的第42次《中国互联网络发展状况统计报告》显示,截至2018年6月30日,我国网民规模达8.02亿,互联网普及率为57.7%。可见,网民是民众的一部分,网民在网上表达自己的感受和意愿,体现强大真实的民意。而人际信任和政府信任是信任范畴的两个重要方面。在信任的测量上,人际信任属于社会信任的领域。韦伯(1995)将其划分为普遍信任和特殊信任,普遍信任以相同信仰为基础,也称和陌生人之间的信任;特殊信任以血缘或裙带关系为纽带,建立在自己所熟知的群体之上。而政府信任是民众对政府或政治制度所持有信心的正面期望。因此,网民的政府信任度不仅能够反映民众政府信任的重要组成部分,而且可以在某种程度上展现着社会整体信任水平和国家政府信任的未来趋势。诸多学者在关于人际信任与政府信任的关系研究中,均一致认为人际信任特别是普遍信任是政府信任的基础,普遍信任可以作为解释居民政府信任的有效变量。且随着新媒体的快速发展,网民获取信息方式多元化,网民能从微博、微信等多渠道更及时地了解官方媒体未能及时报道的事态发展与事实真相,可能更能客观、理性地看待政府行为。因此,本文在借鉴前人关于人际信任与政治信任相关研究的基础上,运用cgss2015网民社会意识的调查数据,利用spss软件对调查数据进行深入分析,以了解当前网民对政府信任的现状以及探讨普遍信任对政府信任的影响程度。

1文献回顾与述评。

普遍信任是社会关系网络中重要的社会资本组成要素之一,长期以来深受国内外学者重视,他们通过理论和实证两个层面对人际信任与政府信任之间的关系进行探讨,得出不同的结论。尤斯拉纳(2006)在研究中发现,政府部门之间信任水平会随着人际信任的下跌而降低,政府工作效率会随之发生影响,导致公众对政府信任度下降,即人际信任对政府信任产生正向影响。而stolle(1998)认为,普遍信任会对自己所属的公共组织产生负向影响,即普遍信任越高,政府信任越低。同时,还有部分学者认为人际信任与政府信任之间不存在显著关系,如帕特南(2011)认为应该从理论上区分人际信任和政府信任,二者可能不存在相關关系。

近年来,我国不少学者将普遍信任作为解释变量、政府信任作为被解释变量来研究普遍信任对政府信任的影响。如谢治菊(2013)在基于江浙和贵州高校的实证研究中发现,普遍信任的增强对政府信任的提升有着重要的促进作用。张海良、许伟(2015)在探讨普遍信任与政府信任的关系中,提出普遍信任程度越高,政府信任会随之产生正向影响。赵慧增、王思琦(2016)以西南交通大学为例,普遍信任对政府信任具有显著的正向影响,且与中央政府信任相比,普遍信任对地方政府的影响程度更大。

相较于大量学者研究普遍信任对政府信任的影响,部分国外学者认为信任可以分为政府信任和社会信任,两者是属于不同范畴的概念,并将政府信任作为自变量,普遍信任作为因变量进行研究,得出了相应的成果。什托姆普卡(2005)在对波兰的实证研究中指出,政府信任态势的提高促进了公众间的普遍信任。rahn(1999)、rothstein(2008)等认为,政府信任是影响公众信任他人的重要影响因素,公众对政府的信心程度的提升,直接促进公众信任社会其他人。

关于政府信任的影响因素研究,从不同的角度来看,考察的因素各不相同。从政府的角度来说,社会经济、依法行政、社会治安状况等政府绩效发展良好会提高公众对政府的满意度,进而影响政府信任度。(胡荣、胡康等,2011)从公众的角度来说,公众对公共事务的参与程度展现了他们的互惠态度、社会网络和人际信任的结果,政府关注公民的要求,对行政会产生一定的影响,公民的意愿得以表达,最终促进政府信任度的提升。(高勇,2014)从传播的角度来看,媒体对政府形象的塑造起着不容忽视的作用,公众更关注媒体,便更能多渠道了解社会事件各种观点,进而更能客观、理性看待政府行为,相应地会表现出更加信任政府的态度。此外,刘建平、周云(2017)在对政府信任的影响因素进行探讨时,提出性别、年龄、政治面貌、受教育程度等个人特征因素和文化因素都会对政府信任产生影响。卢海阳、郑逸芳等(2016)基于对我国16个城市的实证研究中,发现引入公共政策满意度及对政府工作满意度后,政府信任程度表现得更加显著。

总的来说,前人对普遍信任与政府信任的关系研究存在争议性,并未有统一的认识,并且国内外研究以理论描述研究居多,定量研究较少。从政府信任影响因素来看,其潜变量还有待深入探究,学者大多从人口学特征变量、政府绩效论、社会资本论等潜变量来研究,可能疏漏了关于公众对公共政策等满意度的影响研究。并且,新媒体时代迅速发展,网民队伍不断扩大,媒体因素对政府信任程度起着举足轻重的作用,而国内外学者基于网民群体的检验较为缺乏。鉴于此,本文致力于通过cgss2015调查数据深入探讨普遍信任对政府信任的影响。

2数据来源与变量测定。

2.1数据来源。

本文所采用的变量的名称、定义和描述性统计如表2所示。

本文将政治信任划分为中央政府信任和地方政府信任,选取问卷中的问题“对中央政府/地方政府,请问您多大程度上信任它们”加以测量,将选项“非常不信任”“比较不信任”“一般”“比较信任”、“非常信任”分别赋值为1、2、3、4、5,平均值越高,表明网民政府信任程度越高。结果发现,中央政府信任均值明显高于地方政府,分别为3.01、1.97。且网民对中央政府比较信任比例最高,约占36.33%,这比地方政府多了20.43个百分比。可见,我国网民对中央政府的信任度明显高于对地方政府的信任度,这与谢星泉(2015)在研究政治信任结构与群体性事件中认为我国公民“央强地弱”的政治信任趋势相一致。

本文将普遍信任作为自变量,选取“在我们这个社会,大多数人是可以信任的”进行数据处理,将选项“非常不信任”“比较不信任”“一般”“比较信任”、“非常信任”分别赋值为1、2、3、4、5,平均值越高,表明网民普遍信任程度越高。结果发现,我国网民对普遍信任度介于“一般”与“比较信任”之间,约为3.27。如表4所示,网民普遍信任程度最高为“一般”,约占36.68%,其次为“比较信任”,约占35.98%。

朱春奎等(2017)认为性别、年龄、收入、教育程度等社会人口属性因素对政府信任具有不同程度的影响,卢海阳等(2016)在公共政策满意度与中央政府信任研究中认为人口学特征、公共政策满意度等方面对政府信任有一定的影响。罗龙真(2014)在基于cgss2010对政府信任与社会信任关联性实证研究中将教育程度、生活幸福感、社会公平感等作为控制变量,实证检验了政府信任的影响效应。生活幸福感本文借鉴前人的研究,选取性别、年龄、政治面貌、学历、职业等人口学特征、民主政治满意度、主观幸福感和时政关注频率为控制变量纳入回归模型中。

3网民政府信任的logistic回归分析。

为研究普遍信任是否对政府信任有正向影响,本文对网民的政府信任进行logistic回归分析。模型1和模型2考察人口学特征、民主政策满意度以及普遍信任变量对网民政府信任度的影响,统计结果见表5。

3.1普遍信任的影响。

模型1和模型2显示,在控制人口学变量及民主政治满意度、主观满意度、时政关注频率等控制变量后,普遍信任在10%的水平上负向影响中央政府信任度,但对地方政府信任度影响不显著,影响系数分别为-0.694和-0.248。这表明,网民普遍信任度越高,政府信任度越低,这与现有的相关的实证研究相抵触,如张海良,许伟等基于数据cgss2010对人际信任、社会公平与政府信任关系的实证研究。由于研究群体的不同,选取控制变量的不同,得出的结论也会有所不同。本文认为普遍信任度对政府信任度的负向影响的解释是:本文选取cgss2015网民社会意识的调查数据进行分析,新媒体背景下,网民更容易依托社交平台与他人交流,刚开始他们愿意选择相信对方,由于网络的虚拟世界存在很多动机不纯或者带有不良目的的行为,经过多起网络交友失范事件报道后,最终使网民趋于培养理性的信任态度,让政府信任度有较大的改变,由此对政府信任产生负向影响。

3.2人口学变量的影响。

在模型1和模型2中,性别在1%的水平上正向影响中央政府信任度,但对地方政府信任度影响不显著,系数分别为0.349和0.012。这可能存在的原因有:首先,这是我国“央强地弱”的政治信任格局的体现;其次,无论在劳动力市场还是政治参与中,性别歧视是常见的现象,这种刻板印象会直接影响公众对女性的信任和支持,长期以来女性便对政府产生较低额度的信任值。而年龄、政治面貌、学历、职业等其他人口学变量对中央政府和地方政府信任度影響均不显著。

3.3其他控制变量的影响。

在模型1、模型2中,网民主观幸福感在1%的水平上正向影响地方政府信任度,但对中央政府信任度未产生显著影响,影响系数分别为1.675和0.530。第一,公众在生活中获得了制度上的保障,自身的幸福感更能表现出来,由此会在一定程度上增强政府信任度,这与刘明明(2016)在对“社会信任对公众主观幸福感的影响研究”的结论相符。第二,地方政府保障制度的执行对公众基本生活以及权益保障有着较大程度的影响,而中央政府的提案等还需地方政府的试点以及真正执行,加之公众主观幸福感会在一定程度上增强政府信任度,最终主观幸福感对地方政府信任度影响更大。而民主政治满意度、时政关注频率均未对中央政府和地方政府产生显著影响。

4结论与讨论。

通过数据调查结果分析,本文得到以下结论:

第一,信任状况方面,我国网民普遍信任同意度平均值为2.78,介于“一般”和“比较信任”之间,中央政府信任均值明显高于地方政府,分别为3.01、1.97,呈现“央强地弱”的政治信任格局。

以“政府绩效与公众信任”为撰写小论文范文

摘要:以山东省950名青少年学生为调查对象,对青少年的政府信任影响因素进行比较分析与研究,结果发现,理性选择路径、社会文化路径、政治参与路径均对青少年的政府信任有显著影响。在此基础上,提出了改善青少年政府信任的有效策略。

关键词:青少年政府信任影响因素心理机制。

中图分类号:g641文献标识码:a。

1问题的提出。

政府信任,指民众对于政府或政治制度所抱有的信心,相信它们会制定符合民众利益的政策并提供其预期的政策结果[1]。青少年是未来社会主义建设的接班人和生力军,他们的政府信任直接决定了我国政治生态和民主建设的发展与未来。学界关于青少年政府信任现状的研究,得出的结果不尽相同。有些研究表明,当代中国青少年政治信任的总体状况良好[2-3],另外一些研究显示,当代青少年政治认同度不高且呈工见非均衡状态[4]。

加快新旧动能转换是山东省促进经济结构转型升级的重要途径。在新旧动能转换中,各级政府肩负着提供公共产品,支持科技创新,引导产业发展,健全社会保障等重要作用。在这样的背景下,研究山东省青少年的政府信任具有重要的现实意义。

目前关于青少年政府信任的影响因素,主体方面的影响因素主要有青少年的性别、年龄、家庭情况、人际信任、思政政治教育、大众传媒等;客体方面主要有政府的服务水平、公务员的素质、行政执行力等因素[5]。对政治信任的研究主要存在三种竞争性理论解释路径,即理性选择路径、社会文化路径、政治参与路径[6]。青少年政府信任影响因素及心理机制的分析模型,见图1。

什么因素影响山东省青少年的政府信任?三种解释理论预测作用如何?如何提升青少年政府信任?对此,本文进行有益的探索。

2研究方法。

2.1研究对象。

本研究采用随机抽样的方法,对山东省4所中学和4所大学的950名青少年进行调查研究。其中,男生386人(40.6%),女生564人(59.4%);高一学生147人(15.5%),高二学生140人(14.7%),高三学生118人(12.4%),大一学生166人(17.5%),大二学生146人(15.4%),大三学生120人(12.6%),大四学生112人(11.8%);生源地为农村的571人(60.1%),城镇的为379人(39.9%);政治面貌为中共党员(含预备党员)的50人(5.3%),非党员900人(94.7%);学生干部97人(10.2%),非学生干部853人(89.8%)。

2.2研究工具。

通过对国内外大量文献梳理,建构了包含五个因素的大学生政府信任初测问卷,分别为基本信息、政府信任(中央政府、省级政府、市级政府、县区政府、乡镇街道政府)、理性文化路径(制度信任、过程信任、人员信任、结果信任、生活满意度)、社会文化路径(人际信任)、政治参与路径(制度性参与、非制度行参与)。对初测问卷数据实施因子分析和信度分析,最终确定正式问卷包括以上4个维度,共19题。问卷中每个维度的信度检验结果,表明青少年政府信任影响因素问卷是稳定可信的。题项信度分析,见表1。

3调查结果。

3.1青少年政府信任的总体状况。

数据结果显示,在青少年政府信任的四维结构中,受访学生各维度的题项均分在3.591-3.704,处于中等偏上水平。青少年政府信任的总量表得分为3.621,也处于中等偏上水平[7]。青少年政府信任的总体状况,见表2.

青少年在“结果信任”维度上的得分最低,在“制度信任”维度上的得分最高,各维度平均值的大小顺序为:制度信任过程信任人员信任结果信任。

3.2青少年政府信任影响因素的多元回归。

为了分析和比较理性选择路径、社会文化路径以及政治参与路径对于青少年政府信任的影响,构建了四个多元统计分析模型,具体结果见表2。在控制变量方面,模型一到三的结果显示,性别、年级、政治面貌、生源地均对青少年政府信任没有统计上的影响,学生干部这一个变量却具有显著差异。三种理论解释路径对青少年政府信任的影响,见表3。

模型1关注理性选择路径对于因变量的影响。结果显示,除制度信任外,过程信任、人员信任、结果信任、生活满意度均与政府信任具有显著的关系,由此可以推断理性选择路径对于青少年的政府信任具有强大的预测作用。

模型2关注社会文化路径对于因变量的影响。结果显示,人际信任与青少年政府信任具有显著的关系。

模型3关注政治參与路径对于因变量的影响。结果显示,政治参与与青少年政府信任具有显著的关系。

模型4采用逐步回归的方式,保留显著影响青少年政府信任的因子。从表中可以看出,理性选择路径对青少年政府信任的解释力最强(0.282+0.137+0.162=0.581),其次为社会文化路径(0.149),最后为政治参与路径(0.134)。由此可见三种解释路径都对青少年政府信任具有一定的预测力。

4结论与建议。

从表2可以看出,理性选择路径对青少年政府信任的解释力最强,这与现在文献研究结果较为一致。因此,在中国特色社会主义进入新时代的社会背景下,各级政府仍需加大教育投入,加强校园软硬件建设,热爱学生、关注学生、善待学生,不断提高办学水平,让青少年切切实实体会到国家经济和社会发展的成果,提高广大青少年的学校满意度。

社会文化路径(人际信任)对青少年的政府信任有重要的塑造作用。当前青少年学生的人际信任不容乐观,研究发现我国大学生人际信任水平与年代之间呈显著的负相关,说明大学生人际信任正逐年降低[8]。从学校角度来讲,应该通过多种形式加强青少年的“三观”培养和教育,增加社会责任感,提升人格,完善道德,不断提高对学校、教师和同学的信任水平。

政治参与路径对青少年的政府信任有重要影响。学校要通过思想政治教育理论课、选修课、专题讲座、主题班会等形式,加强大学生政治主体观教育,使大学生充分认识其政治权利和义务,增强大学生的政治效能感,提高大学生的政府信任度。高校应该从学生全面发展的高度出发,开设政治实践活动和政治技能训练。积极推行学生自我管理、自我服务、自我教育,鼓励学生参与学校管理,提出建设性的意见和建议;积极与当地政府部门和团群组织协作,开展丰富多彩的政治实践活动,丰富大学生的政治参与经验。

参考文献:

[2]冯岩.当代中国大学生政治信任研究[d].济南大学,2015.

以“政府绩效与公众信任”为撰写小论文范文

摘要:政府信任反映了居民对政府的认可程度,是政府政策执行的基础。汶川的地震灾区居民政府信任水平没有随着灾区公共服务水平的提高得到相应的提升,通过运用“汶川地震十周年居民生活情况调查”数据,实证分析探究了人际信任、政府绩效与政府信任之间的关系,研究结果显示,政府绩效、人际信任均对政府信任产生显著的影响。

关键词:灾区居民;政府信任;人际信任;政府绩效。

1问题的提出。

2008年汶川8.0级特大地震给震区带来了巨大的财产和生命安全损失,我国政府在救灾所表现出的强大动员能力、应急能力也成为世界关注的焦点。尤其通过政府的各项震后恢复政策,使得汶川灾区的社会经济发展获得历史性机遇,基础公共社会高水平重构,产业发展能力也持续提升,百姓生活水平相对于震前有较大幅度提高。

较高的政府信任度会有效降低政府运行的成本,于政府及公民都是有利的。而政府信任会受一系列因素的影响,在突发事件发生的关键时期政府政策执行对政府信任的影响更为显著。政府信任作为公民对政府的主观评价,受诸多因素的影响。现有的研究中不乏对政府影响因素的探讨,程倩认为,政府信任是政府合法性的本质内涵,政府信任关系是公民对政府及其政策执行过程中各相关要素、政策执行过程以及政策执行结果的心理预期。在二者的关系解释上,政府高校的运作过程以及较好的绩效水平可以带来良好的政府信任水平,两者呈正相关关系(程倩,2011)。缪婷婷认为,影响政府信任的因素具体包括政府作为、政府绩效以及新闻媒体的兴起(缪婷婷,2016)。在关键时期,政府政策对居民的政府信任影响更为显著。在突发事件中,公民对政府的信任感往往受领导者的价值观以及政府政策执行情况的影响,政府信任与政府及其工作人员对受灾民众的真诚关怀和激励呈显著正相关的影响(王璐,2008)。

因此,本文在梳理了现有研究成果的基础上,基于“汶川地震灾区灾后重建十周年状况调查”数据,对影响灾区居民政府信任度的影响因素进行实证分析。

2文献综述及研究假设。

2.1人际信任。

在文化主义视角下,人际信任会对政府信任产生显著的影响。高巍通过实证研究证实了人际信任对政府信任的影响:民众越容易相信他人,则更容易信任政府(高巍,2015)。胡荣等学者研究发现个人的信任水平对政府信任度会产生显著影响(胡荣,2007)。陈天祥构建了包括村内信任和社会信任的影响因素模型,验证了民众个人的信任水平对政府信任会产生显著的影响(陈天祥,2017)。人际信任程度越高,越有利于民众与政府的良性互动(刘人宁,2018)。人际信任变量作为文化路径中重要的解释变量,对居民政府信任的影响不言而喻,因此本文将其作为自变量的指标之一。据此,提出假设1:

假设1:人际信任度越高,政府信任度越高;人际信任度越低,政府信任度越低。

2.2政府绩效。

制度主义路径认为政府绩效很大程度上影响人们对政府信任度的判定,政府绩效涵盖政治、经济、环境保护等多方面的绩效。许伟通过研究发现,政府绩效变量对政府信任会产生显著的影响,同时验证了政府绩效与政府信任之间的正相关关系。杨培鸿(2007)通过实证研究证明了公共服务绩效作为关键性的影响因素,对公民政府信任评价起着重要的作用。陈天祥在研究中国乡镇政府的政府信任情况时也将政府绩效作为自变量进行分析(陈天祥等,2017)。由此,提出假设2:

假设2:政府绩效水平与政府信任呈正相关关系。

3研究方法。

3.1数据来源。

本研究使用数据来源于国家科技部中国发展战略研究院联同西南交通大学开展的“汶川十周年居民生活情况调查”数据,调查范围包括四川省5市/州30县/区共198个行政村/社区,共抽取了30個受灾县中的4950户住户,共完成家户问卷3826份,完成个人问卷3751份。

3.2变量的操作化。

因变量:政府信任变量。

在“汶川地震十周年居民生活状况调查”问卷中,关于政府信任被划分为对中央政府、省政府、县/市政府、乡/镇/街道四个层级政府的信任,选项分别为“完全信任”、“比较信任”、“不太信任”、“根本不信任”,对四个信任度赋值为1-4,1代表“完全信任”,4代表“根本不信任”,以此类推。通过探索性因子分析提取出“政府信任”因子。

自变量一:人际信任。

“汶川地震十周年居民生活状况调查”问卷中,对人际信任变量操作为题目——对“家人”、“住在你周围的人”、“市场上的商人/买卖人”、“外地人”、“村干部”、“警察”、“医生”、“法官”共8种不同类型群体信任情况,答案与测量“政府信任”的答案一致。通过探索性因子分析,根据指标特征,提取出三个因子,分别为“权威信任”、“社会信任”、“邻里信任”。

自变量二:政府绩效。

本研究中关于政府绩效感知被测量为居民对“贫困问题”、“贫富差距”、“失业情况”、“空气污染”、“水污染”、“缺水问题”、“孩子上学不方便”的严重程度评价,答案包括“特别严重”、“比较严重”、“不太严重”、“不成问题”四个程度的测量,从“特别严重”至“不成问题”分别赋值为1-4。通过因子分析,提取3个公因子为“人民生活”、“环境保护”、“公共服务”。

4研究分析。

4.1相关性分析。

通过相关性分析可知,居民人际信任的三个维度与政府信任的相关显著性均为0.000,小于0.05,均呈显著性相关关系,表明民众个人信任水平会显著影响民众对政府的信任,其中权威人群信任维度的得分系数最高,为0.665。

该部分基于因子分析和相关性分析的基础上,采取线性回归方法进一步验证各影响因素与政府信任的关系,建立了2个回归模型,对影响灾区居民政府信任的影响因素进行实证分析。

模型1中检验的是人际信任与政府信任之间的关系。在研究人际信任与政府信任的回归中国,采用逐步回归法,将人际信任的3个公因子作为自变量,政府信任因子作为因变量进行分析。根据选取自变量的规则p0.05,共选择了3个变量,包括权威信任、邻里信任以及社会信任。模型1的相关系数为0.678,调整后的r方为0.459。因此,最终建立的模型1包含权威信任,邻里信任,社会信任3个影响因素。通过anova表考察模型是否有效,该模型回归的显著性水平均显示p0.05,表明回归程度显著,可以认为该模型有效。从回归系数表可知,权威信任、邻里信任以及社会信任的回归系数分别为0.616、0.161、0.085,均大于0,说明权威信任、邻里信任以及社会信任程度越高,居民对政府信任的程度越高。t检验中,权威信任、邻里信任、社会信任和常数项的显著性p值都小于0.05,因而均有显著意义。

模型2检验政府绩效与政府信任之间的关系。在研究政府绩效与政府信任的回归中,采用逐步回归法,将绩效感知的3个因子作为自变量,政府信任因子作为因变量进行分析。根据选取自变量的规则p0.05,共选择了3个变量,包括制环境保护、公共服务以及人民生活三个维度的变量,建立模型2。根据模型2可知,环境保护、公共服务以及人民生活的回归系数分别为-0.179、-0.118、-0.109,絕对值均大于0,回归系数为负,说明环境保护、公共服务以人民生活各方面问题的严重程度越高,居民对政府信任的程度越低,即政府绩效水平与政府信任呈正相关关系,政府绩效越好,居民政府信任度越高。t检验中,环境保护、公共服务、人民生活和常数项的显著性p值都小于0.05,因而均有显著意义。

5研究结论。

本章首先在因子分析以及验证各影响因素与因变量存在相关性的基础上进行线性回归分析,进而验证了相关假设。具体结论如下:

(1)假设1得到验证:人际信任度越高,政府信任度越高;人际信任度越低,政府信任度越低。通过分析发现,人际信任对政府信任存在正相关关系。人际信任包括权威信任、社会信任及邻里信任。其中对权威群体的信任是本研究中影响政府信任最显著的因素,人际信任对居民的政府信任产生影响最为显著,其次是邻里信任,社会信任对政府信任的影响程度最低。

人际信任一直都被当作影响政府信任的重要因素,本研究通过实证分析也得出相同的结论。如何通过提高居民的人际信任水平从而提高政府信任水平是当前应该思考的问题。从本文的研究中可知,人际信任中居民对警察、医生、法官、村干部这类具有一定社会公信力的群体有着较高的信任水平,这与我国传统的信任特征有关,人们对这部分人群持有敬畏之心。但是,我们也可以看到在当今社会中,这部分过去极具公信力的人群也面临公信力下降的困境。因此,需提高这部分人群的服务能力,提高公共服务水平,更好地为人民服务。人际信任中得分最低的是社会信任,即对陌生人的信任水平的解释。在人际关系越来越“冷漠”的今天,社会信任显得尤为重要。近年出现的“扶不扶”、“小悦悦”等问题和事件极大的打击了人们的信任感,使得人们对外界保持极高的警惕心,人们越发地变得更为“冷漠”。因此,大力弘扬社会主义核心价值观,严厉打击破坏社会和谐稳定的因素。

(2)假设2得到验证:政府绩效越好,公民对政府的信任越强。政府绩效越差,公民对政府的信任越低。通过回归分析发现,居民对政府绩效的感知对政府信任产生显著的影响。本研究从政府在公共服务、环境保护、人民生活三方面的绩效成果来考察政府绩效对政府信任的影响,三个维度中环境保护绩效对政府信任的影响最为显著。

政府绩效是政府行为的直观体现,政府绩效的好坏直接影响民众对政府的信任水平,提高政府工作绩效是我国政府一直努力的方向。本研究关于灾区政府绩效与政府信任的关系中,人民生活、公共服务、环境保护方面的绩效对政府信任产生显著的影响。因此,提高灾区居民政府信任需要提高政府绩效,转变政府职能,提高政府的公共服务能力和水平。

参考文献。

[2]缪婷婷.政府绩效对政府信任的影响[d].苏州:苏州大学,2016.

[4]高巍.当前中国居民政府信任的区域比较[d].济南:山东大学,2015.

[7]刘人宁.基层政府信任的影响因素研究[d].哈尔滨:黑龙江省社会科学院,2018.

[8]许伟.我国当代政府信任的比较研究[d].武汉:武汉大学,2014.

以“政府绩效与公众信任”为撰写小论文范文

摘要:政府机关部门内的人力资源管理中包含的绩效管理,常被大家简称为政府绩效,其是人力资源管理中理论的主要突破环节,也是现代化行政管理不断创新的结果。政府绩效所具有的最大特点就是使用科学的方法、程序以及标准,对政府机关部门的业绩、实际工作以及成就进行准确地评价,并在这一基础上对政府绩效环节中的相关内容进行完善以及提升。根据当前政府绩效的实际情况来看,需要针对地方政府人力资源管理系统对政府绩效影响进行研究,从而更好地推动我国政府绩效的发展。

关键词:地方政府;人力资源管理系统;政府績效;影响;策略。

中图分类号:d630.3文献标识码:a。

0引言。

现阶段,尤其是部分西方比较发达的国家政府,常选用政府绩效评估政府的工作内容,并充分体现出政府绩效所具有的作用。这些国家通过政府绩效评估,对政府相关工作进行管理,促使行政管理以及社会管理取得了有效的统一,促使社会一体化的高度不断提升。因此,地方政府落实人力资源相关管理工作,保证人力资源的规划工作以及人力选拔、配置的高质量完成,促使人力资源管理的水准有所提高,对增加政府绩效存在着重要的作用。本文将主要围绕地方政府人力资源管理系统对政府绩效产生的影响进行研究,并针对研究内容展开多方面的探索,最终总结出地方政府人力资源管理各环节对政府绩效产生的影响。

1地方政府人力资源管理系统以及政府绩效相关概述。

地方政府人力资源管理其实就是当地政府对人力资源进行管理的相关工作,以及协同机制的客观性描述,地方政府人力资源管理的最终目的,是服务于政府的相关工作[1]。若对人力资源管理从宏观的角度来看,人力资源管理的构成以及运行环节主要包括:人力资源管理制度;人力资源管理相关政策;人力资源的管理机构;人力资源管理的总量;人力资源实践环节。如果从微观的角度来看待人力资源管理系统,则该系统正常运行的环节主要包括:人力资源相关规则以及战略内容;工作分析以及组织设计内容;考核绩效内容;薪酬管理;企业员工职业生涯的相关规划内容;人力资源的招聘内容等;人力资源的相关培训内容;企业员工之间的关系协调;企业员工的行为监督以及奖罚内容;人力资源的保证等。从人力资源的结构进行分析,能够发现广义上政府人力资源管理主要是由行政相关人员、国家权力机关相关人员、党务机关相关人员、国家监察机关相关人员、国家审判机关相关人员以及人民团体和事业单位共同组成[2]。

地方政府人力资源管理主要是将提高行政的效率作为目标,并将地方的行政人力资源作为主要的对象,严格按照相关法律法规对其地方政府的人力资源进行规划、开发、获取以及维护的主要管理行为。地方政府人力资源内的主要整体是政府部门,该系统的获取主要是指对公务员的编制管理、考核、职位分析等内容。而地方政府人力资源的维护,主要是对公务员的薪酬进行相关的保证,开发则是将提高公务员的素养作为主要的目的,通过绩效的管理对公务员展开相关培训。

地方政府绩效作为中多维度的政治学理念,对其进行衡量充分反映了政府内外部存在的各种利益,以及力量之间的平衡状态。我国政府人力资源绩效的评估模式常是根据政府系统内的绩效合同进行的相关评估,我国政府与其他部门、部门与官员之间通过绩效合同达成的协议量化分解任务,让各部门分头实现分解后的任务内容。与此同时,当各部分分头完成自身的任务的过程中,必要时可以共同或者抽调组成临时小组完成相关任务内容。我国大多数地级常将绩效管理的结果与人事管理制度相结合,从而提高地级政府人力资源绩效管理的效果[3]。因此,政府部门以及相关工作人员是地级政府人力资源绩效管理的主体,其与政府的管理制度存在不可分割的关系。

2地方政府人力资源管理系统对政府绩效的影响。

2.1人力资源管理政策。

地方各级政府中包含的人力资源管理系统在实际工作中采取的相关策略,与政府绩效之间存在着正相关的关系,对其进行深入的研究能够发现,地方政府人力资源管理所具有的科学、合理的人才政策,吸引并聚焦了大量的人才。地方政府人力资源通过完善岗位管理制度、培训教育制度以及考核奖惩制度等,均有助于地方政府建设高质量的人力资管管理团队,并在提高政府绩效的同时,推动地方政府的人力资源管理发展情况。与此同时,政府作为我国社会组成部分中的重要内容之一,想要提升地方政府人力资管管理的工作水平,促使人力资源管理的水准有所提高,从而带动社会经济的高速发展[4]。但是,政府部分人力资源管理的政策作用,没有充分对政府绩效产生作用,很难充分调动公务员对工作产生的热情以及工作积极性,甚至部分公务员还会对工作出现不满的情绪,这些情况均严重的影响了公务员的工作效率,最终影响政府绩效。

2.2职位管理。

地方政府的职位管理对政府绩效存在着显著的作用。从政府人力资源管理系统来看,岗位管理内容是重要的保障。提高岗位的实际管理力度,并结合地方政府未来发展的切实需求,为更多的人才搭建良好的发展道路,让人才能够充分实现自身的价值,使人才看到更多的工作希望,从而激发人才的创新能力以及创造能力,才能从根本上提高政府绩效的管理水平。此外,地方政府还可以积极推广非领导职务序列,从而缓解部分职务不够分配的情况,让更多的人才获得实现的机会,促使人才充分发挥自身的作用。此外,政府的对岗位管理采取相关的措施,可以让更多的人才对政府工作职位的设置产生一定的了解,并充分掌握相关职位晋升的内容,从而让政府工作实现公开化以及透明程度,让人才产生更多的公平感,促使人才朝向预期进行发展[5]。也就是说,政府通过有效的职位管理措施,能够减少部分人员暗箱操作的可能,并提高人才的认同程度,为人才创造一个公平竞争的环境氛围。

2.3人力资源规划。

人才的选拔以及配置环节,均和政府内部的绩效管理存在着正向的关系。处于当前我国经济发展的模式下,人才资源作为第一的资源,其属于竞争的主要资源组成部分。政府制定合理的人才管理政策,能吸引较多的人才加入到政府部门中。想要不断提高政府绩效的水准,就需要重视人力资管管理中的人才选拔以及配置环节,加大工作的把控力度,必要时可以选择经验丰富的专业型人才,专门负责人才岗位的相关工作,从而构建并完善人才金字塔,推动社会经济的发展[6]。

2.5人才培训。

根据政府绩效的相关要求,政府内部的工作人员,除了需要基本符合要求的政治素养以及理论知识能力之外,还需要具备较高的专业素养水平。现阶段,我国政府的经济逐渐趋向复杂的趋势,这就需要政府重视人才团队的建设工作,并创新自身的管理模式,显著提高人力资源管理的水平。根据政府工作的实际需求,从岗位的实际情况出发,制定合理的人才培养方法,不断挖掘人力资源具有的潜在价值,从而提高政府绩效的水平。

2.6人才晋升。

地方政府人力资源完善晋升制度,能够有效提高系统内人才的工作满意程度,促使人才全身心地投入到工作中,充分发挥自身具有的价值意义,有助于增加政府绩效。地方政府内人才的晋升,代表着人才获得了政府机关的认可,对激发人才的潜在创造力具有较高的积极作用。地方政府为人才制定科学的晋升制度以及渠道,让人才充分预知努力后可能获取的发展道路,促使人才为这一发展目标不断努力,最终通过自身创造的出色业绩,成功实现自己的晋升目标[7]。简单来说,地方政府想要提高政府绩效,首先需要重视并满足人才的价值实现需求,充分调动人才的创造能力以及创新能力,让人才通过高质量的工作,帮助政府实现绩效的目标。

2.7人才参与机制。

地方政府人力资源管理中的人才参与机制,主要是参与决策的人员大多是领导层人员,虽然基层人员具有提出意见的机会,但较多的基层人员常选择将精力放在工作上,严重缺乏参与的精神。此种情况没有驗证人才参与机制对政府绩效产生的正向作用,严重影响了政府绩效的水准。

2.8绩效评估。

从政府绩效评估以及政府内部的绩效管理来看,如果能够充分发挥政府人才资源管理中的绩效评估作用,对政府绩效存在着积极的影响。政府实施有效的绩效考核环节,能够及时发现人才在工作中存在的不足以及实际问题,通过调整并给出建议,可以显著提高政府绩效的水平[8]。

3结束语。

简而言之,地方政府人力资源管理系统对政府绩效产生的影响比较大,这些影响分别体现在政府机关的各个环节中。在地方政府人力资源系统对政府绩效产生的影响下,政府需要深刻认识到公务员是地方政府的行政管理人才,并且公务员也是社会经济建设的重要力量。想要加快地方政府人力资源管理系统的建设,显著提高政府绩效的水平,需要促进人才资本的外溢,推动社会经济的发展、进步。此外,政府机关内的公务员也需提高自身的能力,积极为政府机关奉献力量,推动政府绩效的水准提升。

参考文献:

[8]刘长江,熊莹,张宝龙.基于cnki(1998—2016)政府绩效研究知识图谱的量化评析[j].黑龙江工业学院学报(综合版),2018,18(7):51-60.

学校公众信息撰写心得体会

随着社会发展和科技进步,学校的公众信息传播越来越受到重视。作为一名学校的宣传部成员,我有幸参与了本学期的公众信息撰写工作。通过这一经历,我深感学校公众信息的撰写是一项既充满挑战又富有启迪的任务。下面,我想就我的心得体会与大家分享。

首先,学校公众信息撰写需要充分了解目标受众。在撰写公众信息之前,我们必须明确信息传播的目标人群是谁,以及他们对什么内容感兴趣。只有充分了解受众才能准确把握信息的传达重点和方式。例如,对于学生而言,他们更关心的是有关课程安排、校园活动和学术竞赛等方面的信息,而对于家长而言,则更关心学校的教育教学质量和安全保障措施等方面的信息。因此,我们需要针对不同的受众编写不同的信息,以满足不同人群的需求。

其次,学校公众信息撰写需注意用语规范。作为学校的公众信息,我们必须要以正确、规范的语言进行撰写。首先,要避免使用口语化的表达方式,确保信息的专业性和公信力。其次,要注意避免使用任何歧视性、冒犯性的语言,尊重每个人的文化背景和身份。同时,尽量使用简洁清晰的语言,避免使用过多的专业术语。这样才能更好地传达信息,让读者能够轻松理解。

再次,学校公众信息撰写需要注意信息的准确性和客观性。在撰写过程中,我们应该严谨对待每一个信息,确保其准确无误。不管是学校的活动通知还是学术研讨会的报道,我们都应该进行反复核实,确保没有错误的事实和数据。同时,作为公众信息的传播者,我们应该保持客观中立的态度,不应对信息进行任何歪曲或夸大,以保持信息的可信度和公正性。

另外,学校公众信息撰写需要注意信息的合理组织。在传达信息时,我们应该尽量将信息进行合理分段和组织,以确保读者能够轻松阅读并理解主要内容。可以通过采用标题、副标题、段落等方式进行信息的分级传递,使读者能够迅速获取信息的核心内容。同时,为了避免信息冗长或枯燥,我们还可以加入一些适当的图片、图表和实例来丰富信息的呈现形式,提高读者的阅读兴趣。

最后,学校公众信息撰写需要注意信息的时效性和更新性。随着时代的发展,信息的更新速度越来越快。因此,在撰写公众信息时,我们应该及时获取最新的资讯,确保信息的时效性和准确性。同时,我们还应定期更新和删除过期的信息,以确保读者能够获取到最新、最有价值的信息。

综上所述,学校公众信息的撰写需要充分了解目标受众,注意用语规范,准确和客观地传达信息,合理组织信息,以及保持信息的时效性和更新性。这是一项需要不断学习和提高的能力,但通过这些努力,我们可以更好地为学校服务,传递正能量,为学校的发展做出贡献。

以“政府绩效与公众信任”为撰写小论文范文

[摘要]自从新媒体迅速发展,我国网民规模大幅度扩大。文章运用中国综合社会调查(cgss)2015网民社会意识调查的公开数据,探究普遍信任和政府信任现状以及普遍信任对政府信任的影响效果。研究结果表明,普遍信任同意度介于同意和中立之间,政府信任度呈现“央强地弱”的总体格局。普遍信任对政府信任存在显著的负向影响,且对中央政府的影响程度要高于地方政府。

[关键词]新媒体;普遍信任;政府信任;影响。

[中图分类号]d035[文献标识码]a。

随着新媒体时代的到来,政治生活与网络媒介密切相关,据中国互联网络信息中心(cnnic)发布的第42次《中国互联网络发展状况统计报告》显示,截至2018年6月30日,我国网民规模达8.02亿,互联网普及率为57.7%。可见,网民是民众的一部分,网民在网上表达自己的感受和意愿,体现强大真实的民意。而人际信任和政府信任是信任范畴的两个重要方面。在信任的测量上,人际信任属于社会信任的领域。韦伯(1995)将其划分为普遍信任和特殊信任,普遍信任以相同信仰为基础,也称和陌生人之间的信任;特殊信任以血缘或裙带关系为纽带,建立在自己所熟知的群体之上。而政府信任是民众对政府或政治制度所持有信心的正面期望。因此,网民的政府信任度不仅能够反映民众政府信任的重要组成部分,而且可以在某种程度上展现着社会整体信任水平和国家政府信任的未来趋势。诸多学者在关于人际信任与政府信任的关系研究中,均一致认为人际信任特别是普遍信任是政府信任的基础,普遍信任可以作为解释居民政府信任的有效变量。且随着新媒体的快速发展,网民获取信息方式多元化,网民能从微博、微信等多渠道更及时地了解官方媒体未能及时报道的事态发展与事实真相,可能更能客观、理性地看待政府行为。因此,本文在借鉴前人关于人际信任与政治信任相关研究的基础上,运用cgss2015网民社会意识的调查数据,利用spss软件对调查数据进行深入分析,以了解当前网民对政府信任的现状以及探讨普遍信任对政府信任的影响程度。

1文献回顾与述评。

普遍信任是社会关系网络中重要的社会资本组成要素之一,长期以来深受国内外学者重视,他们通过理论和实证两个层面对人际信任与政府信任之间的关系进行探讨,得出不同的结论。尤斯拉纳(2006)在研究中发现,政府部门之间信任水平会随着人际信任的下跌而降低,政府工作效率会随之发生影响,导致公众对政府信任度下降,即人际信任对政府信任产生正向影响。而stolle(1998)认为,普遍信任会对自己所属的公共组织产生负向影响,即普遍信任越高,政府信任越低。同时,还有部分学者认为人际信任与政府信任之间不存在显著关系,如帕特南(2011)认为应该从理论上区分人际信任和政府信任,二者可能不存在相關关系。

近年来,我国不少学者将普遍信任作为解释变量、政府信任作为被解释变量来研究普遍信任对政府信任的影响。如谢治菊(2013)在基于江浙和贵州高校的实证研究中发现,普遍信任的增强对政府信任的提升有着重要的促进作用。张海良、许伟(2015)在探讨普遍信任与政府信任的关系中,提出普遍信任程度越高,政府信任会随之产生正向影响。赵慧增、王思琦(2016)以西南交通大学为例,普遍信任对政府信任具有显著的正向影响,且与中央政府信任相比,普遍信任对地方政府的影响程度更大。

相较于大量学者研究普遍信任对政府信任的影响,部分国外学者认为信任可以分为政府信任和社会信任,两者是属于不同范畴的概念,并将政府信任作为自变量,普遍信任作为因变量进行研究,得出了相应的成果。什托姆普卡(2005)在对波兰的实证研究中指出,政府信任态势的提高促进了公众间的普遍信任。rahn(1999)、rothstein(2008)等认为,政府信任是影响公众信任他人的重要影响因素,公众对政府的信心程度的提升,直接促进公众信任社会其他人。

关于政府信任的影响因素研究,从不同的角度来看,考察的因素各不相同。从政府的角度来说,社会经济、依法行政、社会治安状况等政府绩效发展良好会提高公众对政府的满意度,进而影响政府信任度。(胡荣、胡康等,2011)从公众的角度来说,公众对公共事务的参与程度展现了他们的互惠态度、社会网络和人际信任的结果,政府关注公民的要求,对行政会产生一定的影响,公民的意愿得以表达,最终促进政府信任度的提升。(高勇,2014)从传播的角度来看,媒体对政府形象的塑造起着不容忽视的作用,公众更关注媒体,便更能多渠道了解社会事件各种观点,进而更能客观、理性看待政府行为,相应地会表现出更加信任政府的态度。此外,刘建平、周云(2017)在对政府信任的影响因素进行探讨时,提出性别、年龄、政治面貌、受教育程度等个人特征因素和文化因素都会对政府信任产生影响。卢海阳、郑逸芳等(2016)基于对我国16个城市的实证研究中,发现引入公共政策满意度及对政府工作满意度后,政府信任程度表现得更加显著。

总的来说,前人对普遍信任与政府信任的关系研究存在争议性,并未有统一的认识,并且国内外研究以理论描述研究居多,定量研究较少。从政府信任影响因素来看,其潜变量还有待深入探究,学者大多从人口学特征变量、政府绩效论、社会资本论等潜变量来研究,可能疏漏了关于公众对公共政策等满意度的影响研究。并且,新媒体时代迅速发展,网民队伍不断扩大,媒体因素对政府信任程度起着举足轻重的作用,而国内外学者基于网民群体的检验较为缺乏。鉴于此,本文致力于通过cgss2015调查数据深入探讨普遍信任对政府信任的影响。

2数据来源与变量测定。

2.1数据来源。

本文所采用的变量的名称、定义和描述性统计如表2所示。

本文将政治信任划分为中央政府信任和地方政府信任,选取问卷中的问题“对中央政府/地方政府,请问您多大程度上信任它们”加以测量,将选项“非常不信任”“比较不信任”“一般”“比较信任”、“非常信任”分别赋值为1、2、3、4、5,平均值越高,表明网民政府信任程度越高。结果发现,中央政府信任均值明显高于地方政府,分别为3.01、1.97。且网民对中央政府比较信任比例最高,约占36.33%,这比地方政府多了20.43个百分比。可见,我国网民对中央政府的信任度明显高于对地方政府的信任度,这与谢星泉(2015)在研究政治信任结构与群体性事件中认为我国公民“央强地弱”的政治信任趋势相一致。

本文将普遍信任作为自变量,选取“在我们这个社会,大多数人是可以信任的”进行数据处理,将选项“非常不信任”“比较不信任”“一般”“比较信任”、“非常信任”分别赋值为1、2、3、4、5,平均值越高,表明网民普遍信任程度越高。结果发现,我国网民对普遍信任度介于“一般”与“比较信任”之间,约为3.27。如表4所示,网民普遍信任程度最高为“一般”,约占36.68%,其次为“比较信任”,约占35.98%。

朱春奎等(2017)认为性别、年龄、收入、教育程度等社会人口属性因素对政府信任具有不同程度的影响,卢海阳等(2016)在公共政策满意度与中央政府信任研究中认为人口学特征、公共政策满意度等方面对政府信任有一定的影响。罗龙真(2014)在基于cgss2010对政府信任与社会信任关联性实证研究中将教育程度、生活幸福感、社会公平感等作为控制变量,实证检验了政府信任的影响效应。生活幸福感本文借鉴前人的研究,选取性别、年龄、政治面貌、学历、职业等人口学特征、民主政治满意度、主观幸福感和时政关注频率为控制变量纳入回归模型中。

3网民政府信任的logistic回归分析。

为研究普遍信任是否对政府信任有正向影响,本文对网民的政府信任进行logistic回归分析。模型1和模型2考察人口学特征、民主政策满意度以及普遍信任变量对网民政府信任度的影响,统计结果见表5。

3.1普遍信任的影响。

模型1和模型2显示,在控制人口学变量及民主政治满意度、主观满意度、时政关注频率等控制变量后,普遍信任在10%的水平上负向影响中央政府信任度,但对地方政府信任度影响不显著,影响系数分别为-0.694和-0.248。这表明,网民普遍信任度越高,政府信任度越低,这与现有的相关的实证研究相抵触,如张海良,许伟等基于数据cgss2010对人际信任、社会公平与政府信任关系的实证研究。由于研究群体的不同,选取控制变量的不同,得出的结论也会有所不同。本文认为普遍信任度对政府信任度的负向影响的解释是:本文选取cgss2015网民社会意识的调查数据进行分析,新媒体背景下,网民更容易依托社交平台与他人交流,刚开始他们愿意选择相信对方,由于网络的虚拟世界存在很多动机不纯或者带有不良目的的行为,经过多起网络交友失范事件报道后,最终使网民趋于培养理性的信任态度,让政府信任度有较大的改变,由此对政府信任产生负向影响。

3.2人口学变量的影响。

在模型1和模型2中,性别在1%的水平上正向影响中央政府信任度,但对地方政府信任度影响不显著,系数分别为0.349和0.012。这可能存在的原因有:首先,这是我国“央强地弱”的政治信任格局的体现;其次,无论在劳动力市场还是政治参与中,性别歧视是常见的现象,这种刻板印象会直接影响公众对女性的信任和支持,长期以来女性便对政府产生较低额度的信任值。而年龄、政治面貌、学历、职业等其他人口学变量对中央政府和地方政府信任度影響均不显著。

3.3其他控制变量的影响。

在模型1、模型2中,网民主观幸福感在1%的水平上正向影响地方政府信任度,但对中央政府信任度未产生显著影响,影响系数分别为1.675和0.530。第一,公众在生活中获得了制度上的保障,自身的幸福感更能表现出来,由此会在一定程度上增强政府信任度,这与刘明明(2016)在对“社会信任对公众主观幸福感的影响研究”的结论相符。第二,地方政府保障制度的执行对公众基本生活以及权益保障有着较大程度的影响,而中央政府的提案等还需地方政府的试点以及真正执行,加之公众主观幸福感会在一定程度上增强政府信任度,最终主观幸福感对地方政府信任度影响更大。而民主政治满意度、时政关注频率均未对中央政府和地方政府产生显著影响。

4结论与讨论。

通过数据调查结果分析,本文得到以下结论:

第一,信任状况方面,我国网民普遍信任同意度平均值为2.78,介于“一般”和“比较信任”之间,中央政府信任均值明显高于地方政府,分别为3.01、1.97,呈现“央强地弱”的政治信任格局。

第二,对政府信任的影响方面,普遍信任对中央政府产生显著的负向影响,而对地方政府信任影响不显著。性别在1%的水平上正向影响中央政府信任度,但对地方政府信任度影响不显著。网民的主观幸福感在1%的水平上正向影响地方政府信任度,而未对中央政府产生显著影响。其他控制变量对中央政府和地方政府影响均不显著。

本文研究的创新点主要有:第一,选择在新媒体背景下考察网民的普遍信任状况,更具有针对性和新颖性。第二,采用logistic回归模型测量有无普遍信任协变量,其他潜在变量对政府信任的影响,研究结论证实了普遍信任对政府信任度有促进作用,同时也证实了其他潜在变量对其影响程度的变化。第三,运用实证方法,为学术界关于普遍信任度对政府信任度的影响争论提供新的理论依据。

[1][波兰]埃里克·尤斯拉纳,著,张敦敏译.信任的道德基础[m].北京:中国社会科学出版社,2006.

[2]stolle,gtogether,bowlingalone:thedevelopmentofgeneralizedtrustinvoluntaryassociations[j].politicalpsychology,1998(19).

[3][美]罗伯特·帕特南著,刘波,祝乃娟,张孜异,等译.独自打保龄球——美国社区的衰落与复兴[m].北京:北京大学出版社,2011.

[4]谢治菊.大学生特殊信任、普遍信任与政府信任成长之关联研究——基于江浙和贵州高校的实证调查[j].民族高等教育研究,2013(04).

[5]張海良,许伟.人际信任、社会公平与政府信任的关系研究——基于数据cgss2010的实证分析[j].理论与改革,2015(01).

[6]赵慧增,王思琦.研究生人际信任与政府信任关联性实证研究——以西南交通大学为例[j].西南交通大学学报(社会科学版),2016(01).

[7]彼得·什托姆普卡.信任——一种社会学理论[m].北京:中华书局,2005.

[8]rahnwm,brehmj,alelectionsasinstitutionsforgeneratingsocialcapital[j].civicengagementinamericandemocracy,1999:111-160.

[9]capital,economicgrowthandqualityofgovernment:thecausalmechanism[j].newpoliticaleconomy,2003,8(1):49-71.

[10]胡荣,胡康,温莹莹.社会资本、政府绩效与城市居民对政府的信任[j].社会学研究,2011(01).

[11]高勇.参与行为与政府信任的关系模式研究[j].社会学研究,2014(05).

[12]刘建平,周云.政府信任的概念、影响因素、变化机制与作用[j].广东社会科学,2017(06).

[13]卢海阳,郑逸芳,黄靖洋.公共政策满意度与中央政府信任——基于中国16个城市的实证分析[j].中国行政管理,2016(08).

[14]谢星全.“央强地弱”政治信任结构与群体性事件[d].西南交通大学,2015.

[15]朱春奎,毛万磊.政府信任的概念测量、影响因素与提升策略[j].厦门大学学报(哲学社会科学版),2017(03).

[16]罗龙真.政府信任与社会信任关联性[d].暨南大学,2014.

[17]刘明明.社会信任对公众主观幸福感的影响研究[j].学习与实践,2016(01).

以“政府绩效与公众信任”为撰写小论文范文

政府信任又称政治信任,一般定义为:在政府或者政治系统运转过程中,与公民的期待相一致的结果或者信念。在最高层面,政府信任是指公民对国家的态度。由于政治信任的多层次性,因此其可能是对特定任职者的态度,也可能是对现存制度的态度。根据社会契约论的观点:政府和公众之间是一种委托和代理的关系,公众将行政权委托给政府,同时希望维持相应的权利和利益;而政府代理公众行使行政权利,并通过履行相应的职能获得相关利益。因此,政府的信任是建立在委托、代理关系之上,一旦政府无法满足公众的期望,就会出现信任危机,从而导致两者之间原有的和谐关系破裂,甚至威胁到社会和国家的稳定。

政府绩效视角,主要是通过对政府的表现进行评估,分析政府信任的原因,即确定政府是否值得信任。个人对政府的信任程度受到政府行政能力、经济总体形势、政策执行等多方面的影响。在国外的研究中,将公众不信任政府的原因归纳为以下几点:政府工作效率低下、公款浪费严重、政策执行错误等原因。

政府绩效是影响政府政治信任程度的关键所在。政府绩效主要的参考标准是政府在社会经济管理过程中的行政结果、行政效益、行政效能,以及对政府在行使职能的过程中的管理能力进行研究[1]。政府的绩效是政府执政能力的表现,主要分为社会效益、经济效益和政治效益是三个方面。从理论上分析,政府信任程度对政府工作的顺利运作具有重大意义,因此它也是政治生活中的重要组成部分。

在我国城市居民中,首先城市居民将政府看作是一个统一的整体。城市居民对地方政府的评价相对较高,对中央政府和省政府的评价相对较低。虽然城市居民对中央政府的评价略低,这也体现出地方政府对城市居民日常生活的作用和影响,表明了城市居民对政府信任的理性。其次政府的绩效能够有效提升政府的信任程度,通过政府决策的透明性、依法治国的执行力度、贫富差距的改善情况、社会治安的管理情况,都会对政府信任程度有所加分。因此对于地方政府而言,城市居民的信任程度需求明显对政府绩效提出了更高的要求,只有这样,才能从根本上提升城市居民对政府的信任程度。

结束语。

随着改革开放的深入,我国政府在经济建设方面取得了巨大的成就,但是由于目前我国正处于社会转型期,在经济发展的同时也伴随着贫富差距加大、社会矛盾激化等多种民生问题,由于住房、医疗、教育等问题未能得到有效的解决。这样不仅严重削弱了政府的执政能力,降低了行政效率,也在一定程度上影响了居民对政府的信任程度,所以在一定程度上阻碍了到国家和社会的良性发展。因此,要求政府必须通过有效措施提升政府绩效,为国家转型提供必要的助力和保证,确保服务型政府的构建。

成都市青白江区公众信息网

1、义和团运动四川的主要领导人是(廖观音)。

2、先后镇压四川义和团运动的四川总督是(奎俊、岑春煊)。

3、威震川西的义和团女英雄(廖观音)就义时,年仅(17)岁。4、1888年4月9日,彭家珍出生于今青白江(姚渡)镇,早年留学(日本),1906年,加入(中国同盟会)。

5、彭家珍勇炸(良弼),被孙中山称为(“我老彭收功弹丸”)。

6、1980年,彭大将军专祠被称为四川省(文物保护)单位,2003年被定位四川省(爱国主义教育基地)基地。

7、曾留学法国勤工俭学的是(彭鸿章),进步**的发起人是(赖明果)。

8、(伍村舟)是第一个回到青白江开展革命活动的共产党员;1927年,担任国民革命军营长的(曾学圃)加入中国共产党。

9、新中国成立前夕,被杀害于成都十二桥的烈士是(吴惠安),被杀害于成都王建墓的烈士是(刘仲宣)。

10、从青白江区走出去的我党早期革命家是(刘继增),1928年3月,他与许权忠、刘志丹、谢子长等,组织领导了(渭化)起义。

11、抗战初期,青白江区牺牲的三位营长是(耿明、易明道、沈德怡)。12、1938年,在城厢家珍公园内建立的抗日阵亡将士纪念碑,是为了纪念在抗战中牺牲的(耿明、易明道)两位烈士。

13、1936年秋,中共地下党员(赵郁仙)来到当时的城厢中学,任女生部(训育)主任兼上史地课,向学生宣传抗日救国思想。

14、1937年秋,(邓锡侯)在家珍公园邓公楼召开军事会议,决定出川抗日,(彭传梁)被誉为日本空军的“克星”。

15、新中国成立前,城厢镇的(卷烟)、纸牌、食品等尤为兴盛,唐家寺的(油纸伞)远近闻名。

16、城厢中学创办于(1927)年,第一任校长是(曾桂陵)。

17、抗战时期,铭贤学校内迁至青白江曾家老寨,铭贤学校从中学到大专部在青白江办学长达(7)年之久。

18、《园艺学和庭院学》的作者是(何泽周)。1981年,长期从事教育工作的(秦季忠)被选为青白江区副区长。

19、开凿于清代,坐落在青白江区人和乡的(蓥华祖师石刻摩崖造像、张家兵洞)均有一定的艺术价值。

20、在近代青白江地区,(曾道)被称为“两袖清风的好官”。

21、近代青白江区著名的革命活动家有加入中国民主同盟的(林伯渊)和加入中国国民党革命委员会的(何梁)。

21、1917年,(曾道)被北京政府授予“将军府将军”街;(余锦源是近代金堂县军阶最高的一个军人。)。

以政府绩效与公众信任为撰写小论文

政府是指国家进行统治和社会管理的机关,是国家表示意志、发布命令和处理事务的机关,实际上是国家代理组织和官吏的总称。以下是为大家整理的关于,欢迎大家前来参考查阅!

政府信任又称政治信任,一般定义为:在政府或者政治系统运转过程中,与公民的期待相一致的结果或者信念。在最高层面,政府信任是指公民对国家的态度。由于政治信任的多层次性,因此其可能是对特定任职者的态度,也可能是对现存制度的态度。根据社会契约论的观点:政府和公众之间是一种委托和代理的关系,公众将行政权委托给政府,同时希望维持相应的权利和利益;而政府代理公众行使行政权利,并通过履行相应的职能获得相关利益。因此,政府的信任是建立在委托、代理关系之上,一旦政府无法满足公众的期望,就会出现信任危机,从而导致两者之间原有的和谐关系破裂,甚至威胁到社会和国家的稳定。

政府绩效视角,主要是通过对政府的表现进行评估,分析政府信任的原因,即确定政府是否值得信任。个人对政府的信任程度受到政府行政能力、经济总体形势、政策执行等多方面的影响。在国外的研究中,将公众不信任政府的原因归纳为以下几点:政府工作效率低下、公款浪费严重、政策执行错误等原因。

政府绩效是影响政府政治信任程度的关键所在。政府绩效主要的参考标准是政府在社会经济管理过程中的行政结果、行政效益、行政效能,以及对政府在行使职能的过程中的管理能力进行研究[1]。政府的绩效是政府执政能力的表现,主要分为社会效益、经济效益和政治效益是三个方面。从理论上分析,政府信任程度对政府工作的顺利运作具有重大意义,因此它也是政治生活中的重要组成部分。

在我国城市居民中,首先城市居民将政府看作是一个统一的整体。城市居民对地方政府的评价相对较高,对中央政府和省政府的评价相对较低。虽然城市居民对中央政府的评价略低,这也体现出地方政府对城市居民日常生活的作用和影响,表明了城市居民对政府信任的理性。其次政府的绩效能够有效提升政府的信任程度,通过政府决策的透明性、依法治国的执行力度、贫富差距的改善情况、社会治安的管理情况,都会对政府信任程度有所加分。因此对于地方政府而言,城市居民的信任程度需求明显对政府绩效提出了更高的要求,只有这样,才能从根本上提升城市居民对政府的信任程度。

结束语。

随着改革开放的深入,我国政府在经济建设方面取得了巨大的成就,但是由于目前我国正处于社会转型期,在经济发展的同时也伴随着贫富差距加大、社会矛盾激化等多种民生问题,由于住房、医疗、教育等问题未能得到有效的解决。这样不仅严重削弱了政府的执政能力,降低了行政效率,也在一定程度上影响了居民对政府的信任程度,所以在一定程度上阻碍了到国家和社会的良性发展。因此,要求政府必须通过有效措施提升政府绩效,为国家转型提供必要的助力和保证,确保服务型政府的构建。

政府业绩评价与传统政府部门管理方法不同,是一种新的管理模式,体现了时代发展对政府行政体制改革的要求。其中,第三方评价作为政府业绩管理的重要机制,能够有效地进行外部平衡,弥补传统政府自我评价的缺陷,在促进我国服务型政府变革方面发挥了不可或缺的推进作用。从西方国家实施第三者评价的经验来看,第三者是指与被评价对象和服务对象独立的第三者,由于第三者和被评价对象(第一者)、被服务对象(第二者)没有行政所属关系和利益关系,也被称为独立的第三者。在西方国家,政府业绩第三者的评价多由一些专业的非政府组织(ngo)作为评价机构。这些非政府组织可以保证第三者的独立性、专业性和权威性要求。

从实践层面看,中国政府业绩的传统评价模式以内部进行的自我评价为主,成为运动员和裁判员,评价过程关闭,评价效果差,影响政府的信赖力。近年来大力发展的政府业绩第三方评价,有效提高政府的信赖力,有助于完善政府业绩监督机制。我国从上世纪90年代初开始重视政府业绩评价,从西方参考经验,从90年代中期开始进行政府业绩评价的理论研究,至今为止。

止,基于我国具体情况的政府绩效第三方评估也有了十几年的实践经验。在这一过程中也发现了一些问题。因此,针对我国的实际情况和实际问题,分析政府绩效第三方评估的未来发展趋势,能够有助于丰富相关理论,引导政府绩效第三方评估更好地发展。

第三方评估与传统的政府自我评估相比,不但具有客观性以及独立性,还具有公正性和专业性等优势,能够有力改变政府部门“既当运动员又当裁判员”的传统局面,但是在实践过程中,我国政府绩效第三方评估的优势并没有得到充分发挥,第三方评估在实践过程中还面临着一些亟待解决的问题。

(一)独立性问题。

政府绩效第三方评估需要做到公平公正,首先应该得到保障的就是“独立性”。然而在我国,由于相关法制尚不健全,对于政府绩效第三方评估的管理制度也不够明确,因此在实践过程中,第三方在评估政府绩效时很容易受到各方的干预,缺乏独立性。这样的情况容易造成评估结果失真、评估结果的公平性和公正性无法保障,评估结果容易遭受质疑,从而丧失第三方评估的意义和作用。

(二)权威性问题。

政府绩效第三方评估的权威性主要依托于评估机构的“专业性”。我国目前的政府绩效第三方评估主要由部分高校或一些商业评估机构承担,这些评估机构的专业性达标与否还得由“官方”说了算。这就在一定程度上造成了第三方评估独立性和权威性的“诺斯悖论”:想要获得独立性就必须具有权威性,想要具有权威性就必须得到官方的认可,想要得到官方的认可又会在一定程度上丧失独立性。

(三)专业性问题。

我国的政府绩效第三方评估起步较晚,到现在为止也只不过积累了短短十几年的实践经验,与二战前就开始有实践经验的一些西方国家相比,在专业性程度上还有很大的差距。我国目前的政府绩效第三方评估机构的资格设置还缺乏一个相对完善的标准,因此各个机构的专业化水平良莠不一,专业人才不足,在评估指标设计、量化分析以及评估结果运用等方面都还存在各种问题。

(四)硬件设施与技术保障问题。

与商业的第三方评估机构相比,这个问题在高校参与的政府绩效评估过程中更为严重。现在已经是信息时代,在评估过程中运用智能终端设备能够有效提高评估效率、提高评估准确性。然而在许多实践中,硬件设施与相关技术得不到保障,评估过程中依然只能采用传统的评估方式,费时又费力。

(五)评估结果的反馈与应用问题。

政府绩效的第三方评估的目的主要是为了发现政府在工作过程中所出现的问题、为了更好地提升政府行政水平。但在实践中,依然存在一些不重视第三方评估结果的现象。一些政府机构对第三方评估的重要性认识不足,认为第三方评估只是“走过场”,抱着这种想法,自然也就缺乏对第三方评估结果的反馈和应用,这样非常不利于提升治理水平和行政能力,第三方评估的结果沦为“花瓶”。

(一)评估机构更加具有独立性、权威性和专业性。

随着法律制度的不断完善和第三方评估人才的不断培养,未来的政府绩效第三方评估将更加具有独立性和权威性。同时对于第三方评估机构资格的设置标准也在不断完善,有利于未来的第三方评估机构将朝着更加专业化的方向发展。

(二)商业性质的第三方评估机构增多。

从目前的实践来看,商业性的第三方评估机构与部分高校相比,具有更加效率、更强独立性、设备与技术保障更完善等优点。未来的第三方评估机构可能会朝着商业性的方向发展,或是以高校与商业性第三方评估机构合作的方式来开展针对政府绩效的第三方评估。

(三)人工智能(artificialintelligence)的引入与运用。

随着政府绩效第三方评估实践的不断发展,评估结果的运用成效也日益显现,政府对于第三方评估的结果越来越重视,第三方评估正在逐渐有力地发挥其应有的作用。在未来,依托于更加先进的信息技术,第三方评估的过程能够变得更加透明、公正公平和公开,第三方评估的结果也能够更加全面、具体。在有授权的情况下,第三方评估的结果可以通过互联网等平台公开,供广大科研工作者进行学术研究和使用,也能够让广大人民群众和媒体通过这些公开的评估結果数据对政府的工作进行监督,在很大程度上提高政府的公信力。

根据问题分析和未来发展趋势分析,要更好地发展我国政府绩效第三方评估,应从以下几个方面着力:

(一)紧扣国情、借鉴国外。

我国的政府绩效第三方评估实践起步较晚,因此可以借鉴许多国外已有的理论或经验,在紧扣我国国情的基础之上进行实践,不断进行理论丰富和创新,最终形成与我国实际情况更加符合的、具有我国特色的政府绩效第三方评估理论,能够从实践层面上更好地指导我国政府绩效第三方评估的发展。

(二)做好法制保障、制度建设与监督工作。

完善相关法律制度与管理制度,能够有效提升第三方评估机构的独立性、专业性和权威性。建立健全的对第三方法评估机构的监管制度、准入机制、行业标准,才能够有利于我国第三方评估机构健康、长久地发展。

(三)推进相关技术运用与开发。

科学技术是第一生产力。相关科学技术的成熟与运用是一个行业或产业快速发展的重要助推器。在已有的实践基础上,还应该大力推进与政府绩效第三方评估相关的技术开发与运用。例如智能辅助评估终端、评估数据和结果信息的网络化管理等等,都是非常有助于提升评估效率和科学性的技术。

综上所述,政府绩效第三方评估在我国的实践中依然还存在一些问题,还有很长的路要走,但总的发展趋势是好的,是在不断向前的。经过十几年的实践,我国的政府绩效第三方评估也取得了十分喜人的成效,越来越多的政府机构开始引入第三方评估的机制、开始重视第三方评估结果的作用、开始公开和反馈第三方评估的结果与数据。未来,我国政府绩效第三方评估一定会朝着更好的方向发展,为深化我国行政体制改革、提升政府行政水平、建设服务型政府提供助力。

【参考文献】。

[1]杜娟、杜义国、张微:“我国政府绩效第三方评估的研究现状及未来展望”,《领导科学》,2019年06期。

[2]徐双敏、陈尉:“第三方评估政府绩效的制度环境分析”,《学习与实践》,2013年09期。

[3]孟惠南:“第三方评估在我国政府绩效评估中的应用”,《领导科学》,2012年23期。

摘要:素质教育的宗旨是以人为本,面向全体学生,使人人学有专长,从而大面积提高教学质量。这就要求我们要更新观念,重视后进生,关心后进生,研究后进生,帮助后进生,转化后进生。

关键词:关爱;尊重;信任;后进生。

没有爱的教育是不会取得成功的,对后进生的爱尤为重要,后进生对教师给予的爱就像茁壮成长的小树渴望雨露和太阳。一些后进生是留守学生,一些是由于家庭上的烦恼,生活上的困难影响了学习。后进生往往因成绩不理想,有自卑心理而愧于见老师,更不会主动地向老师提问。长此以往,问题越聚越多,学习就越来越感到困难;教师必须与后进生有所沟通,与他们建立感情,让他们真正体会到老师对他们真诚的关爱,在心理上有亲切感,信任感,老师愿意与他们交朋友。古人曰:“亲其师,信其道。”爱的力量是无穷的,把我们无私的、真挚的爱倾注给后进生,打开他们心灵的窗户,让他们沐浴阳光,感受温暖。

教育也是一种尊重,苏霍姆林斯基说:“教育的核心,就其本质来说,就在于让学生始终体验到自己的尊严感:我是一个勤奋的脑力劳动者,是祖国的好公民,是父母的好儿女,是一个有着高尚的志趣、激情和不断取得进步的完美的人。”怎样看待后进生,把他们看成什么样的人,对他们的态度,一直是教育理论和教育实践中的重要问题。在平时教学中不自觉地对他们冷眼相看,对他们的关注不够,有偏见,会使他们在人格上有自卑感,在学习中缺少自信,尊重他们做人的尊严和价值,平等公正地对待他们,课堂上有意对他们投送亲切的目光和友好的微笑,让他们回答一些较容易的问题,回答有困难时给予引导性的提示,答对了便进行表扬,鼓励。这样增強了后进生的自尊心,可以使他们感到自己和所有的同学一样,都是集体的一份。尊重后进生,赞赏他们的兴趣、爱好、特长及微小的进步,是转化后进生的关键。

后进生由于学习不理想或纪律性差,长期受到排斥,他们一般都心虚,对外界很敏感,他们内心深处渴望得到老师和同学的理解和信任。根据后进生的心理特点,教师在转化工作时,不能持有偏见,应对他们充分信任,要善于捕捉他们身上的闪光点,给予肯定,并及时给予鼓励,使他们的心理上得到满足,教师要适时的,不断地鼓励他们把这些闪光点发扬光大,去照亮那些偏“暗”的方面。在生活和学习中,多与他们接触,多参与他们的讨论,使他们感受教师的关心和帮助,感受到自己在班集体中的重要地位,从而加强他们的自律意识,促使他们良好的心理状态和行为习惯的养成。使他们树立起“我能行”“我也行”的观念,实现身心如一的转化,并在健康发展中发挥他们的潜能。

后进生不是永恒的,作为教师只要找准转化后进生的有效方法,以身作则,以理服人,以情感人;关爱、尊重、信任他们,使这些迟开的花转变得更加灿烂,鲜艳,芳香四溢。

一排排整齐的小楼房,家家户户的大门都敞亮地开着,走过去,几乎能看见每家每户的大客厅。大彩电、八仙桌、冰箱,一应皆有,条件早己不比城里差。在这个安静的小山村,那一座座敞开的楼房,就像敞开的怀抱,随时欢迎着远方客人的到来。

镇上每逢农历的一、四、七都有集市,方圆十里的乡亲们都会赶着日子去镇上进行买卖活动。小超市、小商铺、小地摊,—下子比平常热闹了好几分。母亲见我一年难得回一次家,便吆喝着全家去赶集,匆匆忙忙,没来得及跟邻居打招呼,便到村口的公交站去赶公交车去了。走的时候,母亲没锁门,我提醒了—下,母亲说:没事,我们从来不锁门。

中午吃饭的时候,为了让我安心吃饭,父亲抱着外孙女串门去了。街头巷尾的邻居们都开着门,父亲如同在自家一样悠闲地抱着孩子四处串门。他抱着孩子先在巷尾的良叔家聊了一会天,良叔家的孙女和我闺女一般大,两个小小孩乐呵呵地在地垫上打滚。等他家要开始吃饭了,他又接着抱着孩子去了村头的菁姨家,直到觉得我吃得差不多了,才抱孩子回家来。

吃完饭,儿子闲得无聊,约了堂妹家的哥哥一起去村里的小广场玩去了,打篮球、玩双杠,不一会玩得满头大汗,口渴得不行。儿子想回外婆家喝水,广场边的阿姨走出来:“还回外婆家喝水干嘛呀,来我家喝吧,喝完了继续痛快地玩”。儿子果真去人家家里喝了水,玩了—下午才回来。

在这个安静的村庄,邻居家的门时常都敞开着,喝水、聊天,是经常有的事。人们可以自由地逛逛这家遛遛那家,侃大山、话闲篇、逗小孩,大家都乐在其中。每逢过年闲下来的时候,大家也一起打牌,赢的输的都是块块钱,图的是一份轻松娱乐。因为门敞开着,热闹的声音便传了出来,成了村子里美好的咏叹调,此起彼伏,一片欢乐。

记得母亲有一次得了急性阑尾炎,父亲正好不在家,儿女又不在身边,一个人疼得走不了路,趴在门口疼得直叫。幸好邻居及时发现,迅速送去了医院,母亲的病才得到了及时的治疗。要不是敞开着门,邻居也就听不到叫喊声,也没那么快能及时赶到。感谢这个不成文的習惯,让邻里之间的关爱得到了延续。

[摘要]自从新媒体迅速发展,我国网民规模大幅度扩大。文章运用中国综合社会调查(cgss)2015网民社会意识调查的公开数据,探究普遍信任和政府信任现状以及普遍信任对政府信任的影响效果。研究结果表明,普遍信任同意度介于同意和中立之间,政府信任度呈现“央强地弱”的总体格局。普遍信任对政府信任存在显著的负向影响,且对中央政府的影响程度要高于地方政府。

[关键词]新媒体;普遍信任;政府信任;影响。

[中图分类号]d035[文献标识码]a。

随着新媒体时代的到来,政治生活与网络媒介密切相关,据中国互联网络信息中心(cnnic)发布的第42次《中国互联网络发展状况统计报告》显示,截至2018年6月30日,我国网民规模达8.02亿,互联网普及率为57.7%。可见,网民是民众的一部分,网民在网上表达自己的感受和意愿,体现强大真实的民意。而人际信任和政府信任是信任范畴的两个重要方面。在信任的测量上,人际信任属于社会信任的领域。韦伯(1995)将其划分为普遍信任和特殊信任,普遍信任以相同信仰为基础,也称和陌生人之间的信任;特殊信任以血缘或裙带关系为纽带,建立在自己所熟知的群体之上。而政府信任是民众对政府或政治制度所持有信心的正面期望。因此,网民的政府信任度不仅能够反映民众政府信任的重要组成部分,而且可以在某种程度上展现着社会整体信任水平和国家政府信任的未来趋势。诸多学者在关于人际信任与政府信任的关系研究中,均一致认为人际信任特别是普遍信任是政府信任的基础,普遍信任可以作为解释居民政府信任的有效变量。且随着新媒体的快速发展,网民获取信息方式多元化,网民能从微博、微信等多渠道更及时地了解官方媒体未能及时报道的事态发展与事实真相,可能更能客观、理性地看待政府行为。因此,本文在借鉴前人关于人际信任与政治信任相关研究的基础上,运用cgss2015网民社会意识的调查数据,利用spss软件对调查数据进行深入分析,以了解当前网民对政府信任的现状以及探讨普遍信任对政府信任的影响程度。

1文献回顾与述评。

普遍信任是社会关系网络中重要的社会资本组成要素之一,长期以来深受国内外学者重视,他们通过理论和实证两个层面对人际信任与政府信任之间的关系进行探讨,得出不同的结论。尤斯拉纳(2006)在研究中发现,政府部门之间信任水平会随着人际信任的下跌而降低,政府工作效率会随之发生影响,导致公众对政府信任度下降,即人际信任对政府信任产生正向影响。而stolle(1998)认为,普遍信任会对自己所属的公共组织产生负向影响,即普遍信任越高,政府信任越低。同时,还有部分学者认为人际信任与政府信任之间不存在显著关系,如帕特南(2011)认为应该从理论上区分人际信任和政府信任,二者可能不存在相關关系。

近年来,我国不少学者将普遍信任作为解释变量、政府信任作为被解释变量来研究普遍信任对政府信任的影响。如谢治菊(2013)在基于江浙和贵州高校的实证研究中发现,普遍信任的增强对政府信任的提升有着重要的促进作用。张海良、许伟(2015)在探讨普遍信任与政府信任的关系中,提出普遍信任程度越高,政府信任会随之产生正向影响。赵慧增、王思琦(2016)以西南交通大学为例,普遍信任对政府信任具有显著的正向影响,且与中央政府信任相比,普遍信任对地方政府的影响程度更大。

相较于大量学者研究普遍信任对政府信任的影响,部分国外学者认为信任可以分为政府信任和社会信任,两者是属于不同范畴的概念,并将政府信任作为自变量,普遍信任作为因变量进行研究,得出了相应的成果。什托姆普卡(2005)在对波兰的实证研究中指出,政府信任态势的提高促进了公众间的普遍信任。rahn(1999)、rothstein(2008)等认为,政府信任是影响公众信任他人的重要影响因素,公众对政府的信心程度的提升,直接促进公众信任社会其他人。

关于政府信任的影响因素研究,从不同的角度来看,考察的因素各不相同。从政府的角度来说,社会经济、依法行政、社会治安状况等政府绩效发展良好会提高公众对政府的满意度,进而影响政府信任度。(胡荣、胡康等,2011)从公众的角度来说,公众对公共事务的参与程度展现了他们的互惠态度、社会网络和人际信任的结果,政府关注公民的要求,对行政会产生一定的影响,公民的意愿得以表达,最终促进政府信任度的提升。(高勇,2014)从传播的角度来看,媒体对政府形象的塑造起着不容忽视的作用,公众更关注媒体,便更能多渠道了解社会事件各种观点,进而更能客观、理性看待政府行为,相应地会表现出更加信任政府的态度。此外,刘建平、周云(2017)在对政府信任的影响因素进行探讨时,提出性别、年龄、政治面貌、受教育程度等个人特征因素和文化因素都会对政府信任产生影响。卢海阳、郑逸芳等(2016)基于对我国16个城市的实证研究中,发现引入公共政策满意度及对政府工作满意度后,政府信任程度表现得更加显著。

总的来说,前人对普遍信任与政府信任的关系研究存在争议性,并未有统一的认识,并且国内外研究以理论描述研究居多,定量研究较少。从政府信任影响因素来看,其潜变量还有待深入探究,学者大多从人口学特征变量、政府绩效论、社会资本论等潜变量来研究,可能疏漏了关于公众对公共政策等满意度的影响研究。并且,新媒体时代迅速发展,网民队伍不断扩大,媒体因素对政府信任程度起着举足轻重的作用,而国内外学者基于网民群体的检验较为缺乏。鉴于此,本文致力于通过cgss2015调查数据深入探讨普遍信任对政府信任的影响。

2数据来源与变量测定。

2.1数据来源。

本文所采用的变量的名称、定义和描述性统计如表2所示。

本文将政治信任划分为中央政府信任和地方政府信任,选取问卷中的问题“对中央政府/地方政府,请问您多大程度上信任它们”加以测量,将选项“非常不信任”“比较不信任”“一般”“比较信任”、“非常信任”分别赋值为1、2、3、4、5,平均值越高,表明网民政府信任程度越高。结果发现,中央政府信任均值明显高于地方政府,分别为3.01、1.97。且网民对中央政府比较信任比例最高,约占36.33%,这比地方政府多了20.43个百分比。可见,我国网民对中央政府的信任度明显高于对地方政府的信任度,这与谢星泉(2015)在研究政治信任结构与群体性事件中认为我国公民“央强地弱”的政治信任趋势相一致。

本文将普遍信任作为自变量,选取“在我们这个社会,大多数人是可以信任的”进行数据处理,将选项“非常不信任”“比较不信任”“一般”“比较信任”、“非常信任”分别赋值为1、2、3、4、5,平均值越高,表明网民普遍信任程度越高。结果发现,我国网民对普遍信任度介于“一般”与“比较信任”之间,约为3.27。如表4所示,网民普遍信任程度最高为“一般”,约占36.68%,其次为“比较信任”,约占35.98%。

朱春奎等(2017)认为性别、年龄、收入、教育程度等社会人口属性因素对政府信任具有不同程度的影响,卢海阳等(2016)在公共政策满意度与中央政府信任研究中认为人口学特征、公共政策满意度等方面对政府信任有一定的影响。罗龙真(2014)在基于cgss2010对政府信任与社会信任关联性实证研究中将教育程度、生活幸福感、社会公平感等作为控制变量,实证检验了政府信任的影响效应。生活幸福感本文借鉴前人的研究,选取性别、年龄、政治面貌、学历、职业等人口学特征、民主政治满意度、主观幸福感和时政关注频率为控制变量纳入回归模型中。

3网民政府信任的logistic回归分析。

为研究普遍信任是否对政府信任有正向影响,本文对网民的政府信任进行logistic回归分析。模型1和模型2考察人口学特征、民主政策满意度以及普遍信任变量对网民政府信任度的影响,统计结果见表5。

3.1普遍信任的影响。

模型1和模型2显示,在控制人口学变量及民主政治满意度、主观满意度、时政关注频率等控制变量后,普遍信任在10%的水平上负向影响中央政府信任度,但对地方政府信任度影响不显著,影响系数分别为-0.694和-0.248。这表明,网民普遍信任度越高,政府信任度越低,这与现有的相关的实证研究相抵触,如张海良,许伟等基于数据cgss2010对人际信任、社会公平与政府信任关系的实证研究。由于研究群体的不同,选取控制变量的不同,得出的结论也会有所不同。本文认为普遍信任度对政府信任度的负向影响的解释是:本文选取cgss2015网民社会意识的调查数据进行分析,新媒体背景下,网民更容易依托社交平台与他人交流,刚开始他们愿意选择相信对方,由于网络的虚拟世界存在很多动机不纯或者带有不良目的的行为,经过多起网络交友失范事件报道后,最终使网民趋于培养理性的信任态度,让政府信任度有较大的改变,由此对政府信任产生负向影响。

3.2人口学变量的影响。

在模型1和模型2中,性别在1%的水平上正向影响中央政府信任度,但对地方政府信任度影响不显著,系数分别为0.349和0.012。这可能存在的原因有:首先,这是我国“央强地弱”的政治信任格局的体现;其次,无论在劳动力市场还是政治参与中,性别歧视是常见的现象,这种刻板印象会直接影响公众对女性的信任和支持,长期以来女性便对政府产生较低额度的信任值。而年龄、政治面貌、学历、职业等其他人口学变量对中央政府和地方政府信任度影響均不显著。

3.3其他控制变量的影响。

在模型1、模型2中,网民主观幸福感在1%的水平上正向影响地方政府信任度,但对中央政府信任度未产生显著影响,影响系数分别为1.675和0.530。第一,公众在生活中获得了制度上的保障,自身的幸福感更能表现出来,由此会在一定程度上增强政府信任度,这与刘明明(2016)在对“社会信任对公众主观幸福感的影响研究”的结论相符。第二,地方政府保障制度的执行对公众基本生活以及权益保障有着较大程度的影响,而中央政府的提案等还需地方政府的试点以及真正执行,加之公众主观幸福感会在一定程度上增强政府信任度,最终主观幸福感对地方政府信任度影响更大。而民主政治满意度、时政关注频率均未对中央政府和地方政府产生显著影响。

4结论与讨论。

通过数据调查结果分析,本文得到以下结论:

第一,信任状况方面,我国网民普遍信任同意度平均值为2.78,介于“一般”和“比较信任”之间,中央政府信任均值明显高于地方政府,分别为3.01、1.97,呈现“央强地弱”的政治信任格局。

第二,对政府信任的影响方面,普遍信任对中央政府产生显著的负向影响,而对地方政府信任影响不显著。性别在1%的水平上正向影响中央政府信任度,但对地方政府信任度影响不显著。网民的主观幸福感在1%的水平上正向影响地方政府信任度,而未对中央政府产生显著影响。其他控制变量对中央政府和地方政府影响均不显著。

本文研究的创新点主要有:第一,选择在新媒体背景下考察网民的普遍信任状况,更具有针对性和新颖性。第二,采用logistic回归模型测量有无普遍信任协变量,其他潜在变量对政府信任的影响,研究结论证实了普遍信任对政府信任度有促进作用,同时也证实了其他潜在变量对其影响程度的变化。第三,运用实证方法,为学术界关于普遍信任度对政府信任度的影响争论提供新的理论依据。

[1][波兰]埃里克·尤斯拉纳,著,张敦敏译.信任的道德基础[m].北京:中国社会科学出版社,2006.

[2]stolle,gtogether,bowlingalone:thedevelopmentofgeneralizedtrustinvoluntaryassociations[j].politicalpsychology,1998(19).

[3][美]罗伯特·帕特南著,刘波,祝乃娟,张孜异,等译.独自打保龄球——美国社区的衰落与复兴[m].北京:北京大学出版社,2011.

[4]谢治菊.大学生特殊信任、普遍信任与政府信任成长之关联研究——基于江浙和贵州高校的实证调查[j].民族高等教育研究,2013(04).

[5]張海良,许伟.人际信任、社会公平与政府信任的关系研究——基于数据cgss2010的实证分析[j].理论与改革,2015(01).

[6]赵慧增,王思琦.研究生人际信任与政府信任关联性实证研究——以西南交通大学为例[j].西南交通大学学报(社会科学版),2016(01).

[7]彼得·什托姆普卡.信任——一种社会学理论[m].北京:中华书局,2005.

[8]rahnwm,brehmj,alelectionsasinstitutionsforgeneratingsocialcapital[j].civicengagementinamericandemocracy,1999:111-160.

[9]capital,economicgrowthandqualityofgovernment:thecausalmechanism[j].newpoliticaleconomy,2003,8(1):49-71.

[10]胡荣,胡康,温莹莹.社会资本、政府绩效与城市居民对政府的信任[j].社会学研究,2011(01).

[11]高勇.参与行为与政府信任的关系模式研究[j].社会学研究,2014(05).

[12]刘建平,周云.政府信任的概念、影响因素、变化机制与作用[j].广东社会科学,2017(06).

[13]卢海阳,郑逸芳,黄靖洋.公共政策满意度与中央政府信任——基于中国16个城市的实证分析[j].中国行政管理,2016(08).

[14]谢星全.“央强地弱”政治信任结构与群体性事件[d].西南交通大学,2015.

[15]朱春奎,毛万磊.政府信任的概念测量、影响因素与提升策略[j].厦门大学学报(哲学社会科学版),2017(03).

[16]罗龙真.政府信任与社会信任关联性[d].暨南大学,2014.

[17]刘明明.社会信任对公众主观幸福感的影响研究[j].学习与实践,2016(01).

摘要:近年来,随着中国经济的持续发展和我国政府审计工作的改善,业绩审计在政府监督中发挥着越来越重要的作用。本文阐述了政府绩效审计的现状,深入分析了中国政府发展绩效审计的必要性,揭示了当前政府绩效审计存在的问题及其原因,最后从审计观念、审计监督制度、审计法律环境和预算制度等方面提出了优化政府绩效审计途径的建议,为中国政府绩效审计的高效合理运行和持续发展提供了一定的参考。

关键词:政府业绩审计审计监督优化路径公共管理。

近年来,随着我国经济持续发展和制度的不断完善,人民群众对政府公共财政支出的效率和效果关注度也日益提高,为了加强政府责任,适应我国建设社会主义法治国家的客观要求,产生了政府业绩评价。经过数十年的发展,政府审计在理论和实践层面进入了新阶段,现在以业绩审计为政府审计的重要内容,成为政府审计的新发展趋势。目前,我国政府绩效审计尚不成熟,仍处于起步阶段,存在诸多问题,如何构建适合我国国情的绩效审计体系成为我国政府审计的关键。

(一)是建设廉洁透明服务型政府的需要。

党的十九大报告明确了积极安全推进政治体制改革,建设高效透明的服务型政府。如今,我国正处在经济高速发展阶段,政府公共财政支出成为社会焦点问题。政府绩效考核的主要内容是对政府官员在任期间的工作情况和成果进行绩效考核和财政资金运用的有效性和经济性考核,政府绩效考核的开展拓宽了考核监督的领域和范围,加强了考核机构在行政监督中的作用,增强了各级政府工作人员的责任感,减少了腐败事件的发生,进一步避免了大量经济损失的发生,为建设高效透明的服务型政府营造了良好的氛围。

(2)是提高公共管理效率的需要。

近年来,政府财政收入大幅增加,公共管理投入增加,但公共建设相关项目从立项到最终完成需要多部门多层次审查,项目实施时失去最佳实施时机,同时浪费大量人力物力,这些现象的出现强调政府机构工作管理效率低,人浮于事件。审计机构在对政府部门及其下属单位进行绩效审计过程中,对公共资源配置的经济效率、配置效果等方面进行审查,有效管理和评价政府权力的使用,提高政府行政服务的工作效率水平。

(3)是保证政府职能发挥作用的需要。

近年来,中国财政支出呈现稳定增长态势。但由于目前中国政府财政支出结构不完善,财政支出表现相对较低。从近年来的开支来看,经济建设和行政管理所用的地方财政金额差异较大,人员资金开支已成为主要部分。该支出严重限制了政府职能的正常履行,因此需要加强对政府财政资金的监管和分配。根据政府财政资源有限的基本国情,及时对财政监督财政开支,能够有效控制各级政府部门公共资金开支,提升公共资金业绩。

(1)审查范围逐渐扩大。

21世纪初,政府有关机构及其下属单位的业绩审计提上日程。审计机构审计重点从以前被审计机构财务收支的客观性和合法性转变为被审计机构财务资源使用的有效性和经济性。与此同时,2016年1-11月,国家审计机构完成了21334业绩审计项目。其中,审计局完成113个,省级审计机构完成847个,地级审计机构完成6288个,县级审计机构完成14086个。另外,各省市地方审计机构实施的绩效审计项目数量占审计项目总数的31%。其中,18个省(自治区、直辖市)和5个独立计划城市均超过这一平均水平。2017年1-11月,绩效考核工作量约占考核总工作量的一半左右,比2016年同期增长10%。

(二)加强政策支持力度。

近年来,审计局发布了多份文件,规划了业绩审计的发展方向,十三五国家审计工作的发展规划提出进一步加大业绩审计力度,始终贯穿审计工作,促进发展质量和利益的重大违纪违法问题的暴露和审查力度,促进廉政建设。与此同时,各省市也积极制定政府业绩审计相关政策,2011年徐州市审计部门发布《徐州市审计机关政府投资项目业绩审计操作指南》,规范徐州市业绩审计工作,2012年浦东新区人民政府发布《关于印发浦东新区业绩审计方法的通知》,对浦东新区政府部门业绩审计提出了实施要求。

(一)审计机构缺乏独立性。

从我国政府行政结构来看,我国各地市审计机构属于各地政府,受当地政府和上级审计机构的指导,受经费和行政管理模式的影响,业绩审计目标和具体任务的制定受当地政府的制约,阻碍审计作用确实有效地发挥,同时审计机构业绩审计活动完成后,审计机构发行的审计报告由同级地方政府和上级审计机构审计,不利于审计工作报告的完整性和客观性宝座,审计报告的真实性受到制约。另外,审计机构的组成人员审计机构本身只有提案的权利,其人员编制、人事选拔、任免、奖惩均由同级组织部和同级政府审查批准,在一定程度上阻碍了审计机构职能的正常发挥。

*河北省哲学社会科学计划办公室:ppp模式支持正确扶贫绩效评价研究(hb18gl058)。

(二)缺乏相关法律法规的指导。

我国政府业绩审计起步晚,审计工作的具体方法和程序尚不成熟,目前只有少量法律法规提到政府业绩审计,政府业绩审计工作开展的完整法律依据不足,现行《审计法实施条例》第二条指出,《审计机关依法独立检查被审计机关的会计证明书、会计账簿、财务会计报告书及其他与财政收支、财务收支相关的资料和资产,监督财政收支、财务收支的真实性、合法性和效益行为》,相关法律只进一步明确了该法律规定,但没有列出具体的审计内容和我国政府的具体规定。在这种背景下,审计相关人员只能通过现有的知识储存和现有的经验来探索,审计人员的工作水平也不同,业绩审计效果的正确客观性增加了一定的风险。(3)审计团队人才结构不合理。

完成政府业绩审计需要多个专业审计师团队。在此期间,中国审计机构主要从事传统财务审计。审计师的知识结构比较简单,工程技术、法律、宏观经济、对外贸易和计算机专家数量比较少。根据中国审计师知识结构的调查,会计和审计专家占所有审计师的71.8%。绩效考核要求考核广泛的经济和管理活动的有效性,要求多样化、创新的方法和多学科知识。有些审计师很难满足高标准的绩效审计。另外,审计员整体的数量也很少,难以适应大的任务量,水平越低的机构越明显。

(一)完善独立监督体制。

一是要进一步加强垂直管理改革力度。不仅将地方审计机关正职的任免权接受省级机关,普通审计人员也由省审计厅管理,招聘、提拔、任免由地方审计机关提名,由省审计厅和省人社会局决定,二是地方审计机关经费由省财政厅统一保证,纳入省财政预算。由此可以将人力资源和财务完全独立于地方政府和财政部门,提高独立性,使地方审计部门更有效地对地方政府领导进行审计监督,更有力地揭示和反映问题。

(二)健全相关法律制度。

由于缺乏明确的法律条款,我国政府业绩审计推广和拓展面临一定的困难。我国立法机关应进一步完善我国政府业绩审计相关法律制度。首先要完善审计报告的公开制度,通过将审计活动的各个阶段以审计报告的形式向公众展示。在完善审计报告制度的过程中,可以充分利用媒体作为中介的作用,及时准确地将审计结果传递给社会公众,扩大政府审计的影响力。其次,要不断健全审计结果问责制度,加强审计结果的应用,实现审计报告的公开与审计结果问责的有效联系。

(3)构建合理的评价标准体系。

随着政府职能的逐步完善,经济性和利益性也是政府业绩审计应该关注的重要内容。在预算绩效考核体系的基础上,审计机构应积极探索绩效考核考核标准,建立全国公认、适用、规范的体系,不断修改、完善。业绩评价体系应当包括预期的目标、决策过程、投入资源和能源、生产效益、对社会效益的影响等,评价指标应当坚持定量和定性的结合,定量指标主要指财务指标,定性指标的确定应以被审计项目的特点和被审计部门的管理特点为基础。

(四)改善政府业绩审计人员结构。

审计团队人员的综合素质和业务能力是政府业绩审计顺利进行的重要保障。审计机构不仅要提高现有审计人员的综合素质能力,还要不断引进高水平的审计人才,尤其是专业型的审计人员。要提高审计机构的准入门槛,完善审计人员的准入制度,确保招聘人员在综合素质和业务水平上能够满足政府业绩审计的要求,确保审计队伍中的人才能够胜任其职责。此外,审计机构人员不仅要包括专业审计人员,还要在实际绩效审计工作中招聘工科类、经济管理类等其他专业人员,不断优化审计人员结构。

全面优化我国政府业绩审计可有效促进我国审计持续健康发展,有利于推进社会主义法治社会建设,但结合政府业绩审计在我国起步晚、发展缓慢的现状,如何更好地开发和引导还需要进一步深入研究和探讨。

参考文献。

[1]齐兴利,绍辉.中国政府业绩审计发展途径研究[j].审计和经济研究,2007.2。

[2]周亚荣.中国政府业绩审计理论研究与实践现状[j].审计与经济研究,2008.2。

[3]宋夏云,我国政府业绩审计师能力框架研究[j].会计研究,2013.4。

[4]曲明,中国政府绩效审计评价标准体系框架构建[j].财经问题研究,2016.5。

[5]闵晓蕾.政府业绩审计理论文献综述[j].财经论坛,2006.2。

摘要:政府信任反映了居民对政府的认可程度,是政府政策执行的基础。汶川的地震灾区居民政府信任水平没有随着灾区公共服务水平的提高得到相应的提升,通过运用“汶川地震十周年居民生活情况调查”数据,实证分析探究了人际信任、政府绩效与政府信任之间的关系,研究结果显示,政府绩效、人际信任均对政府信任产生显著的影响。

关键词:灾区居民;政府信任;人际信任;政府绩效。

2008年汶川8.0级特大地震给震区带来了巨大的财产和生命安全损失,我国政府在救灾所表现出的强大动员能力、应急能力也成为世界关注的焦点。尤其通过政府的各项震后恢复政策,使得汶川灾区的社会经济发展获得历史性机遇,基础公共社会高水平重构,产业发展能力也持续提升,百姓生活水平相对于震前有较大幅度提高。

较高的政府信任度会有效降低政府运行的成本,于政府及公民都是有利的。而政府信任会受一系列因素的影响,在突发事件发生的关键时期政府政策执行对政府信任的影响更为显著。政府信任作为公民对政府的主观评价,受诸多因素的影响。现有的研究中不乏对政府影响因素的探讨,程倩认为,政府信任是政府合法性的本质内涵,政府信任关系是公民对政府及其政策执行过程中各相关要素、政策执行过程以及政策执行结果的心理预期。在二者的关系解释上,政府高校的运作过程以及较好的绩效水平可以带来良好的政府信任水平,两者呈正相关关系(程倩,2011)。缪婷婷认为,影响政府信任的因素具体包括政府作为、政府绩效以及新闻媒体的兴起(缪婷婷,2016)。在关键时期,政府政策对居民的政府信任影响更为显著。在突发事件中,公民对政府的信任感往往受领导者的价值观以及政府政策执行情况的影响,政府信任与政府及其工作人员对受灾民众的真诚关怀和激励呈显著正相关的影响(王璐,2008)。

因此,本文在梳理了现有研究成果的基础上,基于“汶川地震灾区灾后重建十周年状况调查”数据,对影响灾区居民政府信任度的影响因素进行实证分析。

2.1人际信任。

学校公众信息撰写心得体会

随着信息时代的到来,学校的公众信息传播越来越重要。作为一名学校信息部的成员,我有幸参与了学校公众信息的撰写工作,通过这一过程,我不仅提高了自己的写作能力,也深刻领悟到了信息传播的重要性。在此,我想分享一下我在学校公众信息撰写中的心得体会。

首先,在公众信息撰写中,准确的信息是最为关键的。作为舆论的传播者,我们有责任确保所传播的信息是真实准确的。因此,在撰写公众信息时,我们要在多个渠道获取信息,尽量亲自采访相关人士,确保获得的信息权威可信。此外,我们要注重信息的时效性,及时更新信息以满足读者的需求。准确的信息不仅能够保证信息的有效传播,也能够增强公众对学校的信任度。

其次,在公众信息撰写中,语言表达的规范性和清晰度很重要。信息部作为学校的宣传窗口,我们的信息撰写既要确保行文规范,又要使读者能够迅速理解。为此,我们要避免使用过于专业的术语,尽量用通俗易懂的语言表达。此外,我们还要注重文风的稳定统一,以保持整体的风格一致性,提升信息的品质。

同时,在撰写公众信息时,传递正能量也是非常重要的。作为一名学校信息部的成员,我们要时刻牢记自己是为学校传递信息的使者,我们的言行会影响到学校的形象与声誉。因此,我们要尽量选择一些正能量的新闻事件进行撰写,鼓舞人心、激发学生的积极性。我们要善于发现学校中的先进人物和事迹,积极宣传他们的优秀品质,让更多的人了解他们,激发学生的追求卓越的热情。

此外,在公众信息撰写中,与读者的互动也是至关重要的。我们要时刻关注读者的阅读需求和反馈意见,根据读者的反馈及时调整信息的撰写方式和内容。通过与读者的互动,我们可以了解到读者的需求和期望,从而更好地提供符合读者需求的公众信息。同时,我们还可以借助社交媒体等渠道进行信息传播,与读者进行更直接的互动交流。

最后,在公众信息撰写中,团队合作是不可或缺的。作为学校信息部的成员,我们要时刻与其他成员进行有效的沟通与协作,互相激发创意,相互监督,共同提高。更重要的是,我们要注重团队建设与培养团队合作精神,鼓励成员之间相互学习,共同进步。只有通过团队的合作,我们才能够把学校的公众信息撰写工作做得更好,传递出更多有效的信息。

总之,学校公众信息撰写对于传递学校形象和宣传精神非常重要。在学校公众信息撰写中,我们要注重准确的信息、规范的语言表达、传递正能量、与读者的互动和团队合作。通过不断的学习和实践,我们可以提高自己的写作能力,同时也可以为学校传播正面健康的信息,促进学校的良性循环。相信在不久的将来,我们的公众信息撰写工作将会越做越好!

成都市青白江区公众信息网

1四川义和团运动的主要领导人是,6月17日,与前来搜捕的华阳县团练发生冲突,揭开了川西义和团武装起义的序幕。

2先后镇压四川义和团的总督分别是。

3威震川西的义和团女英雄就义时,年仅。

4、1888生于今青白江区,早年曾留学。1906年,在日本东京加入中国同盟会,开始秘密从事反清活动。

5了不朽功勋,被临时大总统孙中山赞为“我老彭收功弹丸”,追赠大将军衔。

6、1980年被列为2003年,被定为育基地。7曾留法勤工俭学的是彭鸿章,进步**的发起人是赖明果。

81927年,担任国民革命军营长的曾学圃加入中国共产党。

9新中国成立前夕,被杀害于成都十二桥的烈士是。1949年12月,被杀害于成都十二桥的烈士是吴惠安。被杀害于成都王建墓的烈士是刘仲宣。

10从青白江区走出去的我党早期革命家是,1928年3月,他与许权忠、刘志丹、谢子长等,组织领导了渭化起义。

11抗日战争初期,青白江区牺牲的三位营长是他们分别在南京保。

卫战、南口战役和武汉会战中英勇牺牲。

12、1938年,在城厢家珍公园内建立的抗日阵亡将士纪念碑,是为了纪念在抗日战争中牺牲的易明道和沈德怡两位烈士。、1936年秋,中共地下党员主任兼上史地课,向学生宣传抗日救国思想。

14、1937年秋,在城厢公园邓公楼召开军事会议,决定出川抗日;被誉为日本空军的克星。

15新中国成立前,城厢镇的伞远近闻名。

16城厢中学(原金堂县立中学)创办于1927年,第一任校长是。

17抗日战争时期,内迁至青白江区姚渡镇曾家老寨,铭贤学校从中学部到大专部在青白江区办学长达年之久。《园艺学庭院学》的作者是何泽周。1981年,长期从事教育工作的江区副区长。

19.开凿于清代坐落在青白江区人和乡的崟华祖师石刻摩崖造像、张家兵洞均有一定艺术价值。20在近代青白江区,21近代青白江区著名的革命活动家有加入中国民主同盟会的林伯渊和加入中国国民党革命委员会的何梁。

22、近代金堂县军阶最高的一个军人是余锦源。

以“政府绩效与公众信任”为撰写小论文范文

虽然我国已经开始认识到政府部门服务能力的重要性,并且采用了多种方式不断提高政府部门的服务水平。然而,在进行服务型转变的过程中,也需要注重以人为本的理念。目前,很多政府部门在应用以人为本理念的过程中,依然存在问题,主要体现在以下几点:

1、思想认识方面存在不足。

为了更好地在政府绩效管理过程中应用以人为本的理念,需要对相关理念有着深入认识,充分挖掘以人为本理念的内涵。但是,目前很多政府部门对以人为本理念的思想认识还存在不足:首先,很多政府部门依然没有认识到政府绩效评价体系的重要性,认为绩效管理体系无法取得应有的效果,而且还浪费人力物力,对提高政府部门的服务水平并无益处。其次,我国受传统思想文化的影响,官僚主义和封建主义的影响严重,导致很多政府部门并没有重视以人为本理念,在进行政府部门办公的过程中,依然采取高高在上的态度,导致民众投诉政府部门工作态度的事件频繁发生。最后,很多政府部门在进行绩效管理的过程中,过分地重视自身的利益,如果以人为本理念没有触及到政府工作人员的利益,那么工作人员并不重视以人为本理念的普及和应用,从而导致政府绩效管理在发展过程中遇到瓶颈。

2、政府绩效管理无法体现人文关怀。

目前,虽然很多政府部门为了积极响应国家的号召,在进行绩效管理过程中,将以人为本的理念应用其中,但是却仅仅停留在表面,没有更好地体现人文关怀。主要表现在:第一,很多政府部门为了更好地应用以人为本理念进行绩效管理,在对政府部门工作人员进行绩效考核时,往往利用不公平的手段强制要求民众对其工作水平进行认可,导致考核数据不准确,考核不公正的现象发生,无法更好地体现绩效考核的公平、公正、公开理念。第二,很多政府部门将绩效管理流于表面,虽然对工作人员进行绩效考核,但是考核的结果基本上都是内定的,因此无法更好地提高工作人员工作的积极性,同时也无法体现以人为本的理念。

3、加重政府或民众的负担。

在政府绩效考核的过程中加入以人为本的理念,如果应用不合理很可能会造成政府或者民众负担的情况出现:第一,目前,我国很多政府部门都是非营利性的组织,政府本身在推动绩效管理或者以人为本理念落实的过程中动力不足,如果强制使用以人为本理念,可能会加重政府工作的负担;第二,如果政府部门加重负担,那么很可能将这部分负担转嫁到普通民众的身上,导致民众使用政府工作的成本增高,也不利于普及以人为本理念应用到政府绩效的管理工作中。

综上,以人为本理念在政府绩效管理渗透过程中依然存在很多问题。因此,本文在分析了这些问题成因的基础上,进而提出相应的完善策略,主要体现在以下几个方面:

1、提高以人为本理念的重视程度。

以人为本理念能够更好地提高政府绩效管理效果,所以必须要提高政府部门对以人为本理念的重视程度:首先,需要对政府相关部门人员进行思想教育,使其充分认识到以人为本理念的重要性,学习以人为本理念的深刻含义,更好地将其应用到政府部门的工作中,从而提高政府部门的办事效率,更好地为民众提供相关服务;其次,政府绩效机制不仅仅关系到政府部门的未来发展,而且也对民众的生活有着重要的影响,因此必须要充分落实以人为本理念在政府绩效管理的重要程度,从而更好地树立政府部门“正直、廉洁、公正”的形象。

2、需要体现民众的根本利益。

以人为本理念应用到政府绩效管理中,不仅需要保持绩效考核的公正性,而且还需要体现民众的根本利益,可以参考以下几个方面:第一,政府部门是为人民群众服务的部门,因此必须要重视人民群众的利益,所以在为人民群众提供服务的同时,需要重视群众的意见,因此需要建立政府绩效的反馈机制,使人民群众参与到政府绩效的考核过程中,接受群众对政府服务的监督和反馈;第二,政府需要改变以往的官僚主义思想,充分认识到群众的位置,坚定政府部门的服务地位,切实维护好人民群众的根本利益。

3、促进人的全面发展。

以人为本理念应用到政府绩效管理的部门中,需要促进人的全面发展:第一,确保政府绩效管理的公平公正,从而能够对政府部门相关人员的工作内容进行科学合理地评价,对其长处进行表扬,对其不足予以指正,从而能够更好地促进政府部门人员工作能力和服务能力的提高;第二,政府绩效管理人员应该认识到,采用绩效管理的目标是为了更好地为群众服务,提高政府部门的服务水平,提高政府人员的服务水平,避免因为绩效考核损害到政府工作人员的工作积极性,切忌过度损害工作人员的相关利益。

4、鼓励群众参与政府绩效管理。

人民群众是政府的服务对象,因此有权利也有责任对政府部门的服务效果进行评价,所以为了更好地体现以人为本的理念,需要鼓励群众参与政府绩效管理:第一,当群众去政府部门中进行事务办理时,可以对服务人员的工作能力和工作态度进行评价,从而能够更好地从群众角度了解到政府部門的工作程度,真正了解群众关心的内容;第二,建立群众意见反馈渠道,当群众在办理事务时,如果遇到不懂的地方,或者是觉得不合理的地方,可以通过特定渠道进行反馈,政府部门需要积极归纳群众的意见,对合理的意见进行采纳,从而能够更好地提高政府部门的服务水平。

综上所述,我们不仅看到了目前政府绩效管理工作中存在的各类突出问题,更意识到了以人为本理念在政府绩效管理工作中渗透的重要意义。为了更好地打造服务型政府部门,我们需要对政府相关人员的工作进行评价和管理,并坚持以人为本的理念,使得评价的结果公平、公正、合理,更好地提高政府部门的工作成效。与此同时,作者也希望借助本课题的研究,在社会上引起人们对政府绩效管理工作创新改革的深刻关注,促使更多的社会专家、学者广泛的参与到本课题的研究中来,有效推动服务型政府的建设,促使政府绩效管理工作朝着更好的方向发展。

【参考文献】。

[2]李彦娅,何植民.基于科学发展观的地方政府绩效评估体系构建研究[j].人才资源开发,2007(06)9-10.

以“政府绩效与公众信任”为撰写小论文范文

摘要:自从分税制模式实施后,出现了中央与地方政府财权和事权不匹配的问题。2008年金融危机对我国的经济影响非常大,于是我国扩大内需,推出了4万亿元的投资计划。很多地方政府融资平台开始迅速扩张,扩大自己的融资规模。然而这种现象也产生了负面影响,各地政府实施不同的项目,导致政府债务规模不断扩大,政府债务风险严重。鉴于此,对政府财务风险进行分析,并提出解决政府财务风险的对策。

关键词:政府债务;财务风险;防范对策。

我国从1994年开始进行分税制改革,导致政府的财权和事权不统一,政府支出的责任变更非常频繁,但是没有相应的财力作为支撑。2008年,我国为了应对金融危机,提出了4万亿政策,结合中央政府的投资计划,地方政府开展了各种形式的融资。大规模的融资导致债务隐性化,债务风险非常严峻。在党的十九大报告中,明确提出要防范各类风险。在风险防控中,政府的债务风险防控是重点,从而有效地预防金融风险的发生。

1.政府财权事权与支出责任不符。自从1994年我国实施分税制改革后,税种划分成中央税种和地方税种。地方的事权不断增多,导致地方性政府财政支出越来越多。随着城镇化进程的加快,居民对公共产品的需求量非常大,政府需要更多的资金。地方政府为了进行基础设施建设,他们每年都要融资,产生债务风险。地方政府的财政收入非常有限,受到中央强化房地产调控的影响,在转移支付环节中存在各类问题,不能结合地方政府的实际需求。在增量调节环节,地方之间的贫富差距非常大。地方政府的事权不断增加,但是财权非常有限。地方政府体制内的收入不能弥补财政支出,导致地方政府的预算外收入增加,他们只能通过举债的方式弥补财政不足。

2.政府融资不够规范。政府在融资环节中,产生非常大的随意性。地方政府常常通过发行债券的方式,但是资金还是不能满足需求。于是,地方政府开始建立各类融资平台,在融资环节中产生盲目性和随意性。政府的融资运作效率不高,融资的成本非常高,导致政府投资效率不佳。在融资平台建设中,资金不足。政府会通过各类方式补充资金,通过挪用年度预算等方式,甚至出现虚假注资的方式。政府在融資方面,相关的管理体制存在缺陷,很多融资平台的建设没有相关的监督措施加以辅助,采用多头管理的方式。融资监督机制尚未建立,政府在融资环节中,缺乏法律和公众的监督,导致商业腐败的产生。

3.政府债务管理机制不够健全。政府的债务管理机构不完善,尽管我国政府已经积累了很多政府性债务,但是债务管理机构尚不完善。债务资金使用不够规范,有些地方政府的债务资金不能纳入到预算管理环节,债务资金不能充分进行基础设施建设。债务资金管理效果不好,不能进行全方位的预算管理。政府债务领域的信息不够透明,地方政府的融资情况不能及时地回报给人大常委,信息不公开,导致地方性债务的口径不能完全的纳入到预算管理系统中。债务管理的法制建设比较落后,不能结合债务管理建立法律体系。尽管我国制定了《预算法》等,对政府的举债行为加以约束,但随着社会经济的发展,政府投资还是越来越多,已经超过政府的财力。

1.提升思想认识,强化债务管理。政府应强化风险认识,防止债务风险扩散为经济风险,要正确地认识债务风险问题,纵观大局,做到未雨绸缪。政府应树立忧患意识,认识到抵御债务风险的紧迫性,也要认识到政府举债对当地经济发展产生的影响,确保偿债有序。强化底线意识,底线意识在一定程度上是底线思维的应用,其与风险意识是相辅相成的,政府要在对现实情况进行分析的基础上,提高警惕。将国家法律作为“红线”,做好风险的防范准备工作。政府还要提升自身的法治意识,杜绝违法违规的举债行为。有些政府的领导干部,他们的法治意识比较弱,对于一些惩戒视而不见。因此,政府应树立法治意识,规范举债行为,维护法律的权威。充分了解政府职能的权限,掌握政府和市场之间的关系,强化政府职能的转变,发挥市场这只无形手的资源配置作用。

2.推进体制机制改革。合理划分财权和事权,中央政府和地方政府应该明确财政事权和支出责任,有效地抑制政府债务规模。结合权、责和利相统一的原则,各地区的财权和事权应得到明确,制定财权事权调整机制,形成完整和清晰的财权和事权清单,从而提升政府对于财权和事权改革的可操作性。建立相应的配套措施,完善相关的改革协同配套方案,提升转移支付制度的可操作性。进行干部任用制度的改革,完善债务监督考核问责制度。对考核的方法进行改进,抓住基础和发展的根基,充分发挥潜能,改善民生,促进社会进步。完善政府债务考核和问责制度,建立公共服务为中心的理念。在新政策背景下,完善债务监督制度,对金融机构进行科学的监管工作,有效地杜绝违法违规操作,强化责任人制度。对政府债券资金进行全面管理,明确责任主体和财务主体。提升债务信息的透明度,通过政府政务网站的方式,对债务限额明确,建立债务限额报告制度。通过财政预算的方式,对预算情况进行分析。

3.提升政府债务风险管理能力。建立政府债务管理小组,发挥其职能。对政府债务的借、用和还阶段进行监督,进行债务管理工作的指导,对于债务风险要实时掌握。当风险发生后,应采取必要的干预措施。完善政府债务的预算管理工作,分别对政府债务收入、支出、付息和还本进行预算,防止超预算举债的发生。各地的人大应发挥审议作用,充分履行监督职能。提升风险化解的能力,在政府债务管理中,隐性债务管理存在一定的难度。因此,在债务管理中,要对各类风险进行评估,出具评估报告。有些政府债务风险防范能力比较差,风险防控机制比较落后。因此,要提升政府债务风险化解能力,抓住风险产生的主要因素。

政府债务风险是重大风险的类型之一,因此,我国应有效地防范和化解政府财务风险,促进全面小康社会的建设。同时,政府应提升债务风险的化解能力,树立危机意识。

参考文献:

[2]胡才龙,魏建国.多任务委托代理模型下地方政府债务管理激励契约设计——基于省级面板数据的实证检验[j].审计与经济研究,2019,(5):118-127.

学校公众信息撰写心得体会

第一段:引言(200字)。

  随着社会的进步和信息技术的快速发展,学校公众信息的撰写和传播已经成为现代教育的重要组成部分。作为一名学生,在参与学校公众信息撰写的过程中,我收获良多。这不仅提高了我的写作能力,还培养了我的团队合作和沟通能力。在这篇文章中,我将分享我在学校公众信息撰写中的心得体会。

第二段:提高写作能力(200字)。

  学校公众信息的撰写对我的写作能力起到了很大的促进作用。在撰写学校公众信息时,我们必须做到信息准确、简洁明了,让读者能够迅速了解到我们要传达的信息。这需要我锻炼自己的文字表达能力,提高写作水平。除了文字表达,写作还需要有良好的结构和逻辑,让读者能够顺畅地理解和接受信息。通过不断的实践和反思,我的写作能力得到了有效的提高。

第三段:培养团队合作能力(200字)。

  学校公众信息的撰写通常是由一个团队合作完成的。在团队中,每个人都有自己的任务和职责,需要相互配合,共同完成任务。在这个过程中,我学会了更好地与他人合作,明确分工、沟通交流、协调资源。这不仅提高了我的团队合作能力,还培养了我的组织协调能力和人际交往能力。同时,学会倾听他人的建议和意见,灵活调整自己的观点和做法,让团队更加高效地完成任务。

第四段:注重信息传递效果(200字)。

  在学校公众信息撰写过程中,我深刻体会到信息传递效果的重要性。一份好的公众信息不仅要准确传达信息,还需要吸引读者的注意力,让他们愿意阅读并理解。因此,撰写公众信息时必须注重选择合适的语言和表达方式,用简洁明了的语言描述问题,使受众能够快速、准确地理解信息的核心内容。同时,结合不同受众的特点和需求,采取合适的传播渠道和形式,提高信息的传播效果。

第五段:总结(200字)。

  通过学校公众信息的撰写,我不仅提高了写作能力,还培养了团队合作和沟通能力,注重信息传递效果。这些对我日后的学习和工作都将有着重要的影响。同时,我还意识到学校公众信息的撰写并不是一个简单的过程,需要不断实践和积累经验。我将继续努力,不断提升自己在学校公众信息撰写方面的能力,为学校提供更加优质的公众信息服务。

相关范文推荐
  • 01-05 倔强的小红军电影心得体会(优质19篇)
    写心得体会是一种对自己成长的记录和见证,可以让我们更好地领悟人生的真谛和价值。以下是一些精选的心得体会范文,希望对大家的写作有所启发和帮助。第一段:引言(100
  • 01-05 科室护理工作总结(实用21篇)
    月工作总结不仅有助于个人的发展,也是团队进行评估和调整的基础,为整个团队的发展提供支持和指导。以下是一些成功人士的月工作总结,他们的写作风格和思考角度可能会给我
  • 01-05 攀岩课的心得体会(优秀17篇)
    在这段时间里,我深刻感受到了学习的重要性,只有不断地学习,才能不断地进步。接下来是一些精选的心得体会范文,希望能够对大家的写作提供一些帮助。“啦啦啦啦啦啦啦”节
  • 01-05 环保工作会议记录与研究范文(18篇)
    环境保护是可持续发展的基石,我们需要重视并付诸行动。这些总结范文是对过去环保工作的一个完整回顾和总结,可以让我们更好地评估工作的效果和成果。11月30日下午,桃
  • 01-05 高中毕业的个人鉴定(模板20篇)
    通过研究范文范本,我们可以学习到各种总结的写作方法和技巧,从而提高我们的写作水平。在此,小编为大家收集了一些经典的范文范本,供大家学习参考。三年来,学习上我严格
  • 01-05 小学数学认识面积听课心得(汇总18篇)
    心得体会是我们对自己行为、思考和感悟的总结,可以帮助我们更好地认识自己和改进自己。在下面的内容中,我们将会看到一些关于心得体会的精彩范文,让我们引以为戒并学习他
  • 01-05 变频器实训个人心得体会报告(实用18篇)
    心得体会是对自己过去的经历和成就进行总结和回顾,为未来的发展提供借鉴和指导。下面是小编整理的一些关于心得体会的范文,供大家参考和学习。课程是会计学专业的主干课程
  • 01-05 活动专员心得体会范文(19篇)
    心得体会是个人的思考和感悟的结晶,体现了个体对经历和困惑的理解和思考。以下是小编整理的一些优秀心得体会范文,供大家互相学习和借鉴。第一段:引言(介绍活动专员的工
  • 01-05 非公党建会议记录(优秀18篇)
    范文范本的阅读可以激发我们对写作的兴趣,提高我们的写作积极性。范文是指在某个领域或者某个主题下已经成型且具有代表性的文本,它可以作为写作参考或者学习的榜样。如何
  • 01-05 自律心态心得体会(优质19篇)
    心得体会不仅是对个人经验的总结,也是对问题和困惑的思考和解决。小编整理了一些精选的心得体会样本,希望能够对大家的写作提供帮助。自律是指个体在行为、思维和情感方面